Revista (format .pdf, 4.1 MB)

98
RECREA Ţ II MATEMATICE REVIST Ă DE MATEMATIC Ă PENTRU ELEVI Ş I PROFESORI Seminarul Matematic “A. Myller” (1910 – 2010) Asocia ţ ia “Recrea ţ ii Matematice” IA Ş I - 2010 Ianuarie Iunie 2010 Anul XII, Nr. 1 1 = π i e

Transcript of Revista (format .pdf, 4.1 MB)

Page 1: Revista (format .pdf, 4.1 MB)

RECREAŢII MATEMATICE

RE V IS TĂ DE M AT E M AT I CĂ P E NT R U E L E V I Ş I P R O F E S O R I

Seminarul Matematic “A. Myller”

(1910 – 2010)

A s o c i a ţ i a “ R e c r e a ţ i i M a t e m a t i c e ” I AŞ I - 2 0 1 0

Ianuarie – Iunie 2010Anul XII, Nr. 1

1−=πie

Page 2: Revista (format .pdf, 4.1 MB)

Semnificaţia formulei de pe copertă: Într-o formă concisă, formula 1−=πie leagă cele patru ramuri fundamentale

ale matematicii: ARITMETICA reprezentată de 1 GEOMETRIA reprezentată de π ALGEBRA reprezentată de i ANALIZA MATEMATICĂ reprezentată de e

Redacţia revistei :

Petru ASAFTEI, Dumitru BĂTINEŢU-GIURGIU (Bucureşti), Temistocle BÎRSAN, Dan BRÂNZEI, Alexandru CĂRĂUŞU, Constantin CHIRILĂ, Eugenia COHAL, Adrian CORDUNEANU, Mihai CRĂCIUN (Paşcani), Paraschiva GALIA, Paul GEORGESCU, Mihai HAIVAS, Gheorghe IUREA, Lucian-Georges LĂDUNCĂ, Mircea LUPAN, Gabriel MÎRŞANU, Alexandru NEGRESCU (student, Iaşi), Gabriel POPA, Dan POPESCU (Suceava), Florin POPOVICI (Braşov), Maria RACU, Neculai ROMAN (Mirceşti), Ioan SĂCĂLEANU (Hârlău), Ioan ŞERDEAN (Orăştie), Dan TIBA (Bucureşti), Marian TETIVA (Bârlad), Lucian TUŢESCU (Craiova), Adrian ZANOSCHI, Titu ZVONARU (Comăneşti)

COPYRIGHT © 2010, ASOCIAŢIA “RECREAŢII MATEMATICE” Toate drepturile aparţin Asociaţiei “Recreaţii Matematice”. Reproducerea integrală sau

parţială a textului sau a ilustraţiilor din această revistă este posibilă numai cu acordul prealabil scris al acesteia. Se consideră că autorii materialelor trimise redacţiei revistei sunt, în mod implicit, de acord cu publicarea lor, îşi asumă responsabilitatea conţinutului lor şi cedează Asociaţiei “Recreaţii Matematice” dreptul de proprietate intelectuală asupra acestora.

TIPĂRITĂ LA S.C. BLUE SIM & CO S.R.L. Bd. Carol I, nr. 3-5 Tel. 0788 498933 E-mail: [email protected]

ISSN 1582 - 1765

Page 3: Revista (format .pdf, 4.1 MB)

RECREAŢ I I MATEMATICE

RE V IS TĂ DE M AT E M AT I CĂ P E NT R U E L E V I Ş I P R O F E S O R I

Revistă cu apariţie semestrială

EDITURA “RECREAŢII MATEMATICE”

IAŞI - 2010

Anul XII, Nr. 1 Ianuarie – Iunie 2010

1−=πie

Page 4: Revista (format .pdf, 4.1 MB)
Page 5: Revista (format .pdf, 4.1 MB)

Centenarul Seminarului Matematic ”A. Myller”

Au trecut 100 de ani de cand, ın toamna 1910, Alexandru Myller, proaspatprofesor de geometrie analitica la Universitatea din Iasi, a avut initiativa crearii uneibiblioteci de matematica la aceasta universitate. Biblioteca a fost gandita, la ınceput,pentru uz personal. Destul de curand, ınsa, s-a permis accesul la publicatii si altorcadre didactice, biblioteca devenind astfel de uz comun. Prin aceasta, se ımbunatateaclimatul favorabil ridicarii calitative a activitatii de cercetare stiintifica ın univer-sitatile romanesti, creat de legea ınvatamantului, care l-a avut ca principal autor pematematicianul si astronomul Spiru Haret, pe atunci ministru al Instructiunii Publice.In Iasi, terenul fusese pregatit de activitatea vechilor profesori de matematica de laUniversitatea din Iasi: St. Emilian, N. Culianu, I. Melik, M. Tzony, C. Climescu,A. Manescu, I.D. Rallet, V. Costin. Dintre acestia, N. Culianu, C. Climescu siI. Melik, ımpreuna cu cativa profesori de liceu, contribuisera la aparitia la Iasi, pe operioada de 6 ani (1883-1888), a revistei Recreatii Stiintifice, precursoare a GazeteiMatematice. Au existat dificultati legate de eterna problema a absentei fondurilorsi de mentalitatea unor persoane aflate ın posturi de conducere ale tarii si Iasilor,care considerau ca nu este necesara o biblioteca de uz comun; profesorii interesati deanumite carti si reviste puteau sa si le procure pe cheltuiala proprie si sa-si constituiebiblioteci personale.

A fost nevoie de conceptia clara si tenacitatea tanarului profesor A. Myller,pentru ca o astfel de institutie sa ınceapa sa functioneze, sub denumirea de SeminarulMatematic, avand ca model biblioteca de acelasi tip de la Gottingen, Germania.

1

Page 6: Revista (format .pdf, 4.1 MB)

Mai ıntai cu un numar restrans de publicatii, s-a dezvoltat treptat, ajungand ca ıntrecele doua razboaie mondiale sa fie cea mai buna biblioteca de matematici din sud-estul Europei. Pe langa aceasta biblioteca, si-au desfasurat, mai mult sau mai putinsporadic, activitatea stiintifica si de cercetare mai multi tineri talentati, care aveausa devina nume cunoscute ın matematica romaneasca: C. Popovici, D. Pompeiu,V. Valcovici, S. Stoilow, S. Sanielevici, Gh. Vranceanu, Gr. Moisil, T. Popoviciu,O. Mayer s.a.

Obtinerea fondurilor pentru achizitionarea de publicatii stiintifice era o preocu-pare permanenta a lui A. Myller, care a condus biblioteca de la ınfiintare pana ınanul 1947. Aceste fonduri erau obtinute prin demersuri insistente pe langa forurileconducatoare ale ınvatamantului superior si ale culturii romanesti, prin convingereaunor persoane particulare sa faca donatii pentru biblioteca si prin contributii per-sonale. Mai ales ın perioada dintre cele doua razboaie mondiale, A. Myller plecaın mod regulat ın tarile din vestul Europei, unde colinda prin librarii si anticariate,facand achizitii masive de carti si de colectii de reviste, ajungand sa cheltuiasca si dinbanii sai. El insista mereu pe langa colegii sai sa contribuie la ımbogatirea biblioteciiprin publicatii si este cunoscut ca era tot timpul nemultumit, spunand ca membriiSeminarului Matematic nu fac suficient de mult pentru ımbogatirea patrimoniuluiacestuia. A. Myller a scris un manual de geometrie analitica (foarte bun!) pentruliceu, iar drepturile de autor au fost folosite ın ıntregime pentru cumpararea de cartisi reviste pentru Seminarul Matematic. Multi ani, aceasta carte a fost folosita pentrupremierea studentilor fruntasi de la Facultatea de Matematica. Unul din succeseleimportante ale lui A. Myller a fost convingerea industriasului N. Malaxa sa faca odonatie pentru Seminarul Matematic. Prin aceste eforturi, biblioteca SeminaruluiMatematic s-a ımbogatit cu colectiile celor mai prestigioase reviste de matematicaaparute ın lume, cu ultimele carti publicate la diverse edituri de specialitate din Eu-ropa si din America, precum si cu colectii complete ale unora din cele mai vechi revistede matematica (Acta Eruditorum, Crelle′s Journal, Journal de l′Ecole Polytechniquede Paris etc.) si cu carti vechi de matematica din perioada Renasterii si de mai tarziu.

O alta directie ın care a actionat staruitor A. Myller, a fost mentinerea uneidiscipline stricte ın Seminarul Matematic. Accesul la publicatii era liber, fiecaremembru putea sa ımprumute, fara sa apeleze la un bibliotecar (care, de fapt, nici nuexista), dar era obligat sa respecte cateva reguli simple ınsa stricte. Orice publicatieımprumutata era trecuta ıntr-un registru de ımprumuturi si se restituia la anumitedate, publicatiile trebuiau pastrate cu grija pentru a se ımpiedica deteriorarea saupierderea lor, nimeni nu avea voie sa plece din Iasi fara sa restituie mai ıntai toatepublicatiile ımprumutate. Unele abateri, rare, erau sanctionate fara rezerve de prof.A. Myller prin ridicarea dreptului de acces ın biblioteca pentru o perioada determinatasau chiar pentru totdeauna. Sanctiunile se aplicau la orice beneficiar gasit ın culpa,indiferent de vechimea si prestigiul acestuia. Acest spirit de disciplina instaurat laSeminarul Matematic a facut ca din biblioteca sa se piarda extrem de putine publicatii.

In 1944, Universitatea din Iasi a fost evacuata, ımpreuna cu biblioteca SeminaruluiMatematic, la Alba Iulia. Mutarea a fost organizata asa de bine, ıncat la ıntoarcereın 1945, nu a lipsit decat un singur volum din fondul de publicatii al bibliotecii.

Dupa razboi, au trebuit ınfruntate mari greutati rezultate din distrugerile cauzate

2

Page 7: Revista (format .pdf, 4.1 MB)

de luptele purtate pe teritoriul tarii (inclusiv la Iasi), izolarea ın care a intrat taranoastra, ca urmare a constituirii blocului statelor comuniste, si lipsa cronica a fon-durilor pentru achizitionarea publicatiilor. Chiar ın aceste conditii, ın biblioteca Se-minarului Matematic au continuat sa soseasca numeroase volume de reviste, obtinuteprin schimb cu revista Analele Stiintifice ale Universitatii ”Al.I. Cuza” Iasi, Mate-matica, carti trimise de diverse edituri pentru recenzare ın aceeasi revista si, ın micamasura, reviste obtinute prin abonament si carti achizitionate din fondurile oficiale,mai ales din fosta Uniune Sovietica.

Trebuie mentionate aici si numeroasele donatii generoase ale unor membri ai Se-minarului Matematic, constand din carti si reviste cumparate cu ocazia deplasarilorın strainatate.

In acest moment, exista un schimb cu circa 200 de reviste de specialitate. Inultimul timp, prin dezvoltarea internetulului, se pot accesa liber diverse carti si revistedin lume, precum si articole separate aparute ca preprinturi pe diverse site-uri speciali-zate (e.g. arXiv). Mai mentionam si unele initiative la nivel de tara care ne ofera accespe baza de abonament la diverse baze de date: Science Direct (unde se pot accesa,ın principal, publicatii de la editura Elsevier), SpringerLink (cu acces la pulicatiile dela editurile Springer si Birkhauser), Thomson ISI (Web of Science, Journal CitationsReport si Derwent Inovation Index), CSA Research Pack. Mai mentionez ca, prineforturi financiare deosebite ale facultatii, se asigura accesul on-line la MathematicalReviews si Zentralblatt MATH.

Biblioteca Seminarului Matematic nu este importanta doar ca sursa de informarepentru cercetatorii ın domeniul matematicii sau din alte domenii ınrudite. In salileei domneste o atmosfera calma si sobra, care ındeamna pe cel care intra ın bibliotecacatre meditatie si cercetare. In ultimii ani, la mesele din aceste sali s-au documentatsi au facut descoperiri matematice importante numerosi cercetatori care au devenitcunoscuti ın matematica mondiala. Unele din rezultatele lor au aparut ın reviste despecialitate de prestigiu din tara si din strainatate sau au fost ıncorporate ın monogafiimatematice scrise de autori din Iasi sau din alte centre stiintifice din lume.

Acum, la 100 de ani de la ınfiintare, Seminarul Matematic din Iasi, care poartanumele fondatorului sau, A. Myller, ısi traieste o noua tinerete.

Prof. dr. Vasile OPROIUDirectorul Seminarului Matematic ”A. Myller”

3

Page 8: Revista (format .pdf, 4.1 MB)

Seminarul Matematic din Iasi - 100 de anide ınvatamant matematic romanesc

In acest an, un secol de existenta a Seminarului Matematic se suprapune ınmod fericit cu 150 de ani de la crearea la Iasi ın 1860 a primei universitati din tara.

Invatamantul matematic iesean avea o oarecare traditie ınca ınainte de inaugu-rarea Universitatii, dar un adevarat centru de cercetare matematica a fost initiatde profesorul Alexandru Myller ın 1910, cand tocmai primise postul de titular lacatedra de geometrie analitica a Facultatii de Stiinte din Iasi.

In acelasi an, profesorul Myller a facut demersuri catre ministrul instructiuniipublice de atunci - Spiru Haret - ın vederea obtinerii de fonduri pentru dezvoltareaunei biblioteci de specialitate. Initial s-a primit o suma cu care s-au achizitionatcateva reviste matematice mai importante, iar mai tarziu s-a obtinut chiar o alocatiebugetara fixa.

Astfel, actul de nastere al Bibliotecii Seminarului Matematic (initialBiblioteca Seminarului de Geometrie analitica) putem spune ca a fost semnat la18 octombrie 1910, cand Alexandru Myller a ınregistrat oficial primele o suta devolume din ”Crelle′s Journal fur de reine und angewandte Mathematik”.

La ınceput, spatiul destinat bibliotecii era doar o fosta sala de curs adminis-trata numai de profesorul Myller, ınsa ın timp spatiul s-a extins, iar personalul deadministratie a crescut prin alaturarea unor membri ai Seminarului.

Profesorul Myller concepe Biblioteca Seminarului Matematic dupa modelul saliide lectura a Seminarului Matematic de la Gottingen unde a studiat, sustinand siun doctorat sub ındrumarea celebrului matematician David Hilbert. Cu un corpprofesoral de calitate si cu o biblioteca de valoare, s-a format la Iasi o adevaratascoala de matematica cunoscuta sub numele de Seminarul Matematic din Iasi (S.M.I.),avand ca baza pentru cercetare o biblioteca de specialitate realizata cu entuziasmulsi sacrificiul profesorilor si care ın perioada interbelica era considerata cea mai marebiblioteca din estul Europei.

Dupa evacuarile la Alba Iulia, ın timpul celui de-Al II-lea Razboi Mondial,biblioteca a revenit la Universitate fara pierderi. A urmat o perioada foarte greade refacere a ınvatamantului universitar. La 14 octombrie 1944 Alexandru Myllereste numit rector al Universitatii, iar ın noiembrie 1945 se retrage din aceasta functie,ramanand profesor activ pana la iesirea la pensie din noiembrie 1947.

In 1949, ın anul urmator Reformei ınvatamantului, cand toate bibliotecile fa-cultatilor au devenit filiale ale Bibliotecii Universitare din Iasi, care s-a transformatın Biblioteca Centrala Universitara, si Biblioteca Seminarului Matematic a devenit larandul sau filiala, iar ın 1954 a primit numele initiatorului sau, Alexandru Myller.

Amintim ca, desi Biblioteca Seminarului Matematic a devenit subordonata Biblio-tecii Centrale Universitare, la conducerea sa, din partea Facultatii de Matematica, auurmat dupa profesorul Myller urmatorii directori: prof. Ilie Popa ın perioada 1947-1952; prof. Adolf Haimovici ın anii 1952-1992; prof. Gheorghe Bantas ın anii1994-2006, iar din anul 2006 prof. Vasile Oproiu.

Daca pana ın 1952 toate activitatile erau facute de profesorul Myller si o partedin membrii S.M.I., dupa aceea a fost numit si un bibliotecar din partea B.C.U., ın

4

Page 9: Revista (format .pdf, 4.1 MB)

persoana d-rei Anca Jacota, licentiata ın limba franceza.Membrii Seminarului, ın prezent aproape 150, sunt ın special de la Facultatea de

Matematica a Universitatii ”Al.I. Cuza”, de la catedra de matematica a UniversitatiiTehnice ”Gh. Asachi”, dar si de la alte facultati si institutii de cercetare din tarasau strainatate. Ca utilizatori ai bibliotecii mai sunt, la recomandarea membrilorSeminarului Matematic, si beneficiari externi, majoritatea studenti ın ultimii ani aiFacultatii de Matematica sau doctoranzi.

Cu o asezare alfabetica a majoritatii fondului de carte, Biblioteca SeminaruluiMatematic are peste 81.000 de volume aflate aproape ın totalitate ıntr-o baza de dateinformatizata, iar ıntre documentele bibliotecii se afla, pe langa un fond de cartecurenta, dictionare si enciclopedii generale si de specialitate, reviste de referate, siun fond de carte veche si manuscrise cuprinzand aproximativ 300 de volume aparuteınainte de 1850. Dintre acestea din urma amintim:

• Apollonii Pergaei Conicorum libri quator, 1566;• Gemma Frisius, R. - Arithmeticae Practicae methodus facilus, Lipsiae, 1575;• Clavius Bambergensis, Christophorus - Geometria practica, Mainz, 1606;• La Hire, Philippe de - Nouveaux elements des section coniques, Paris, 1679;• Ozanam, Jacques - Dictionnaire mathematique, Amsterdam, 1691;• Abrege des mathematiques pour l′usage de Sa Majeste Imperiale de toutes les

Roussies, St. Petersbourg, 1728;• Simson, Robert - Sectionum conicarum libri V, Edinburg, 1735;• Saverien, Alexandre - Dictionnaire universel de mathematique et de physique,

Paris, 1753;• Garnier, J.G. - Reciproques de la geometrie, Paris, 1810;• Stoiheia arithmetikes, Iasi, 1818 (carte romaneasca veche).De asemenea, ın biblioteca exista operele complete ın original ale marilor mate-

maticieni si fizicieni cum ar fi: Euclid, Galileo Galilei, Johannes Kepler, PierreFermat, Christian Huygens, Isaac Newton, Jacob Bernoulli, LeonhardEuler, Joseph Louis Lagrange, Pierre-Simon Laplace, Karl Friedrich Gauss,Augustin Louis Cauchy, Niels Henrick Abel, Henri Poincare etc.

Alaturi de colectiile de carti se afla si peste 700 de titluri de periodice de speciali-tate, iar dintre cele cu aparitie mai veche amintim: Acta Eruditorum (1682); Journalde l′Ecole polytechnique (1797); Journal fur die reine und angewandte Mathematik(1829); Compte Rendus de l′Academie des Sciences (1835); Bulletin de l′AcademieRoyale des Sciences (1836); Proceedings of the Royal Society of London (1862); Mathe-matische Annalen (1869); American Journal of Mathematics Pure and Applied (1878);Acta Mathematica (1882); Recreatii Stiintifice (1883); Bulletin of American Mathe-matical Society (1895); Transaction of American Mathematical Society (1900);Annales Scientifiques de l′Universite de Jassy (1900) etc.

In acest loc ıncarcat de istorie, prin care au trecut si s-au format atatia oamenide seama, avem datoria fata de ınaintasi, cat si fata de cei care vor veni, sa pastrammereu, ımbinand traditia cu modernitatea, acest tezaur de cultura.

Andrei PATRASBibliotecar sef al Seminarului Matematic

5

Page 10: Revista (format .pdf, 4.1 MB)

Amintiri de la Seminarul Matematic

Am intrat ca student la Facultatea de Matematica de la Universitatea din Iasiın septembrie 1959. Dupa ce am facut cunostinta cu personalul de la secretariat,unde, ca sef de grupa, trebuia sa predau saptamanal prezenta studentilor la cursuri siseminarii, am ajuns si la biblioteca facultatii, situata deasupra decanatului facultatiide matematica. Imi amintesc de o sala luminoasa si primitoare, ın care se putea studiaın conditii foarte bune. Exista si o bibliotecara, d-ra Timofte, care statea ıntr-o salitavecina, situata deasupra cabinetului decanului, prin care se trecea pentru a se intraın sala de lectura, dupa ce se semna ıntr-o condica care atesta numarul de vizitatori.Studentii din anii mari aveau o alta sala de lectura alaturi, deasupra amfiteatrului II.5.Aveau cheie si puteau sa intre cand doreau (aici nu exista o bibliotecara), gaseau osala de lectura care ıi ımbia la studiu, gaseau o mica biblioteca cu carti dintre cele maifolosite la facultatea de matematica. Multa vreme am privit aceasta sala de lecturaca pe un loc misterios, ın care s-ar fi petrecut lucruri deosebite. Nu am mai ajunssa intru si eu cu drepturi depline ın aceasta sala de lectura, pentru ca s-a desfiintat.Erau unii studenti care stateau mult ın aceste doua sali de lectura, ba chiar ısi fixaseraniste locuri ale lor. Se studia cu o anumita ındarjire, mi se parea ca multi doreausa-si depaseasca conditia de copii din familii nevoiase si cautau sa ajunga la situatiimai bune. Mai tarziu, dupa ce am terminat facultatea, s-a mai ınfiintat o sala delectura la parter, pe coridorul unde se afla acum administratorul facultatii, ba chiarsi pe coridorul paralel, din fata salii metodice. Apoi, biblioteca pentru studenti s-amutat ın corpul B ın locul bibliotecii facultatii de stiinte economice, care, la randulei, s-a mutat ın corpul D, nou construit, langa biserica ”Patruzeci de Sfinti”.

Am ajuns sa intru, de cateva ori, ın biblioteca Seminarului Matematic pentru aconsulta niste materiale necesare pentru lucrul la cercurile stiintifice studentesti. Inacest fel am cunoscut pe bibliotecara A. Jacota, care a activat parca o vesnicie laaceasta biblioteca. Imi amintesc ca pe coridoarele de la parter si de la etajul II eradestul de multa galagie, ın special ın pauze. Prin contrast, pe coridorul de la etajul I,unde era intrarea la Seminarul Matematic era o atmosfera mai linistita care, ıntr-unfel, prevestea atmosfera de lucru, de reculegere, ca ıntr-o biserica, ce o regaseam defiecare data ın interior. Intrarea era prin salita ce desparte sala de carti de cea decolectii de reviste. In sala de carti, aflata ın dreapta, se adunau si lucrau profesorii,poate si conferentiarii mai ın varsta, ın sala de colectii de reviste se adunau si lucrauconferentiarii si lectorii mai ın varsta, iar ın sala de modele, acolo unde acum se aflaamfiteatrul ”A. Myller” (ın care se ajungea, atat din sala de colectii reviste printr-otrecere mica ınzestrata cu o perdea, dupa care se coborau cateva trepte sau, direct,de pe coridorul de la Catedra de geometrie, pe unde se intra cu ajutorul unei chei),se adunau si lucrau asistentii si lectorii mai tineri. Fiecare avea locul lui, la mese cupatru locuri si atmosfera era realmente una de lucru.

Cam prin 1962, facultatea a primit noi spatii de ınvatamant, devenite libere prinmutarea facultatilor de biologie si de geografie ın corpul B, abia dat ın folosinta. Inacest fel, fiecare catedra (Algebra, Analiza matematica, Geometrie, Ecuatii si mate-matici speciale, Mecanica) a primit cate un sediu, la fel si profesorii si conferentiarii.Cursurile si seminariile se desfasurau acum si ın amfiteatrul I.3 si ın salile 1.4 si 1.5.

6

Page 11: Revista (format .pdf, 4.1 MB)

Profesorii Gh. Gheorghiev, A. Haimovici, I. Popa, M. Haimovici, I. Creanga aveaucabinetele lor. Existau cabinete pentru conferentiari la analiza, geometrie, algebra simecanica, sala de mecanica era folosita ca sediu de catedra, mai era un laborator deinformatica ın actualul sediu al Catedrei de geometrie. In acest fel aparuse o degajarea spatiului la Seminarul Matematic, unde vizitatorii puteau sa citeasca ın tihna launa din mesele existente.

Atmosfera de studiu si bogatia de carti existente ın biblioteci m-au incitat sacumpar si eu carti de la librarii sau din anticariate. Trebuie spus ca situatia generalaın Romania de atunci nu prea permitea ca tinerii sa dea bani pe carti. Era destulasaracie si erau multe lipsuri care au fost ındreptate ceva mai tarziu. Am ınceput samerg prin librarii si prin anticariate si am avut destule ocazii sa cumpar carti utile.Dintre titlurile ce mi le amintesc as putea enumera: doua din cele trei volume aletratatului de analiza matematica al lui Miron Nicolescu (primul volum, pe care ılgaseam cel mai interesant si util, ın care se trata convergenta seriilor, era de negasit),cele trei volume ale tratatului de analiza matematica al lui Fichtengholtz, tradus dinlimba rusa, cartea de mecanica teoretica a lui Placinteanu si altele. Stiu ca la BCUcartea lui Placinteanu ca si cea a lui Valcovici aveau regim special, nu se ımprumutauacasa. Mai tarziu, am ınceput sa cumpar si carti ın limba rusa. In timpul scolii,ıncepand cu clasa a IV-a, am avut lectii de rusa. Apoi, ın timpul facultatii, am urmatcursurile de limba rusa concentrata pe diversi termeni matematici, astfel ca puteamıntelege carti si articole scrise ın limba rusa. Nu am studiat deloc limba engleza siacest aspect a fost un handicap pentru mine pentru ca a trebuit s-o ınvat singur sisa ma descurc cum pot la diversele conferinte la care am participat si unde a trebuitsa si vorbesc. Imi permit sa evoc protocolul prin care se achizitionau cartile ın limbarusa. Se trecea, de regula saptamanal, pe la libraria cu carti rusesti, care se aflaatunci la etajul librariei Junimea din Piata Unirii. (Anterior, aceasta librarie fusesela Fundatie, ın cladirea librariei Maxim Gorki, situata pe locul unde se afla acumCasa de Cultura a Studentilor. Mai tarziu, am aflat ca acolo fusese Jockey Clubul,un loc unde se ıntalnea protipendada Iasilor. Pe langa loc de ıntalnire si cultivarea relatiilor sociale, acesta mai era si un loc unde se practicau jocuri de noroc. Amaflat ca aici au pierdut destui bani mai multe personalitati ale Iasului interbelic ; spreexemplu, Cezar Petrescu.) Apoi, sectia de carti ın limba rusa s-a mutat la Casa Cartii.Saptamanal, soseau niste blancuri (brosuri reclama) cu titluri de carti ce urmau saapara. Cautam la sectia de matematica si ne semnam ın dreptul cartilor ce doreamsa le cumparam. La unele titluri era aglomeratie mare de semnaturi, la altele niciuna;totusi soseau si din cele la care nu erau semnaturi. Peste cateva luni, cand soseaucartile, unele titluri erau rezervate celor semnati pe blancuri si se puteau cumparacu usurinta. Atunci cand soseau mai putine carti, nu se mai facea aceasta rezervare.Stiu ca, o buna perioada de timp, sistemul a functionat. Apoi, au ınceput sa aparasincope. Imi amintesc ca nu am mai ajuns sa cumpar cartea de geometrie diferentialaa lui S. Sternberg si a fost mare supararea mea. Oricum, retin perioada ca un momentfoarte bun din viata mea, eram multumit si de ce cumparam si de rezultatele ce leobtineam atunci.

Dupa ce am terminat facultatea, ın 1964, am capatat drept de acces liber ın bi-blioteca Seminarului Matematic. Aceasta ınsemna ca puteam sa intru la orice ora

7

Page 12: Revista (format .pdf, 4.1 MB)

doream, sa consult si sa ımprumut cartile si revistele existente ın biblioteca. Candımprumutam, trebuia sa le notez ıntr-un registru anume facut, unde aveam o rubricaa mea. Cum spuneam si mai sus, erau multi tineri care studiau cu mult sarg, prezentiaproape la orice ora din zi si orice ora rezonabila din noapte. Spre exemplu, colegulmeu V. Barbu era prezent tot timpul la locul sau de munca ce se afla langa cabinetulprof. Gheorghiev. De asemenea, facea vizite foarte dese la biblioteca unde consultaın special revistele de pe panoul cu noutati. Am vazut ın registrul de ımprumuturica obisnuia sa ımprumute reviste sovietice pe care le ınsemna succint : YMH -pentruUspekhi Matematiceskih Nauk sau ∆AH - pentru Doklady Akademii Nauk. Eu, careeram ıncadrat la Academie, veneam des pe la prof. Gheorghiev, pe la D. Papuc siR. Miron, care devenisera conferentiari, si, bineınteles, pe la biblioteca Seminaru-lui Matematic, unde zaboveam destul de mult. Acolo ıl ıntalneam deseori pe prof.C. Corduneanu, care venea sa vada noutatile; din cand ın cand ma aborda si maıntreba cum merge treaba cu cercetarea stiintifica. A fost singurul care se interesa dece fac tinerii ce lucrau ın directii de cercetare diferite de a lui. Organizarea bibliotecii,prin aranjarea cartilor alfabetic dupa autori, ımi permitea sa descopar numeroasecarti de care nu stiam, atunci cand cautam o carte recomandata. Stateam mult sile rasfoiam, mai mult, pe unele le citeam si cultura mea matematica se ımbogateaconsiderabil.

Am putut sa apreciez si eficienta bibliotecarei A. Jacota care tinea biblioteca ınordine. Era tot timpul amabila, un pic cam severa ın legatura cu curatenia ın bi-blioteca si o adevarata enciclopedie ın materie de biblioteconomie. Mai tarziu, amaflat ca ın BCU era considerata o adevarata legenda, datorita acestor vaste cunostintedespre carti, dar si datorita faptului ca a refuzat cu ıncapatanare sa se mute ın sediulBCU (printr-o promovare) sau sa primeasca alti biliotecari ca ajutoare la SeminarulMatematic. Oricum, singura tinea ın ordine vasta colectie de reviste si de carti. Asacum spuneam, d-ra Jacota era foarte exigenta cu curatenia ın salile de lectura alebibliotecii. Atunci cand ploua, era foarte atenta cu vizitatorii, ın special cu cei tineri,care trebuiau sa se stearga foarte bine pe picioare la intrare, pentru ca sa nu ducaapa ın sali. Pe cei mai neglijenti ıi mustra cu asprime si, de regula, a doua oaranu se mai gresea. Trebuie sa mentionez ca, ın salile bibliotecii era un parchet foartefrumos si foarte bine ıntretinut. Cel putin o data pe an, se facea curatenie generala siparchetul era spalat cu petrosin, ceruit si lustruit. Era o adevarata placere sa privestiacel parchet dupa o astfel de curatenie generala. Acum, acel parchet a fost scos siın locul lui s-a pus ceva care seamana cu linoleumul. Oricum, deja s-a valurit si nuarata deloc bine pe la colturi. Tot ın legatura cu cadrul placut din salile de lectura,trebuie sa mentionez existenta unor ciubere cu plante exotice, care erau foarte bineıngrijite. Erau niste femei de serviciu, foarte devotate salilor de lectura de la Semi-narul Matematic, care aveau grija sa le ude, sa le stearga frunzele si sa le faca saarate bine. Mentionez ca tot ele se ocupau si de expedierea numerelor din Anale ınoperatia de schimb cu alte reviste. Atunci cand erau tiparite diverse fascicule dinAnale, nu numai de la sectia matematica, acestea se adunau pe mese ın sala de carti,se ımpachetau ın hartie, se legau cu sfoara si se puneau adresele la care urmau safie expediate. Unele pachete erau mai consistente, altele contineau doar o fascicula,dupa cum era conventia de schimb a revistei noastre cu alte publicatii. La ınceput,

8

Page 13: Revista (format .pdf, 4.1 MB)

adresele erau scrise de mana pe pachetele ın discutie; mai tarziu au ınceput sa fiedactilografiate si lipite pe pachete. Sistemul functiona foarte bine si se ınregistraufoarte putine plangeri ın legatura cu expedierea lor. Cheltuielile erau acoperite, ınmarea lor majoritate, din cotizatiile membrilor Seminarului, ıncasate lunar.

Noi, care veneam de doua-trei ori pe saptamana la Seminarul Matematic (uniiveneau zilnic si studiau la mesele existente), ne opream cu placere la panoul de noutati,unde rasfoiam diversele fascicule de reviste proaspat primite. Era o adevarata placeresa descoperi un articol interesant sau sa gasesti vreo citare utila. Este adevarat ca,ın vremea aceea, nu eram asa de interesati de citari ; era placut sa te stii citat, darnu faceam un obiectiv din aceasta, asa ca nu prea era o vanatoare dupa citari. Nicidespre reviste cotate ISI nu era vorba deloc (de fapt, nu se inventase ınca sistemulrespectiv), stiam ca existau niste reviste bune, ın special cele americane, englezesti,frantuzesti si japoneze, dar, de cele mai multe ori, noi ne publicam lucrarile ın revisteromanesti: Analele de la Iasi, Revue Roumaine de Mathematiques a Academiei, de laBucuresti si altele cateva. In ultimul timp, nici nu prea puteam sa trimitem lucrariın strainatate, avand ın vedere ca pentru expediere exista un adevarat protocol cenecesita timp, nervi si bani si nu erai ıntotdeauna sigur ca plicul respectiv va pleca.

Scene hazlii se petreceau ın perioada reviziilor anuale la biblioteca. In operatia derevizuire, care se desfasura pe o perioada de 10 zile-2 saptamani, trebuiau sa participetoate persoanele tinere ce aveau acces la biblioteca. Acestea, grupate ın perechi,revizuiau doua sau mai multe coloane de publicatii din rafturile bibliotecii. In acelmoment era o harmalaie deosebita ın biblioteca din cauza comunicarii ıntre partenerulaflat pe scara, care verifica existenta cartilor ın raft, si cel aflat la baza scarii, careverifica existenta cartilor ın carnetele cu fisele cartilor. Cu aceasta ocazie se semnalaudiverse lipsuri. Dupa ce se termina aceasta operatie, se trecea la recuperarea lipsurilor.Se mai cauta ınca o data, se mai cauta si ın alte locuri. Se cauta si ın registrele deımprumuturi, pentru a se vedea daca nu cumva vreun cititor nu a restituit publicatiile;daca se ıntampla acest lucru, era posibil ca cititorul respectiv sa-si piarda dreptulde a mai ımprumuta carti. Dupa cateva zile de vanzoleala, majoritatea lipsurilorse recuperau. Mai mult, uneori se recuperau si publicatii gasite lipsa la reviziileanterioare. Pana la urma, se ajungea la o situatie acceptabila, doar 3-5 publicatiilipsa, si se astepta cu sufletul la gura terminarea reviziei si acordarea dreptului dea ımprumuta publicatii. In momentul cand directorul bibliotecii acorda acest drept,se pornea o adevarata cursa (o asemanam uneori cu cursele din filmele western catreterenurile neocupate, ın cucerirea vestulului ın America) pentru gasirea publicatiilordorite a fi ımprumutate. Se mergea ın graba pana la raftul vizat, se cauta o scarasi apoi publicatia la care trebuia sa se ajunga ınaintea altor colegi interesati. Acestepublicatii erau recomandate de catre profesorii de la facultate si erau folosite pentrureferate la doctorat sau pentru pregatirea tezelor de doctorat sau ın documentareapentru o tema de cercetare stiintifica. Dupa ce gaseau publicatiile ce ıi interesau,cititorii asteptau sa le ınregistreze ca publicatii ımprumutate si plecau fericiti cu elela cabinete sau acasa. Erau si colegi care pierdeau cursa, ın sensul ca publicatiilecautate de ei fusesera luate deja de altii, si trebuiau sa se roage de acestia sa-i lase sale consulte.

Era destul de greu ın perioada aceea, avand ın vedere ca nu existau aparate de

9

Page 14: Revista (format .pdf, 4.1 MB)

copiere. Imi amintesc ca, ın anumite cazuri, am copiat pur si simplu de mana catevaarticole ce ma interesau. Pentru altele, faceam fotocopii, adica le fotografiam pursi simplu, developam filmul si faceam copiile pe hartie fotografica de marimea unuicarnet (jumatate de caiet). Erau vremuri grele, dar dispuneam de energia si inventivi-tatea necesare pentru a ne descurca. La un moment dat au aparut aparate de copiat(xeroxuri), dar erau putine, se defectau usor si se cam aflau sub controlul securitatii,avand ın vedere posibilitatea de a se multiplica materiale considerate dusmanoasepentru regim. Copiile erau de proasta calitate, dar puteau fi utilizate.

In perioada ın care am fost bursier la Universitatea din Napoli, Italia, ın 1972, ambeneficiat de o biblioteca organizata la fel ca si cea din Iasi: cu publicatiile ordonatealfabetic si cu acces liber la raft pentru cititori. Director al bibliotecii era profesorulCarlo Miranda, care a facut o specializare la Universitatea din Gottingen, unde fusesesi A. Myller, si a organizat biblioteca dupa modelul de acolo. La un moment dat,m-a ıntrebat despre D. Mangeron cu care studiase ımpreuna. Biblioteca de la Napolidispunea de mai multe fonduri. Imi amintesc ca avea un aparat de fotocopiat la carese puteau face liber diverse copii si ca hartia era de un tip special, cu proprietati ceaminteau de hartia fotografica. Mai mult, erau comandate diverse carti ın mai multeexemplare. Spre exemplu, cartile din colectia Lecture Notes de la Springer se gaseauıntr-un loc dedicat acestei serii, dar si ın raft la autorul respectiv. Acelasi lucru sepetrecea cu cartile din colectiile Grundlehren sau Ergebnisse s.a. de la Springer sau cucartile de la Academic Press, Marcel Dekker etc. Nu mai vorbesc de faptul ca puteamsa comandam cartile pe care le doream, gasite prin brosurile de reclama. Acesteasoseau ın cateva luni. Am regasit aceeasi organizare eficienta si la bibliotecile de laUniversitatea din Freiburg sau de la Centrul de Cercetare de la Oberwolfach.

La biblioteca Seminarului Matematic, majoritatea cartilor ajungeau prin operatiade recenzare a lor ın revista Analele St. Univ. ”Al.I. Cuza” Iasi . Anumiti colegiurmareau atent aparitia diverselor carti si trimiteau prompt la editurile ce le publicaucereri pentru un exemplar care sa fie recenzat. Dupa recenzie, cartea ramanea ınfondul bibliotecii. Daca se ıntarzia, era posibil ca fondul de reclama al editurii sase epuizeze si atunci cartile nu mai ajungeau la noi. Imi amintesc ca ıntr-o anumitaperioada, de aceasta operatie se ocupa colegul nostru J. Weinstein care era foarteinformat si foarte prompt ın comandarea cartilor pentru recenzii. In anumiti ani seajungea pana la 300 de carti venite pentru recenzii. Dupa ce soseau, cartile eraurepartizate unor cititori competenti ın domeniile respective si acestia le recenzau. Afost o perioada extrem de buna pentru biblioteca noastra, cand editurile occidentaleerau foarte generoase cu fondurile de reclama. Mai tarziu, cand editurile nu maidispuneau de fonduri de reclama asa de mari, cartile nu mai ajungeau la noi ın numaratat de mare.

Alta modalitate de procurare a cartilor era achizitia prin intermediul BCU dinIasi. Se lansau diverse comenzi de carti, care nu erau primite la recenzii, si se asteptaca BCU sa dispuna de banii (valuta) pentru a le achizitiona. Aceasta operatie duradestul de mult, uneori si un an. Imi amintesc ca, ıntr-o anumita perioada, profesoriicu notorietate aveau dreptul sa achizitioneze cate o carte din occident, pe an. Eicedau acest drept bibliotecii si se obtineau astfel un numar de carti ın plus. Achizitiise mai faceau si ın cadrul Filialei Iasi a Academiei Romane. Stiu ca exista o a-

10

Page 15: Revista (format .pdf, 4.1 MB)

numita coordonare astfel ca achizitiile facute la BCU si la Filiala erau ıntotdeaunacomplementare.

Majoritatea revistelor erau primite prin schimb cu revista noastra, Analele St.Univ. ”Al.I. Cuza”, Matematica. Mai existau si abonamente la unele reviste cu carenu se putea face schimb. Acestea erau putine si nesigure - ın anii ın care se operaurestrictii la fondurile valutare anumite abonamente se ıntrerupeau. Astfel, multecolectii au fost descompletate. Pentru unele dintre ele s-au facut eforturi deosebitede completare prin fotocopiere. In cazul cand ıntreruperea era foarte mare, nu s-amai putut face nimic. Alte posibilitati de completare a fondului de publicatii auaparut datorita generozitatii unor colegi care, fiind membri ın colectivele de redactieale unor reviste internationale, au donat Seminarului Matematic numerele primite dela redactiile acestora; as mentiona aici pe V. Barbu si C. Corduneanu, dar mai existasi altii.

Acum este momentul sa-mi exprim o neliniste si, partial, nemultumire ın legaturacu orientarea activitatii Seminarului Matematic. In ultimul timp s-a renuntat laabonamentele clasice preferandu-se accesul on-line. Pe de o parte acest fapt constituieun avantaj: se obtine acces la mai multe reviste, accesam informatia foarte rapid, faraa mai cauta volumul ın biblioteca, informatiile noi ajung mai repede la cititori. Spreexemplu, prin accesul on-line la Springer Link se pot consulta circa 190 reviste dematematica (numarul total de reviste din diverse domenii ce pot fi accesate este multmai mare, circa 1600 reviste). Tot asa, prin accesul la Science Direct se pot consultacirca 1800 reviste publicate, ın principal de la editura Elsevier. Pe de alta parte, ınbiblioteca nu mai ramane nimic! In cazul ın care accesul on-line ınceteaza, pierdemtoate informatiile la care aveam anterior acces. Cred ca trebuie reflectat la acesteaspecte si trebuie gasita o modalitate de a pastra informatiile ın biblioteca.

Prof. dr. Vasile OPROIU

11

Page 16: Revista (format .pdf, 4.1 MB)

Rigla si compasulGabriel POPA1

Abstract. The two instruments accepted by the ancient Greeks for performing geometric con-structions, if separately used, are not equally powerful. The compasses alone can accomplish allthe constructions able to be performed by means of the rule and the compasses together (Mohr -Mascheroni), while the rule alone cannot do it (Hilbert). These results are presented in this Note,with some clearing up brought to the proof of reference [1].

Keywords: circle, cone, rule, compasses.

MSC 2000: 51M15.

1. In problemele de constructii geometrice este permisa, ın general, utilizarea adoua instrumente: rigla si compasul. Aceste instrumente sunt considerate ca fiindideale; ele traseaza dreptele si cercurile exact, grosimea liniei de creion si orice alteaproximari nefiind luate ın considerare.

Rigla este presupusa ca fiind infinita, fara gradatii pe ea. Ea poate fi folositapentru a trasa dreapta ce trece prin doua puncte date (ın sensul determinarii oricaruipunct al acesteia). Nu o putem utiliza pentru a masura distante ıntre puncte.

Date O,P doua puncte ın plan, compasul poate fi utilizat pentru a trasa cerculde centru O si care trece prin P (ın sensul determinarii oricarui punct al acestuia).Compasul este considerat ca fiind nerigid: odata ce l-am ridicat de pe hartie, el seınchide, altfel spus nu putem ”transporta” distanta cuprinsa ıntre varfurile sale.

In orice problema de constructii geometrice, se porneste de la o multime data S depuncte ale planului. Putem obtine puncte noi cu ajutorul riglei si compasului asa cumam vazut anterior, precum si prin urmatoarele trei operatii, numite fundamentale:

• determinarea punctului de intersectie a doua drepte;• determinarea punctelor de intersectie a unei drepte cu un cerc;• determinarea punctelor de intersectie a doua cercuri.

Definitie. Spunem ca o problema de constructie este rezolvabila cu rigla si com-pasul daca o putem reduce la o succesiune finita de operatii alese dintre cele treioperatii fundamentale.

Scopul acestui demers este prezentarea posibilitatilor de folosire a acestor douainstrumente. Rezultatele principale sunt date de teoremele 2, 4 si 5 de mai jos.

2. Ne propunem mai ıntai sa aratam ca putem ınlocui compasul nerigid cu uncompas rigid (care, ın plus fata de cel nerigid, poate ”transporta” lungimea unuisegment, deci nu se ınchide automat dupa utilizare). Este adevarata urmatoarea

Teorema. Toate constructiile care pot fi realizate cu rigla si compasul rigid pot firealizate cu rigla si compasul, ın sensul precizat la 1.

Demonstratie. Este suficient sa dam un procedeu de constructie a unui segmentcongruent cu un segment dat si avand un capat fixat, folosind doar rigla si compasulnerigid (altfel spus, sa aratam cum se poate transporta un segment). Pentru aceasta,

1Profesor, Colegiul National, Iasi

12

Page 17: Revista (format .pdf, 4.1 MB)

fie [AB] un segment dat si [MM ′ o semidreapta data; dorim sa gasim unicul punctN ∈ [MM ′ pentru care [MN ] ≡ [AB].

Cercurile de centre A si M si care trec prin M , respectiv prin A, se intersecteazaın doua puncte; fie X unul dintre ele. Avem ca AXM este echilateral. Trasamcercul de centru A care trece prin B; acesta intersecteaza semidreapta [AX ıntr-unpunct C. Deosebim doua situatii:

a) C este ıntre A si X. Fie cercul de centru X care trece prin C si fie P punctulde intersectie dintre acesta si segmentul [XM ]. Exista un asemenea punct, ıntrucatXP = XC = XA − AC < XA = XM . Desenam cercul de centru M si caretrece prin P ; acesta intersecteaza semidreapta [MM ′ ıntr-un punct N si avem caMN =MP =MX − PX = AX − CX = AC = AB, deci N este punctul cautat.

b) X este ıntre A si C. Constructia curge la fel, ınsa punctul P nu se va mai aflape segmentul [MX], ci pe semidreapta opusa lui [XM .

Observatie. In cele ce urmeaza, vom folosi exprimari de genul: ”fie cercul decentru O si raza AB”, unde atat A cat si B sunt diferite de O; aceste constructii suntpermise de teorema precedenta.

3. Dorim sa aratam ın continuare ca un compas rigid poate realiza singur toateconstructiile posibil a fi efectuate cu rigla si compasul. Calea urmata este, ın liniimari, cea prezentata ın [1], unele afirmatii directe de acolo fiind justificate mai riguros.Demonstratia clasica, folosind inversiunea, poate fi gasita, spre exemplu, ın [2], pp.26-29.

Incepem prin a indica algoritmi pentru trei constructii importante.

(i) Constructia simetricului unui punct dat fata de alt punct dat.Presupunem date doua puncte A si B sifie a = d(A,B). Desenam cercul (C1) decentru A si care trece prin B, apoi cercul(C2) de centru B si care trece prin A.Razele celor doua cercuri sunt ambele a,iar distanta centrelor este, de asemenea,a. Deoarece a < a+a, conform teoremeicelor doua cercuri, rezulta ca (C1) si (C2)au ın comun doua puncte P si Q, aflatede o parte si de alta a dreptei AB. In

13

Page 18: Revista (format .pdf, 4.1 MB)

plus, cum PAB si QAB sunt echilaterale, avem ca m(÷AP ) = m(÷AQ) = 60

(arcele sunt gandite ın cercul (C2)).Construim acum cercul (C3) de centru Q, care trece prin P . Cum raza lui (C3)

este PQ < 2AB, urmeaza ca (C3) si (C2) au ın comun doua puncte; fie A′ al doileadintre ele. Deoarece ın cercul (C2) coardele [PQ] si [QA′] sunt congruente, avem ca

si arcele ÷QP si øQA′ sunt egale. Atunci:

m(úAQA′) = m(÷AQ) +m(øQA′) = m(÷AQ) +m(÷PQ) = 60 + 120 = 180,

deci punctele A si A′ sunt diametral opuse ın cercul (C2), altfel spus A′ este simetricul

lui A fata de B pe care ıl cautam.(ii) Constructia mijlocului unui segment dat. Fie A,B doua puncte; aflam ca

mai sus simetricul A′ al lui A fata de B.Trasam cercurile (C1) si (C2), de centre A,respectiv A′ si care trec prin B, respectiv A.Daca a = AB, razele celor doua cercuri sunt asi 2a, iar distanta centrelor este 2a. Sunt ver-ificate ipotezele teoremei celor doua cercuri sifie atunci P,Q = (C1) ∩ (C2). Trasam acumcercurile (C3) si (C4), de centre P , respectiv Qsi care trec prin A. Deoarece distanta centreloreste PQ < 2AB, urmeaza ca (C3) si (C4) auın comun doua puncte; fie M al doilea dintreele. Vom arata ca M este mijlocul cautat alsegmentului [AB].

Se observa usor ca patrulaterul PAQM este romb, deci PQ⊥AM. Pe de alta parte,

A este mijlocul arcului÷PQ ın cercul (C2), deci PQ⊥AA′. De aici, punctele A,M,A′ siB sunt toate coliniare. Triunghiurile A′AP si PAM sunt isoscele: A′A = A′P = 2a caraze ın (C2), PA = PM = a ca raze ın (C3) si au un unghi, ∠PAM, comun. Urmeazaca ele sunt asemenea, raportul de asemanare fiind 2 : 1. Atunci PA = 2AM , deci

AM =1

2AP =

1

2a.

(iii) Constructia piciorului perpendicularei coborata dintr-un punct P pe o dreaptaAB. Fie A,B doua puncte ın plan, iar P un punct necol-iniar cu ele. Trasam cercurile (C1) si (C2), de centre A,respectiv B si care trec prin P . Fie Q al doilea punct deintersectie al acestor cercuri; este clar ca Q este simet-ricul lui P fata de dreapta AB. Atunci mijloculM al seg-mentului [PQ], care poate fi determinat ca ın constructiaprecedenta, este piciorul perpendicularei din P pe [AB].

4. Teorema (Mohr–Mascheroni). Orice con-structie geometrica realizabila cu rigla si compasul sepoate efectua folosind doar compasul rigid.

Demonstratie. Vom considera ca o dreapta este determinata prin doua puncteale sale; pentru a afla un alt punct al dreptei, trebuie sa indicam un procedeu deconstructie a lui folosind compasul. Pentru a demonstra teorema, trebuie sa aratam

14

Page 19: Revista (format .pdf, 4.1 MB)

cum pot fi realizate cele trei operatii fundamentale. Evident, putem limita discutiala primele doua operatii.

(i) Aflarea punctelor de intersectie dintre un cerc si o dreapta. Presupunem caaceste puncte exista si dorim sa le determinam ca intersectii de cercuri. In cazul ıncare, pe parcursul constructiei, vom avea cercuri fara puncte comune, ınseamna cadreapta considerata este exterioara cercului initial. Deosebim doua situatii:

a) Dreapta nu trece prin centrul cercului. Fie (C) un cerc dat de centru O,iar A,B doua puncte astfel ıncat O /∈ AB.Aflam simetricul O′ al punctului O fata dedreapta AB, ca ın constructia precedenta.Trasam apoi cercul (C ′), de centru O′ siavand aceeasi raza ca si cercul (C). CumAB este axa de simetrie a figurii obtinute,urmeaza ca AB ∩ (C) = (C) ∩ (C ′), deunde constructia punctelor de intersectie din-tre AB si (C).

b) Dreapta contine centrul cercului. Fie(C) un cerc dat de centru O si raza R, iarA un punct ın plan. Dorim sa determinampunctele comune pentru (C) si OA. Fie M ∈ (C) oarecare. Conform a), putemdetermina N – al doilea punct de intersectie a lui (C) cu AM . Cu varful compasului ınM , apoi ın N si pastrand aceeasi deschidere, determinam un punct O′ pe mediatoareasegmentului [MN ] si construim un cerc (C1) de centru O

′, care sa aiba raza mai maredecat R.

Fie [PQ] o coarda a lui (C1) de lungime 2R, posibil de determinatconform 3.(i). Aflam B – punct de intersectieal dreptei PQ cu cercul (C2) de centru O′ sicare trece prin A, folosind a). Ca la 3.(ii),fie O′′ mijlocul segmentului [PQ], iar (C3)cercul de centru O′′ care trece prin P . In-tersectam acest cerc cu cercul de centru Bsi raza AN ; fie S unul dintre punctele deintersectie. Determinam acum X,Y pe (C),prin intersectii de cercuri, astfel ıncat [NX] ≡[SP ], [NY ] ≡ [SQ]. Vom arata ca X,Y suntpunctele cautate.

Deoarece cercurile (C) si (C3) sunt congru-ente iar [NX] ≡ [SP ], [NY ] ≡ [SQ], urmeazaca NXY ≡ SPQ, de unde [XY ] ≡ [PQ].Insa [PQ] este diametru ın (C3), deci [XY ] va fidiametru ın (C), adica X,O, Y vor fi coliniare.Ramane sa demonstram ca A ∈ XY .

Punctele A si B sunt situate pe cercul (C2), concentric cu (C1) si atunci ele voravea aceeasi putere fata de (C1), adica AM ·AN = BP ·BQ. Daca T = BS∩ (C3),obtinem ca BP ·BQ = BT ·BS, de unde AM · AN = BT ·BS. Cum [AN ] ≡ [BS],

15

Page 20: Revista (format .pdf, 4.1 MB)

rezulta ca [AM ] ≡ [BT ], deci [MN ] ≡ [TS]. Insa [MN ] si [TS] sunt coarde ın cercuri

egale, deci øMN = öST si apoi øXM = ÷TP , adica ∠XNM ≡ ∠PST . Urmeaza caXNA ≡ PSB si de aici ∠AXN ≡ ∠BPS. Pe de alta parte, ∠NXY ≡ ∠SPQ,deci m(∠AXN) +m(∠NXY ) = m(∠BPS) +m(∠SPQ) = 180, i.e. A ∈ XY , adicaceea ce doream sa dovedim.

(ii) Aflarea punctului de intersectie a doua drepte. Fie AB si A′B′ doua drepte, ınsensul ca avem date perechile de puncte (A,B) si (A′, B′). Folosind 3.(iii), construimpiciorul L al perpendicularei din B′ pe AB, apoi piciorulN al perpendicularei din L peA′B′. Daca N = B′, atunci AB∩A′B′ = ∅. Daca nu putem determina N , atunci ABsi A′B′ sunt drepte perpendicu-lare, concurente ın L.

Presupunem determinate L =N si fie P punctul comun celordoua drepte. P este bine deter-minat de lungimea l a segmen-tului B′P , ıntrucat odata cunos-cuta aceasta, intersectam cerculde centru B′ si raza l cu dreptaA′B′. Aplicand teorema cateteiın LB′P , obtinem ca

(1) B′L2 = B′N ·B′P = B′N · l.

Determinam simetricul B′′ al lui B′ fata de L si construim un cerc avand centrulpe mediatoarea segmentului [B′B′′], de raza suficient de mare. Prin intersectii decercuri, fixam D pe acest cerc astfel ıncat [DL] ≡ [B′N ], apoi fie E punctul ın careDL taie cercul. Din puterea punctului L,

(2) B′L2 = B′L · LB′′ = LD · LE = B′N · LE.

Comparand (1) si (2), rezulta ca LE = l, ceea ce ıncheie demonstratia.

5. In final, vom arata ca rigla este un instrument mai putin puternic decat com-pasul, ın sensul ca rigla singura nu poate realiza toate constructiile geometrice posibila fi efectuate cu rigla si compasul, ın timp ce compasul singur poate realiza toateaceste constructii. Avem nevoie de urmatorul rezultat, a carui demonstratie poate figasita, de exemplu, ın [5], pp. 235-238:

Lema. Fie un con oblic de varf V , avand drept baza ın planul (P ) cercul (C).Fie [AB] diametrul bazei pentru care (V AB)⊥(P ), iar (P ′) un plan perpendicularpe (V AB), care ıl intersecteaza dupa dreapta (A′B′), cu A′ ∈ V A, B′ ∈ V B. Daca∠V A′B′ ≡ ∠V BA, atunci (P ′) intersecteaza conul dupa un cerc.

Putem atunci demonstra urmatoarea

Teorema (Hilbert). Nu orice constructie geometrica realizabila cu rigla si com-pasul poate fi efectuata folosind numai rigla.

Demonstratie. Dat un cerc ın plan, putem sa-i aflam centrul folosind riglasi compasul (trasam mediatoarele a doua laturi ale unui triunghi ınscris ın cerc si

16

Page 21: Revista (format .pdf, 4.1 MB)

consideram intersectia acestora); vom arata ca aceasta constructie nu poate fi realizatanumai cu rigla. Sa presupunem prin absurd ca exista un anumit mod de a gasi centrulunui cerc folosind numai rigla. O transformare geometrica prin care cercul dat estedus ıntr-un cerc, iar orice dreapta este transportata ıntr-o dreapta, ar face ca ın figuratransformata a constructiei presupuse, imaginile dreptelor care initial se intersectauın centrul cercului dat, sa se intersecteze ın centrul cercului nou obtinut. Vom arataınsa ca o anumita proiectie conica duce dreptele ın drepte, cercul dat ıntr-un cerc, ınsanu face sa se corespunda si centrele celor doua cercuri; obtinem astfel o contradictiecare va ıncheia demonstratia.

Fie (C) un cerc de centru O ın planul (P ), iar V un punct astfel ıncat V O sa nu fieperpendiculara pe (P ). Fie (P ′) un plan ca ın ipoteza lemei si consideram proiectiaconica a planului (P ) pe planul (P ′). Este suficient sa mai aratam ca proiectia luiO nu este mijlocul O′ al segmentului [A′B′]. Sa presupunem ca V A > V B; daca

V U este bisectoarea unghiului ÕAV B, rezulta ca AU > UB, deoarece bisectoareadetermina pe latura pe care cade segmente proportionale cu laturile unghiului dincare pleaca. Pe de alta parte, din V A > V B rezulta ca m(∠V BA) > m(∠V AB),deci m(∠V A′B′) > m(V B′A′), de unde V B′ > V A′. Cum V U ′ este bisectoare ınV A′B′, unde U ′ = V U ∩A′B′, deducem ca U ′B′ > U ′A′. In concluzie, puncteleO si O′, mijloacele segmentelor [AB] si respectiv [A′B′], sunt separate de dreapta V Usi deci ele nu pot coincide.

Notam, ın ıncheiere, ca daca pe foaia pe care se realizeaza constructia este desenatun cerc oarecare, ımpreuna cu centrul sau, atunci putem efectua numai cu rigla (sifolosindu-ne de cercul dat) toate constructiile realizabile cu rigla si compasul (teoremaPoncelet-Steiner, demonstrata, de exemplu, ın [3], pp. 98-99).

Bibliografie

1. N. Hungerbuhler - A Short Elementary Proof of the Mohr-Mascheroni Theorem,A.M.M. 101 (1994), pp.784-787.

2. H. Lebesgue - Lecons sur les constructiones geometriques, Gauthier-Villars, 1950.

3. G.E. Martin - Geometric constructions, Springer-Verlag, 1998.

4. E. Moise - Geometrie elementara dintr-un punct de vedere superior, E.D.P., 1980.

5. M.H. Rademacher, O. Toeplitz - Despre numere si figuri, Ed. Stiintifica, 1968.

Vizitati pagina web a revistei:

http://www.recreatiimatematice.ro

17

Page 22: Revista (format .pdf, 4.1 MB)

O inegalitate ponderata cu medii

Gheorghe CIORESCU, Adrian SANDOVICI 1

Abstract. A refinement of the inequality of the means, ma ≥ mg , is given by inequalities (2)and (5), with the condition p ≥ (n− 1)q.

Keywords: arithmetic mean, geometric mean, harmonic mean, Sturm′s method.

MSC 2000: 97D99.

Consideram n ∈ N, n ≥ 2, si numerele strict pozitive ai, 1 ≤ i ≤ n. Notam cuma,mg,mh mediile aritmetica, geometrica si respectiv armonica ale acestor numere.

Scopul acestei note este de a demonstra o inegalitate de tipul

(1) p ·ma + q ·mh ≥ (p+ q) ·mg,

cu p si q numere reale strict pozitive. Observam ca (1) poate fi privita ca o rafinarea inegalitatii ma ≥ mg.

Propozitia 1. Are loc inegalitatea

(2) (n− 1)ma +mh ≥ n ·mg,

cu egalitate daca si numai daca a1 = a2 = . . . = an.Demonstratie. Avem

(n− 1)ma +mh = n · ma + . . .+ma +mh

n≥ n

nÈmn−1a ·mh.

Ca urmare, este suficient sa aratam ca are loc inegalitatea

(3)nÈmn−1a ·mh ≥ mg sau mn−1

a ·mh ≥ mng .

Sa notam xi = ai/nXk=1

ak, 1 ≤ i ≤ n, si sa observam ca xi ∈ (0, 1), 1 ≤ i ≤ n, si

nXi=1

xi = 1. Dupa calcule elementare, inegalitatea (3) se rescrie sub forma

(4) Sn(x1, x2, . . . , xn) =X

1≤j1<...<jn−1≤n

n−1Yk=1

xjk

!≤ 1

nn−2.

Vom demonstra aceasta inegalitate folosind metoda lui Sturm. Presupunem ca x1si x2 sunt astfel ıncat x1 < x2. Consideram numerele x′1 = x1 + ε si x′2 = x2 − ε asa

ca x1 < x′1 < x′2 < x2 si x′k = xk, 3 ≤ k ≤ n. Este clar canXi=1

x′i = 1 si x′i ∈ (0, 1),

1Profesori, Colegiul National ”Petru Rares”, Piatra Neamt

18

Page 23: Revista (format .pdf, 4.1 MB)

1 ≤ i ≤ n. Vom avea

Sn(x′1, x

′2, . . . , x

′n) = x′1x

′2

X1≤j1<...<jn−3≤n

n−3Yk=1

x′jk

!+ (x′1 + x′2)

nYj=3

x′j =

= (x1x2 + ε(x2 − x1 − ε))

X1≤j1<...<jn−3≤n

n−3Yk=1

xjk

!+ (x1 + x2)

nYj=3

xj =

= Sn(x1, x2, . . . , xn) + ε(x2 − x1 − ε)

X1≤j1<...<jn−3≤n

n−3Yk=1

xjk

!>

> Sn(x1, x2, . . . , xn).

Rezulta ca suma Sn(x1, x2, . . . , xn) ısi atinge maximul atunci cand numerele xi sunt

egale, adica pentru x1 = x2 = . . . = xn =1

n. De aici, se obtine inegalitatea (4) si, ca

urmare, (2) este dovedita.In sfarsit, are loc egalitatea ın (2) daca si numai daca mediile ma,mg si mh sunt

egale, adica daca si numai daca numerele ai, 1 ≤ i ≤ n, sunt egale.

Observatie. In membrul ıntai al inegalitatii (2) nu putem lua mai multi termeniegali cu mh, adica nu este adevarata inegalitatea (n− k)ma + k ·mh ≥ n ·mg pentruk ≥ 2. Un exemplu ın acest sens este urmatorul: n = 3, a1 = 1, a2 = 3 si a3 = 9.

O forma mai generala a inegalitatii (2) este data de

Propozitia 2. Are loc inegalitatea

(5) p ·ma + q ·mh ≥ (p+ q)mg,

unde p, q sunt numere reale strict pozitive ce verifica conditia p ≥ (n− 1)q. In (5) areloc egalitate daca si numai daca numerele ai(1 ≤ i ≤ n) sunt egale.

Demonstratie. Tinand cont de (2), avem

p ·ma + q ·mh = [p− (n− 1)q]ma + q[(n− 1)ma +mh] ≥[p− (n− 1)q]mg + qn ·mg = (p+ q)mg,

de unde (5). Ultima afirmatie din enunt se stabileste cu usurinta.

Vom ıncheia cu cateva aplicatii directe ale rezultatului de mai sus.Aplicatia 1 (Problema 26013, G.M.-7-8/2008). Sa se arate ca pentru orice nu-

mere reale a, b, c > 0 are loc inegalitatea

a+ b+ c+3

1a + 1

b +1c

≥ 43√abc.

Solutie. Pentru n = 3, p = 3 si q = 1 luati ın Propozitia 2, obtinem rezultatulproblemei.

19

Page 24: Revista (format .pdf, 4.1 MB)

Aplicatia 2. Fie a, b, p, q ∈ R∗+ cu p ≥ 2q. Rezolvati ın multimea R∗

+ inecuatia

(6) p(x2 + x+ 1) +9qx2

x2 + x+ 1≤ 3(p+ q)x.

Solutie. Pentru n = 3, a = x2, b = x si c = 1, relatia (5) devine

(7) p(x2 + x+ 1) +9qx2

x2 + x+ 1≥ 3(p+ q)x.

Deci, ın relatiile (6) si (7) vom avea egalitate. Conform Propozitiei 2, numerele x2, xsi 1 sunt egale. In consecinta, x = 1 este unica solutie a inecuatiei (6).

Aplicatia 3. Rezolvati ın R∗+ ecuatia

(8)2009

2010(2009 + x) +

2010x

2009x+ 1= 2010 2010

√x.

Solutie. Luand ın (2) n = 2010, x1 = x2 = . . . = x2009 = 1 si x2010 = x, avem

2009

2010(2009 + x) +

2010x

2009x+ 1≥ 2010 2010

√x.

Cum (8) cere egalitate ın relatia precedenta, rezulta ca x1 = x2 = . . . = x2010; deci,x = 1.

Diofant din Alexandria (sec. III d.Hr.)

Despre viata lui Diofant nu se cunoaste aproape nimic; nici data si nici loculnasterii. Se considera ca a trait, cel mai probabil, ın jurul anului 250 d.Hr. Si-adesfasurat activitatea la Alexandria si a scris un tratat ın 13 volume, Aritmetica,care poate fi comparat ca importanta cu Elementele lui Euclid (tot ın 13 volume).Numai sase dintre aceste volume nu au fost pierdute si au devenit sursa de inspiratiepentru matematicienii Renasterii. Pe marginea cartii a II-a a lui Diofant, matema-ticianul francez Pierre Fermat a notat celebra sa teorema, Marea Teorema a luiFermat.

Durata vietii lui Diofant se poate afla rezolvand o problema a sa, care a fost, separe, gravata pe piatra lui funerara.

Dumnezeu i-a ıngaduit sa fie copil o sesime din viata sa si, adaugand la aceasta adouasprezecea parte, i-a acoperit obrazul cu puf gingas, i-a ımpartasit lumina sfanta acasniciei dupa a saptea parte a vietii, iar dupa cinci ani de casatorie i-a oferit un fiu.Dar vai! nefericit copilul nascut tarziu; dupa ce a atins o jumatate din ıntreaga viataa tatalui, copilul a fost rapit de soarta necrutatoare. Dupa ce si-a alinat suferinta,adancindu-se ın stiinta numerelor vreme de patru ani, si-a dat sufletul.

Intrebare. Cati ani a trait Diofant?

N.B. Raspunsul se gaseste la pagina 24.

20

Page 25: Revista (format .pdf, 4.1 MB)

Inegalitatea lui Jensen pentru functii J-convexeın raport cu medii cvasiaritmetice

Florin POPOVICI 1

Abstract. In this Note an elementary proof is given for Jensen′s inequality related to a (M,N)-J-convex function (Definition 3), in the case when M and N are quasi-arithmetic means (Definition2).

Keywords: J-convex function, quasi-arithmetic mean, (M,N)-J-convex function.

MSC 2000: 52A40.

Inlocuind ın definitia functiilor J-convexe, cele doua medii aritmetice cu douamedii oarecare M si N G. Aumann ([1], pag. 4), ın anul 1933, extinde notiunea defunctie J-convexa prin notiunea de functie J-convexa ın raport cu perechea ordonatade medii (M,N). Inegalitatea lui Jensen, adaptata pentru functiile J-convexe ınraport cu perechi ordonate de medii (M,N) are loc pentru o clasa larga de medii, careinclude mediile cvasiaritmetice. Demonstratia de mai jos adapteaza rationamentulprezentat de noi ın [3]; credem ca este noua. In particular, din inegalitatea lui Jensenastfel generalizata, se obtin diferite inegalitati clasice.

Definitia 1. Fie I ⊂ R un interval. Un sir de functii M = (Mn)n≥2 se numestemedie pe I daca pentru orice n ∈ N, n ≥ 2, functia Mn : In → I satisface conditia

(1) minxi|i = 1, n ≤Mn(x1, . . . , xn) ≤ maxxi|i = 1, n,∀xi ∈ (0,∞), i = 1, n;

numarul Mn(x1, . . . , xn) se numeste media numerelor x1, . . . , xn.

Definitia 2. Fie I, J ⊂ R doua intervale. Fie φ : I → J o functie bijectiva strictmonotona. Consideram sirul de functii M = (Mn)n≥2, Mn : In → I, ∀n ≥ 2, definitprin

(2) Mn(x1, . . . , xn) = φ−1

φ(x1) + . . .+ φ(xn)

n

,∀x1, . . . , xn ∈ I.

Evident, M este o medie pe I. Media M se numeste medie cvasiaritmetica.

Observatii. 1) In cazul particular ın care I = J si φ = 1I , (2) este media

aritmetica A = (An)n≥2, unde An(x1, . . . , xn) =x1 + . . .+ xn

n,∀x1, . . . , xn ∈ I.

In cazul particular ın care I = J = (0,∞) si φ(x) =1

x, ∀x ∈ (0,∞), (2) este media

armonica H = (Hn)n≥2, unde Hn(x1, . . . , xn) =n

1x1

+ . . .+ 1xn

,∀x1, . . . , xn ∈ (0,∞).

In cazul particular ın care I = (0,∞), J = R si φ(x) = lnx, ∀x ∈ (0,∞), (2) estemedia geometrica G = (Gn)n≥2, unde G(x1, . . . , xn) = n

√x1 · . . . · xn,∀x1, . . . , xn ∈

(0,∞).2) Daca M = (Mn)n≥2 este o medie cvasiaritmetica pe I, atunci media M este

strict crescatoare, adica pentru orice n ∈ N, n ≥ 2, functia Mn este strict crescatoareın raport cu fiecare din variabilele x1, . . . , xn.

1Profesor dr., Colegiul National de Informatica ”Gr. Moisil”, Brasov

21

Page 26: Revista (format .pdf, 4.1 MB)

Definitia 3. Fie I1, I2 ⊂ R doua intervale date. Fie M o medie pe I1 si fie N omedie pe I2. O functie f : I1 → I2 se numeste convexa ın raport cu perechea ordonatade medii (M,N) (pe scurt, f este (M,N)− J-convexa), daca

(3) f(M2(x, y)) ≤ N2(f(x), f(y)), ∀x, y ∈ I1.

Observatii. 1) In cazul particular ın care M este media aritmetica pe I1 si Neste media aritmetica pe I2, (3) devine

fx+ y

2

≤ f(x) + f(y)

2, ∀x, y ∈ I1;

deci functiile (A,A)− J-convexe sunt functiile J-convexe.

2) Daca M si N sunt medii cvasiaritmetice, atunci conditia (3) devine

(4) f

φ−1

φ(x) + φ(y)

2

≤ ψ−1

ψ(f(x)) + ψ(f(y))

2

, ∀x, y ∈ I1.

Teorema 1. Fie I1, I2, J1, J2 ⊂ R patru intervale date. Fie φ : I1 → J1 siψ : I2 → J2 doua bijectii strict crescatoare. Fie M = (Mn)n≥2 media cvasiaritmeticadeterminata de functia φ, si fie N = (Nn)n≥2 media cvasiaritmetica determinata defunctia ψ. Daca f : I1 → I2 este o functie (M,N)− J-convexa, atunci, pentru oricen ∈ N, n ≥ 2, si orice x1, . . . , xn are loc inegalitatea lui Jensen generalizata

(5) f(Mn(x1, . . . , xn)) ≤ Nn(f(x1), . . . , f(xn)).

Demonstratie. Stabilim (5) prin inductie. Pentru n = 2, (5) are loc conformipotezei. Fie n ∈ N, n ≥ 2, o valoare pentru care are loc (5). Fie a, b ∈ I1. Notamc =Mn+1(a, . . . , a| z

n

, b) si d =Mn+1(a, b, . . . , b| z n

). Avem

c = φ−1

nφ(a) + φ(b)

n+ 1

= φ−1

(n2 − 1)φ(a) + φ(a) + nφ(b)

n(n+ 1)

=

= φ−1

(n− 1)φ(a) + φ(φ−1(φ(a)+nφ(b)n+1 ))

n

!=Mn(a, . . . , a| z

n−1

, d),

deci c = Mn(a, . . . , a| z n−1

, d). In mod analog, obtinem d = Mn(c, b, . . . , b| z n−1

). Rezulta ca

avem c =Mn(a, . . . , a| z n−1

,Mn(c, b, . . . , b| z n−1

)).

Tinand cont de monotonia mediei N si de ipoteza inductiva, obtinem

f(c)=f(Mn(a, . . . , a| z n−1

,Mn(c, b, . . . , b| z n−1

))≤Nn(f(a), . . . , f(a)| z n−1

, Nn(f(c), f(b), . . . , f(b)| z n−1

))) =

ψ−1

(n− 1)ψ(f(a)) + ψ(f(c))+(n−1)ψ(f(b))

n

n

.

22

Page 27: Revista (format .pdf, 4.1 MB)

De aici, obtinem succesiv

nψ(f(c)) ≤ (n− 1)ψ(f(a)) +ψ(f(c)) + (n− 1)ψ(f(b))

n⇐⇒

⇐⇒ (n+ 1)ψ(f(c)) ≤ nψ(f(a)) + ψ(f(b)) ⇐⇒

f(c) ≤ ψ−1

nψ(f(a)) + ψ(f(b))

n+ 1

⇐⇒

f(Mn+1(a, . . . , a| z n

, b)) ≤ Nn+1(f(a), . . . , f(a)| z n

, f(b)).(6)

Pentru orice x1, . . . , xn+1 ∈ I1 avem

Mn+1(x1, . . . , xn+1) = φ−1

nφ(x1)+...+φ(xn)

n + φ(xn+1)

n+ 1

=

= φ−1

nφ(Mn(x1, . . . , xn)) + φ(xn+1)

n+ 1

,

deci

(7) Mn+1(x1, . . . , xn+1) =Mn+1(Mn(x1, . . . , xn), . . . ,Mn(x1, . . . , xn)| z n

, xn+1).

In mod analog, pentru orice y1, . . . , yn+1 ∈ I2 avem

(8) Nn+1(y1, . . . , yn+1) = Nn+1(Nn(y1, . . . , yn), . . . , Nn(y1, . . . , yn)| z n

, yn+1).

Tinand cont de (6), (7) si (8), de ipoteza inductiva si de monotonia mediei N ,rezulta ca pentru orice x1, . . . , xn+1 ∈ I1 avem

f(Mn+1(x1, . . . , xn+1))(7)= f(Mn+1(Mn(x1, . . . , xn), . . . ,Mn(x1, . . . , xn)| z

n

), xn+1)(6)

Nn+1(f(Mn(x1, . . . , xn)), . . . , f(Mn(x1, . . . , xn)), f(xn+1)) ≤

≤ Nn+1(Nn(f(x1), . . . , f(xn)), . . . , Nn(f(x1), . . . , f(xn)), f(xn+1))(8)=

= Nn+1(f(x1), . . . , f(xn+1)),

deci f(Mn+1(x1, . . . , xn+1)) ≤ Nn+1(f(x1), . . . , f(xn+1)). Conform principiului inductieimatematice rezulta ca (5) are loc pentru orice n ∈ N, n ≥ 2.

Observatii. 1) Evident, inegalitatea (5) generalizeaza inegalitatea lui Jensenpentru functii J-convexe.

2) Deoarece√xy ≤ 1

2 (x + y), ∀x, y ∈ (0,∞), rezulta ca functia f = 1(0,∞) este(G,A)− J-convexa; conform Teoremei 1, obtinem inegalitatea mediilor

n√x1 · . . . · xn ≤ x1 + . . .+ xn

n, ∀x1, . . . , xn ∈ (0,∞),∀n ≥ 2.

23

Page 28: Revista (format .pdf, 4.1 MB)

3) Consideram functia f : (0,∞) → (0,∞), definita prin f(x) = 1+x, ∀x ∈ (0,∞).Avem

f(√xy) = 1 +

√xy ≤

È(1 + x)(1 + y) =

Èf(x)f(y), ∀x, y ∈ (0,∞),

deci functia f este (G,G)− J-convexa; rezulta ca are loc inegalitatea lui Huygens

1 + n√x1 · . . . · xn ≤ n

È(1 + x1) · . . . · (1 + xn),∀x1, . . . , xn ∈ (0,∞),∀n ≥ 2.

4) In [2], inegalitatea lui Jensen generalizata este stabilita pentru functii (M,N)−J-convexe, corespunzator unor clase largi de medii, care includ mediile cvasiaritmetice.

5) In [3], prin aplicarea directa a metodei din demonstratia Teoremei 1, am prezen-tat demonstratii simple pentru inegalitatea mediilor si pentru inegalitatea lui Huy-gens.

6) Inegalitatea (5) poate fi stabilita si pe baza rationamentului lui Cauchy pentrudovedirea inegalitatii ıntre mediile aritmetica si geometrica. Propunem acest exercitiucititorului.

Bibliografie

1. C.P. Niculescu, L.E. Persson - Convex Functions and Their Applications, AContemporary Approach, CMS Books in Mathematics, vol. 23, Springer-Verlag, NewYork, 2006.

2. C.P. Niculescu, F. Popovici - Inegalitatea lui Jensen pentru functii (M,N) − J-convexe ın conditii generale, va apare.

3. F. Popovici - Asupra inegalitatii Jensen, Recreatii Matematice, 1/2009, 12-14.

Raspuns la ıntrebarea de la pag. 20.

Notand cu x durata vietii lui Diofant, din problema rezulta urmatoarele:x

6–

perioada copilariei;x

12– adolescenta;

x

7– perioada de dinainte de casatorie; 5 ani

mai tarziu i s-a nascut fiul;x

2– durata vietii fiului; 4 ani au mai trecut pana la

moartea sa. Ca urmare, pentru aflarea necunoscutei x trebuie sa rezolvam ecuatia

x =x

6+

x

12+x

7+ 5 +

x

2+ 4.

Cum ecuatia se scrie3

28x = 9, rezulta ca Diofant a trait 84 de ani.

24

Page 29: Revista (format .pdf, 4.1 MB)

Inegalitatea H ≤ G ≤ A revizitataVasile CHIRIAC 1, Bogdan CHIRIAC 2

Abstract. In this Note, a new proof of the inequality H ≤ G ≤ A is given, together with acouple of applications of this inequality.

Keywords: arithmetic mean, geometric mean, harmonic mean.

MSC 2000: 97D99.

Fie ai > 0, i = 1, n numere reale; notam cu A,G,H mediile aritmetica, geometricasi respectiv armonica, adica

A =a1 + a2 + . . .+ an

n, G = n

√a1 · a2 · . . . · an, H =

n1

a1+

1

a2+ . . .+

1

an

.

Matematicianul englez Colin Maclaurin (1698-1746), ın lucrarea sa Algebraaparuta postum, ın 1748, a aratat relatia H ≤ G ≤ A ([2], p.30; [1] p.43, 44).Se cunosc multe demonstratii pentru aceasta inegalitate. Ne propunem sa dam onoua demonstratie.

Lema. Pentru orice x, y > 0 reale si n ≥ 2 natural, are loc inegalitatea:

(1) xn + yn ≥ xn−1 · y + yn−1 · x

Demonstratie. Cum x− y si xn−1− yn−1 au acelasi semn, oricare ar fi x, y > 0,putem scrie (x− y) ·

xn−1 − yn−1

≥ 0, de unde rezulta (1).

Teorema. Oricare ar fi numerele reale xi > 0 cu i = 1, n si n ≥ 2, avem:

(2) xn1 + xn2 + . . .+ xnn ≥ n · x1x2 · . . . · xn

Demonstratie. Procedam prin inductie matematica. Pentru n = 2 se obtinebinecunoscuta inegalitate x21+x

22 ≥ 2x1x2. Presupunem ca inegalitatea este adevarata

pentru n−1 numere si o vom demonstra pentru n. Putem scrie urmatoarele inegalitati:xn1 + xn2 ≥ xn−1

1 · x2 + xn−12 · x1 . . . xn1 + xnn ≥ xn−1

1 · xn+ xn−1n · x1

xn2 + xn3 ≥ xn−12 · x3 + xn−1

3 · x2 . . . xn2 + xnn ≥ xn−12 · xn+ xn−1

n · x2............................................................................................

xnn−1 + xnn ≥ xn−1n−1 · xn + xn−1

n · xn−1

Adunand membru cu membru si grupand convenabil gasim:(n− 1) (xn1 + xn2 + . . .+ xnn) ≥ x1(x

n−12 + xn−1

3 + . . .+ xn−1n )+

+x2xn−11 + xn−1

3 + . . .+ xn−1n

+ . . .+ xn

xn−11 + xn−1

2 + . . .+ xn−1n−1

.

Tinand seama de presupunerea facuta, obtinem (n− 1) (xn1 + xn2 + . . .+ xnn) ≥≥ x1 · (n− 1)x2x3 · . . . · xn+ x2 · (n− 1)x1x3 · . . . · xn+ . . .+ xn−1 · (n− 1)x1x2 · . . . ·xn−2 · xn + xn · (n− 1)x1x2 · . . . · xn−1 ,de unde se deduce inegalitatea de demonstrat pentru n numere.

Consecinta. Daca ai > 0, i = 1, n si n ≥ 2, atunci are loc relatia H ≤ G ≤ A.

1Profesor, Liceul ”V. Alecsandri”, Bacau2Student, Facultatea de matematica, Univ. ”Al. I. Cuza”, Iasi

25

Page 30: Revista (format .pdf, 4.1 MB)

Demonstratie. In (2), luand x1 = n√a1, x2 = n

√a2, . . . , xn = n

√an obtinem

A ≥ G, iar pentru x1 =1

n√a1, x2 =

1n√a2, . . . , xn =

1n√an

obtinem G ≥ H.

Aplicatie. Fie ai > 0, i = 1, n si n ≥ 4. Sa se arate ca:

a1a2 + a2a3 + a3a1a31 + a32 + a33

+a2a3 + a3a4 + a4a2

a32 + a33 + a34+ . . .+

an−1an + ana1 + a1an−1

a3n−1 + a3n + a31+

+ana1 + a1a2 + a2an

a3n + a31 + a32≤ 1

a1+

1

a2+ . . .+

1

an.

Solutie. Din a3i + a3j + a3k ≥ 3aiajak, avem1

a3i + a3j + a3k≤ 1

3aiajak, oricare ar

fi i, j, k = 1, n, i = j = k = i. Deciaiaj + ajak + akai

a3i + a3j + a3k≤ 1

1

ak+

1

ai+

1

aj

si,

sumand, deducem inegalitatea dorita.

Lasam ın seama cititorului sa demonstreze inegalitatile urmatoare :1) Fie a > 0, b > 0 cu proprietatea ca a+ b = 1. Sa se arate ca

6

r32 +

1

a5+

6

r32 +

1

b5≥ 4.

2) Oricare ar fi numerele reale strict pozitive a1, a2, . . . , an, n ≥ 3, avem

a1 + a2a21 + a22

+a2 + a3a22 + a23

+ . . .+an−1 + ana2n−1 + a2n

+an + a1a2n + a21

≤ 1

a1+

1

a2+ . . .+

1

an.

3) Fie ai > 0, i = 1, n, astfel ıncat a1 + a2 + . . .+ an = 1. Atunci

nXi=1

ai +

1

ai

2

≥n2 + 1

2n

.

4) Sa se arate ca ın orice triunghi ABC au loc inegalitatile:

i) a · sin A2+ b · sin B

2+ c · sin C

2≥ 3p

tgA

2· tg B

2· tg C

2

23

;

ii) a · tg A2+ b · tg B

2+ c · tg C

2≥ 6p · tg A

2· tg B

2· tg C

2.

Bibliografie

1. V. Chiriac - Matematica. Fundamentele Algebrei, Editura Sigma, 2007.

2. N. Mihaileanu - Istoria Matematicii, vol. 2, Ed. St. si Enciclop., 1981.

3. - Gazeta Matematica, seriile A, B, 1969-2009.

26

Page 31: Revista (format .pdf, 4.1 MB)

O extensiune a sirului Fibonacci

Petru MINUT1, Cristina SIMIRAD2

Abstract. A sequence (vn)n∈N, defined by v0 = 0, v1 = 1 and vn+2 = avn+1 + vn, wherea ∈ N∗, is considered. Properties of this sequence are revealed , some of them being similar to thoseof Fibonacci′s sequence.

Keywords: Fibonacci′s sequence, matrix, characteristic equation, Binet′s formula.

MSC 2000: 11B39.

Sirul Fibonacci este sirul (Fn)n∈N determinat de recurenta:

(1) F0 = 0, F1 = 1, Fn+2 = Fn+1 + Fn, n ∈ N.

El poate fi definit si prin egalitatile matriceale:

(2)

1 11 0

n=

Fn+1 FnFn Fn−1

, n ∈ N (cu conventia F−1 = 1).

Intr-adevar, pentru n = 0 relatia (2) devine

1 11 0

0

=

1 00 1

, ceea ce este adevarat.

Apoi, daca (2) este adevarata pentru n, vom avea1 11 0

n+1

=

1 11 0

Fn+1 FnFn Fn−1

=

Fn+1 + Fn Fn + Fn−1

Fn+1 Fn

=

Fn+2 Fn+1

Fn+1 Fn

,

deci (2) este adevarata si pentru n+ 1.

Ne punem problema gasirii sirurilor (vn)n∈N definite cu ajutorul unei matrici A =a bc d

, a, b, c, d ∈ N, prin

(3)

a bc d

n=

vn+1 vnvn vn−1

, n ∈ N.

Deoarece pentru n = 0 avem

a bc d

0

=

1 00 1

, relatia (3) ne va da, ın acest

caz, v1 = 1, v0 = 0 si v−1 = 1, iar egalitatea

a bc d

1

=

v2 v1v1 v0

se scrie

a bc d

=

v2 11 0

si conduce la v2 = a, b = c = 1 si d = 0. Prin urmare, A este de forma

A =

a 11 0

si (3) se scrie

(4)

a 11 0

n=

vn+1 vnvn vn−1

,∀n ∈ N.

1Prof. univ., Univ. ”Al.I. Cuza”, Iasi2Profesoara, Scoala nr. 10 ”Gh. Bratianu”, Iasi

27

Page 32: Revista (format .pdf, 4.1 MB)

Din faptul cavn+2 vn+1

vn+1 vn

= An+1 =

a 11 0

vn+1 vnvn vn−1

=

avn+1 + vn avn + vn−1

vn+1 vn

rezulta imediat ca sirul (vn)n∈N, ce are v0 = 0 si v1 = 1, satisface relatia

(5) vn+2 = avn+1 + vn, n ∈ N.

Numim (vn)n, dat de (5) si v0 = 0, v1 = 1, sir generalizat al lui Fibonacci. Vom vedeamai jos ca multe proprietati ale sirului (Fn)n∈N al lui Fibonacci raman valabile si ınacest caz si cu aceleasi demonstratii ([2], [3]).

Ecuatia caracteristica atasata sirului (vn)n∈N este

x2 − ax− 1 = 0,

cu radacinile x1 =1

2(a+

√a2 + 4) si x2 =

1

2(a−

√a2 + 4).

Sa ınlaturam cazul a = 0, care este banal. Observam ca a2 + 4 nu poate fi patratperfect: pentru a = 1 avem a2 +4 = 5, iar pentru a ≥ 2 avem a2 < a2 +4 < (a+1)2.Adoptam notatia ϕ = x1, deci x2 = ϕ (conujugatul lui ϕ). Se stie ([1], [3]) ca termenulgeneral al sirului (5) este de forma

vn = Aϕn +Bϕn, n ∈ N.

Pentru n = 0 si n = 1 obtinem sistemul de ecuatii: A+B = v0 = 0 si Aϕ+Bϕ = v1 = 1

din care deducem A = −B =1√

a2 + 4. Inlocuind aceste constante, deducem formula

de tip Binet

(6) vn =1√

a2 + 4(ϕn − ϕ

n), n ∈ N.

Mai ıntai , vom enumera cateva proprietati elementare ale sirului (vn)n∈N:

1nXk=1

vk =1

a(vn+1 + vn − 1).

Intr-adevar, tinand seama de (5), avem avk = vk+1−vk−1, k = 1, n. Sumand membrucu membru aceste egalitati, vom obtine pe cea dorita.

2nXk=1

v2k−1 =1

av2n.

Avem: av2k−1 = v2k − v2k−2, k = 1, n. Sumam membru cu membru.

3nXk=1

v2k =1

a(v2n+1 − 1).

La fel, pornind de la av2k = v2k+1 − v2k−1, k = 1, n.

42nXk=1

(−1)k−1vk =1

a(v2n − v2n+1 + 1).

28

Page 33: Revista (format .pdf, 4.1 MB)

Intr-adevar,2nXk=1

(−1)k−1vk =nXk=1

v2k−1 −nXk=1

vk =1

av2n − 1

a(v2n+1 − 1) =

1

a(v2n −

v2n+1 + 1).

5nXk=1

v2k =1

avn · vn+1.

Observam ca vkvk+1 − vk−1vk = vk(vk+1 − vk−1) = av2k si sumam pentru k = 1, n.

6. vm+n = vm−1vn + vmvn+1; ın particular, v2n =1

a(v2n+1 − v2n−1).

Se poate arata prin inductie dupa n sau ın felul urmator: egalitatea Am ·An = Am+n,cu An dat de (4), devine:

vm+1 vmvm vm−1

vn+1 vnvn vn−1

=

vm+n+1 vm+n

vm+n vm+n−1

.

Se efectueaza produsul matricelor si se scrie apoi egalitatea elementelor situate pelinia a doua si coloana ıntai.

Se numeste functie generatoare a unui sir (un)n∈N functia F data de F (z) =∞Pn=0

unzn.

Teorema 1. Functia generatoare a sirului (vn)n∈N este

(7) F (x) =x

1− ax− x2.

Demonstratie. Avem:

F (x) = v0 + v1x+ v2x2 + . . .+ vn+2x

n+2 + . . .−axF (x) = −av0x− av1x

2 − . . .− avn+1xn+2 + . . .

−x2F (x) = −v0x2 − . . .− vnxn+2 + . . .

Sumand, se obtine (1−ax−x2)F (x) = v0+(v1−av0)x sau, deoarece v0 = 0 si v1 = 1,(1− ax− x2)F (x) = x, de unde rezulta (7).

Observatie. Conform teoremei precedente, sirul (vn)n∈N este sirul coeficientilorcatului ımpartirii polinomului x la 1− ax− x2.

Urmatoarele patru teoreme indica proprietati de divizibilitate ale sirului (5).Teorema 2. Daca d/n, atunci vd/vn.

Demonstratie. Fie n = dm. Vom avea:

vn =1√

a2 + 4(ϕn − ϕ

n) =

1√a2 + 4

(ϕdm − ϕdm

) =

=1√

a2 + 4(ϕd − ϕ

d)(ϕd(m−1) + ϕd(m−2)ϕ

d+ . . .+ ϕ

d(m−1)) = vdM,

undeM este un polinom simetric ın ϕ si ϕ, radacinile ecuatiei x2−ax−1 = 0. Conformteoremei fundamentale a polinoamelor simetrice, M va fi un polinom cu coeficientiıntregi ın coeficientii acestei ecuatii. Pin urmare, M este un ıntreg si vd|vn.

29

Page 34: Revista (format .pdf, 4.1 MB)

Teorema poate fi demonstrata si pe baza relatiei 6 si procedand prin inductiedupa d.

Teorema 3. Daca n este numar compus, n = 4, atunci vn este numar compus.

Demonstratie. v4 = a3+2a este prim pentru a = 1.Daca n = n1n2, 1 < n1 ≤ n2,cel putin n2 ≥ 2, deci vn2 > 1, vn2 < vn si vn2 |vn.

Teorema 5. (vn+1, vn) = 1,∀n ∈ N.Demonstratie. (vn+1, vn) = (avn + vn−1, vn) = (vn, vn−1) = . . . = (v1, v0) = 1.

Teorema 6. (vm, vn) = v(m,n), ∀m,n ∈ N.Demonstratie. Se face pe baza algoritmului lui Euclid si cu utilizarea proprietatii

6 si Teoremei 5. Pentru detalii se pot vedea [2], [3].

Incheiem cu o proprietate de aproximare:Teorema 7. vn este numarul ıntreg cel mai apropiat de termenul de rang n al

progresiei geometrice cu termenul de rangul zero egal cu1√

a2 + 4si ratia ϕ.

Demonstratie. Intr-adevar, avem:vn − ϕn√a2 + 4

= 1√a2 + 4

(ϕn − ϕn)− ϕn√

a2 + 4

==

|ϕ|n√a2 + 4

=(√a2 + 4− a)n

2n√a2 + 4

<2n

2n√a2 + 4

<1

2.

Bibliografie

1. A. Markusevici – Siruri recurente, Editura Tehnica, Bucuresti, 1954.

2. P. Minut, C. Simirad – Numere prime. Numere prime speciale, Editura MatrixRom, Bucuresti, 2005.

3. N.N. Vorobiev – Numerele lui Fibonacci, Editura Tehnica, Bucuresti, 1953.

Vizitati pagina web a revistei:

http://www.recreatiimatematice.ro

30

Page 35: Revista (format .pdf, 4.1 MB)

Generalizarea unei identitati si aplicatiiLucian TUTESCU 1

Abstract. In this Note, the identity (1) is generalized as (2). Two applications are also given.Keywords: identity, polynomial, divisibility, composed number.

MSC 2000: 13M10.

Identitatea pe care o vom generaliza si utiliza ın aplicatii este

(1) a3 + b3 + c3 − 3abc = (a+ b+ c)(a2 + b2 + c2 − ab− bc− ca),∀a, b, c ∈ R.

Pentru a o demonstra consideram

P (x) = (x− a)(x− b)(x− c) = x3 − (a+ b+ c)x2 + (ab+ bc+ ca)x− abc;

avemP (a) = a3 − (a+ b+ c)a2 + (ab+ bc+ ca)a− abc = 0,P (b) = b3 − (a+ b+ c)b2 + (ab+ bc+ ca)b− abc = 0,P (c) = c3 − (a+ b+ c)c2 + (ab+ bc+ ca)c− abc = 0.

Adunand relatiile de mai sus membru cu membru, obtinem

a3 + b3 + c3 − (a+ b+ c)(a2 + b2 + c2) + (a+ b+ c)(ab+ bc+ ca)− 3abc = 0,

de unde deducem identitatea (1).Vom folosi aceasta identitate binecunocuta ın rezolvarea catorva probleme.

Propozitie. Aratati ca are loc urmatoarea identitate:

(2) x3 + y3 + z3 + t3 − 3xyz − 3xyt− 3xzt− 3yzt =

= (x+ y + z + t)(x2 + y2 + z2 + t2 − xy − xz − xt− yz − yt− zt),∀x, y, z, t ∈ R.

Intr-adevar, avem egalitatile:

x3 + y3 + z3 − 3xyz = (x+ y + z + t− t)(x2 + y2 + z2 − xy − yz − zx),x3 + y3 + t3 − 3xyt = (x+ y + z + t− z)(x2 + y2 + z2 − xy − yt− tx),x3 + z3 + t3 − 3xzt = (x+ y + z + t− y)(x2 + z2 + t2 − xz − zt− tx),y3 + z3 + t3 − 3yzt = (x+ y + z + t− x)(y2 + z2 + t2 − yz − zt− ty),

care, adunate membru cu membru, conduc la:

3(x3 + y3 + z3 + t3)− 3xyz − 3xyt− 3xzt− 3yzt == (x+ y + z + t)(3x2 + 3y2 + 3z2 + 3t2)−−(x+ y + z + t)(2xy + 2xz + 2xt+ 2yz + 2yt+ 2zt)−−t(x2 + y2 + z2 + t2 − t2)− z(x2 + y2 + z2 + t2 − z2)−−y(x2 + y2 + z2 + t2 − y2)− x(x2 + y3 + z2 + t2 − x2)++3(xyz + xyt+ xzt+ yzt),

1Profesor, Colegiul National ”Fratii Buzesti”, Craiova

31

Page 36: Revista (format .pdf, 4.1 MB)

de unde, ın urma unor calcule simple, obtinem (2).

Observatie. Identitatea (1) se obtine din (2) luand t = 0. Daca ın (2) luamz = t = 0, vom obtine x3 + y3 = (x+ y)(x2 + y2 − xy).

Aplicatia 1. Fie x, y, z ∈ N∗ si nu toate egale. Aratati ca daca x+ y + z dividex3 + y3 + z3, atunci numarul x+ y + z este compus.

Presupunand ca x+y+z este numar prim si tinand seama de ipoteza si identitatea(1), rezulta ca x+y+z divide produsul 3xyz, de unde deducem ca x+y+z|3, x+y+z|x,x + y + z|y sau x + y + z|z. Cum, ınsa, x + y + z ≥ 1 + 1 + 2 = 4 si x + y + z > x,x+ y+ z > y, x+ y+ z > z, se ajunge la o contradictie. Asadar, x+ y+ z este numarcompus.

Aplicatia 2. Fie x, y, z ∈ Z astfel ıncat (x − y)2 + (y − z)2 + (z − x)2 = xyz.Aratati ca x3 + y3 + z3 se divide cu x+ y + z + 6.

Relatia din enuntul problemei se mai scrie x2 + y2 + z2 − xy − yz − zx =xyz

2.

Combinand aceasta cu identitatea (1), obtinem

x3 + y3 + z3 =xyz

2(x+ y + z + 6).

Datorita acestei relatii putem afirma ca nu toate numerele x, y, z sunt impare (ın cazcontrar, ar rezulta ca si x + y + z + 6 este impar, ceea ce-i imposibil). Daca macarunul dintre x, y si z este par, aceeasi relatie ne arata ca x+ y + z + 6|x3 + y3 + z3.

1. Cu numerele 1, 3, 4, 6, luate ın ordinea pe care o doriti si punand ıntre ele ınmod convenabil semnele celor patru operatii aritmetice si, eventual, paranteze, obtinetica rezultat fiecare dintre numerele: 21, 22, 23, 24 si 25.

2. In tabelul de mai jos, obtineti rezultatul 7 punand semne convenabile de operatiimatematice sau paranteze:

1 1 1 1 = 72 2 2 2 = 73 3 3 3 = 74 4 4 4 = 75 5 5 5 = 76 6 6 6 = 77 7 7 7 = 78 8 8 8 = 79 9 9 9 = 7

N.B. Puteti gasi raspunsurile la pag. 36.

32

Page 37: Revista (format .pdf, 4.1 MB)

Problema G128 – comentarii

Marian Tetiva1

Abstract. This Note offers a discussion on the conditions ensuring the validity of inequalityG128 that was proposed in Recreatii Matematice No. 2/2007 and ”solved” in No. 2/2008 of thesame journal.

Keywords: Cauchy-Schwarz inequality.

MSC 2000: 97B99.

Dl Dumitru Barac din Sibiu observa, ıntr-o scrisoare trimisa redactiei, ca ınsolutia Problemei G128 [1] s-a strecurat o greseala. Si are perfecta dreptate, sensulinegalitatii Cauchy-Schwarz folosite acolo ın forma

x2

2x2 + (t− 1)xy+

y2

2y2 + (t− 1)yz+

z2

2z2 + (t− 1)zx≤

≤ (x+ y + z)2

2(x2 + y2 + z2) + (t− 1)(xy + yz + zx)

trebuie sa fie (corect) sensul contrar - ceea ce anuleaza demonstratia publicata.Problema cerea sa se arate ca are loc inegalitatea

(1)a

a2 + t+

b

b2 + t+

c

c2 + t≤ 3

t+ 1

pentru orice t ∈ [1, 5] si orice numere pozitive a, b si c cu produsul abc = 1. Iata ceam reusit sa demonstram:

Inegalitatea (1) are loc pentru orice t ∈ [3 − 2√2, 3 + 2

√2] si orice a, b, c ∈

[(√2− 1)

√t, (

√2 + 1)

√t] pentru care abc = 1.

Demonstratia utilizeaza o intercalare (sa-i spunem cu medii) uzuala ın asemeneainegalitati; anume, aratam ca, ın ipotezele anuntate, au loc inegalitatile

a

a2 + t+

b

b2 + t+

c

c2 + t≤ 2

√ab

ab+ t+

c

c2 + t≤ 3

t+ 1

(care implica inegalitatea din G128). Prima inegalitate este succesiv echivalenta cu

(a+ b)(ab+ t)

(a2 + t)(b2 + t)≤ 2

√ab

ab+ t⇔ a+ b

2√ab

≤ (a2 + t)(b2 + t)

(ab+ t)2⇔

⇔ (√a−

√b)2

2√ab

≤ t(a− b)2

(ab+ t)2⇔ (ab+ t)2 ≤ 2

√ab(

√a+

√b)2t.

Ultima inegalitate se mai scrie a2b2 + t2 ≤ 2√ab(a+ b+

√ab)t si rezulta folosind tot

o intercalarea2b2 + t2 ≤ 6abt ≤ 2

√ab(a+ b+

√ab)t.

1Profesor, Colegiul National ”Gheorghe Rosca Codreanu”, Barlad

33

Page 38: Revista (format .pdf, 4.1 MB)

Aici avem a2b2 + t2 ≤ 6abt deoarece ab ∈ [(3− 2√2)t, (3 + 2

√2)t] (conform ipotezei),

iar a doua inegalitate rezulta din 3√ab ≤ a+ b+

√ab.

Pentru2√ab

ab+ t+

c

c2 + t≤ 3

t+ 1

sa notam x =√c (⇔ c = x2) si sa ınlocuim ab cu 1/c = 1/x2. Avem de demonstrat

ca

f(x) =2x

1 + tx2+

x2

x4 + t≤ 3

1 + t

pentru x2 = c ∈ [(√2− 1)

√t, (

√2 + 1)

√t].

Sa remarcam ca aceste inegalitati implica x ∈ [(3−2√2)t, (3+2

√2)t]. Intr-adevar,

avemx2 = c ≤ (

√2 + 1)

√t ≤ (

√2 + 1)4t2

six2 = c ≥ (

√2− 1)

√t ≥ (

√2− 1)4t2,

folosind si√2−1 ≤

√t ≤

√2+1. Dar atunci vom avea x2−6tx+ t2 ≤ 0 si vom putea

arata ca functia f are un maxim ın 1 pe intervalul [(3 − 2√2)t, (3 + 2

√2)t]. Pentru

asta calculam

f ′(x) = 2

1− tx2

(1 + tx2)2+

tx− x5

(x4 + t)2

,

ceea ce va conduce la urmatoarea expresie pentru (1 + tx2)2(x4 + t)2f ′(x)/2:

(1− tx2)(x8 + 2tx4 + t2) + (tx− x5)(1 + 2tx2 + t2x4) =

= (1− x3)(t+ x)[t(x6 + 1) + 4tx3 − x4 − t2x2]

(nu exista alta modalitate de a ajunge aici decat aceea ın care calculam; doar cal-culam!). Deoarece paranteza patrata este nenegativa:

t(x6 + 1) + 4tx3 − x4 − t2x2 ≥ 2tx3 + 4tx3 − x4 − t2x2 = x2(6tx− x2 − t2) ≥ 0

(conform celor observate putin mai sus) rezulta ca f creste de la (3 − 2√2)t la 1 si

descreste pe intervalul [1, (3 + 2√2)t], deci are un maxim ın 1, asa cum am anuntat.

Inegalitatea f(x) ≤ f(1) (pentru x cuprins ıntre (3 − 2√2)t si (3 + 2

√2)t) este cea

care ne trebuie pentru a ıncheia demonstratia.(Merita un studiu separat cazurile t = 3 − 2

√2 si t = 3 + 2

√2, cand 6t − 1 −

t2 = 0, deci paranteza patrata de mai sus se anuleaza si ea pentru x = 1; se vavedea ca inegalitatea ramıne valabila, 1 fiind acum una din extremitatile intervalului[(3− 2

√2t, (3 + 2

√2)t].)

Bibliografie

1. T. Zvonaru, B. Ionita - Solutia problemei G128, Recreatii Matematice, 2/2008, p.161.

34

Page 39: Revista (format .pdf, 4.1 MB)

O problema de numarare

Razvan CEUCA1

Abstract. Problem C.O:5077 of Gazeta Matematica, No. 12/2009 has required to count up thetrapezia whose vertices lie among the vertices of a regular polygon with 2010 sides. This problem isgeneralized to the case of a regular polygon with n sides.

Keywords: regular polygon, trapezium.

MSC 2000: 05A05, 97B99.

In Gazeta Matematica 12/2009, apare problema

C.O.: 5077. Se considera poligonul regulat cu 2010 laturi A1A2 . . . A2010. Catetrapeze AiAjAkAl au varfurile printre cele ale poligonului?

Gabriel Popa si Paul GeorgescuIn nota de fata, ne propunem sa rezolvam aceasta problema ın cazul general,

considerand A1A2 . . . An poligon regulat cu n laturi.Fie C cercul circumscris poligonului. Orice trapez AiAjAkAl, fiind ınscris ın cercul

C, este isoscel si deci admite o axa de simetrie. Deosebim situatiile:

I. n impar. In acest caz, axa de simetrie a unui trapez dintre cele cautateeste diametru ın C, care contine un varf al poligonului si este mediatoare a laturiicare se opune acestui varf. Fie n = 2m + 1; numaram ıntai trapezele care admitaxa de simetrie A1Mm+1, unde Mm+1 este mijlocul segmentului Am+1Am+2. DacaAiAjAkAl este un astfel de trapez, cu 2 ≤ i < j ≤ m+1, atunci perechea (i, j) poate

fi aleasa ın C2m =

m(m− 1)

2moduri, prin urmare exista

m(m− 1)

2asemenea trapeze.

Cum axa de simetrie poate fi aleasa ın n moduri, numarul trapezelor este, ın acest

caz, n · m(m− 1)

2=n(n− 1)(n− 3)

8.

II. n par. Atunci, un trapez dintre cele considerate admite drept axa de sime-trie: 1) fie un diametru ApAp+m, p ∈ 1, 2, . . . ,m, unde n = 2m; 2) fie o me-diatoare MpMp+m, comuna perechii de laturi paralele ApAp+1 si Ap+mAp+m+1, cup ∈ 1, 2, . . . ,m, unde A2m+1 = A1. Apar ınsa doua dificultati suplimentare: con-teaza daca numarul punctelor ramase de-o parte a axei de simetrie este par sau impar,iar o parte dintre perechile de puncte care se formeaza nu furnizeaza trapeze, ci drept-unghiuri. In aceste conditii, deosebim situatiile:

II1. n = 4q. Fie AiAjAkAl un trapez cu axa de simetrie A1A2q+1, unde 2 ≤ i <

j ≤ 2q. Perechea (i, j) poate fi aleasa ın C22q−1 =

(2q − 1)(2q − 2)

2= (2q − 1)(q − 1)

moduri. Dintre acestea furnizeaza dreptunghiuri q − 1 perechi, anume cele de forma(i, 2q + 2 − i), i ∈ 2, 3, . . . , q. Gasim astfel (2q − 1)(q − 1) − (q − 1) = 2(q − 1)2

trapeze cu axa de simetrie A1A2q+1. Cum axele de simetrie de tipul 1) sunt ın numarde 2q, avem 4q(q − 1)2 trapeze de acest tip.

1Elev, Colegiul National Iasi

35

Page 40: Revista (format .pdf, 4.1 MB)

Fie acum AiAjAkAl un trapez cu axa de simetrieM1M2q+1, unde 2 ≤ i < j ≤ 2q+1. Perechea (i, j) poate fi aleasa ın C2

2q = q(2q−1) moduri. Dintre acestea, furnizeazadreptunghiuri q perechi, anume cele de forma (i, 2q+3−i), unde i ∈ 2, 3, . . . , 2q+1.Gasim astfel q(2q − 1) − q = 2q(q − 1) trapeze cu axa de simetrie M1M2q+1. Cumaxele de simetrie de tipul 2) sunt ın numar de 2q, obtinem 4q2(q−1) trapeze de acesttip.

Numarul total al trapezelor este, ın acest caz, 4q(q− 1)2 +4q2(q− 1) = 4q(q− 1)·

·(2q − 1) =n(n− 2)(n− 4)

8.

II2. n = 4q + 2. Cu o analiza asemanatoare, gasim 2q(q − 1)(2q + 1) trapeze cuaxa de simetrie de tipul 1) si ınca 2q2(2q+1) trazepe cu axa de simetrie de tipul 2). In

total, vom avea 2q(q−1)(2q+1)+2q2(2q+1) = 2q(2q+1)(2q−1) =n(n− 2)(n− 4)

8trazepe, deci acelasi rezultat ca ın subcazul II1.

In concluzie, numarul trapezelor AiAjAkAl cu varfurile printre cele ale poligonului

esten(n− 1)(n− 3)

8, daca n impar, respectiv

n(n− 2)(n− 4)

8, daca n par.

1. Raspuns la ”recreatia” 1 de la pag. 32:

6 · 4 : 1− 3 = 216 · 4− (3− 1) = 224 · (6− 1) + 3 = 236 : (1− 3 : 4) = 243 · (1 + 6) + 4 = 25.

2. Raspuns la ”recreatia” 2 de la pag. 32:

1 + (1 + 1 + 1)! = 7(2 · 2)!!− 2 : 2 = 73 + 3 + 3 : 3 = 7

4 + 4 : 4 +√4 = 7

5 + (5 + 5) : 5 = 7

6 : 6 +√6 · 6 = 7

7 + (7− 7) · 7 = 7√8 · 8− 8 : 8 = 7

9 : 9 +√9 +

√9 = 7

(se convine ca n! = 1 · 2 · 3 · . . . · n si (2n)!! = 2 · 4 · 6 · . . . · 2n).

36

Page 41: Revista (format .pdf, 4.1 MB)

Un sous-ensemble particulier de matrices carrees

Adrien REISNER1

Abstract. Let a, b be two strictly positive integer numbers and let A denote the set of matricesof the form (1), with x, y, z, t ∈ Q. It is shown that A is a Q-vector space of dimension 4 and it isalso a subring of the ring M2(R,+,×). Theorem 4 and Corollary 5 state conditions for a nonzeroelement in A to be invertible.

Keywords: Q-vector space, subring, invertible element.

MSC 2000: 11C08, 15A03.

a et b etant deux entiers strictement positifs, on considere l′ensemble A des ma-trices carrees A(x, y, z, t) d′ordre 2 de la forme suivante:

(1) A(x, y, z, t) =

x+ y

√a z

√b+ t

√ab

z√b− t

√ab x− y

√a

ou x, y, z et t appartiennent au corps des rationnels Q. On se propose d′etablirquelques proprietes liees a la structure algebrique de cet ensemble A (voir [1], ch. XI,page 453 et [2]).

Theoreme 1. Muni de l ′addition des matrices et de la multiplication des matricespar un scalaire rationnel l ′ensemble A est un Q - espace vectoriel de dimension 4.

Demonstration. L′ensemble A est un sous - ensemble de M2(R). L′additiondes matrices est une loi de composition interne de A. La matrice nulle appartient aA. Enfin pour toute matrice A ∈ A, la matrice −A ∈ A. Par suite (A,+) est ungroupe abelien.

∀λ ∈ Q,∀A ∈ A :

λx+ λy

√a λz

√b+ λt

√ab

λz√b− λt

√ab λx− λy

√a

∈ A.

L′ensemble A pourvu de l′addition et de la multiplication par les elements de Q estdonc un espace vectoriel sur Q. Etant donne les quatre matrices suivantes:

I =

1 00 1

, J =

√a 00 −

√a

,K =

0

√b√

b 0

, L =

0

√ab

−√ab 0

on a pour toute matrice A ∈ A : A(x, y, z, t) = xI + yJ + zK + tL. Donc la familleI, J,K,L est generatrice. Cette famille est libre puisque

xI+yJ+zK+tL = 0 ⇒ x+y√a = 0, x−y

√a = 0, z

√b+t

√ab = 0, z

√b−t

√ab = 0

⇒ x = y = z = t = 0.Finalement, les quatre matrices I, J,K et L constituent une base du Q - espace

vectoriel A qui est donc de dimension 4.

1Centre de Calcul E.N.S.T., Paris; e-mail: [email protected]

37

Page 42: Revista (format .pdf, 4.1 MB)

Remarque. detA(x, y, z, t) = x2 − ay2 − bz2 + abt2.

Theoreme 2. (A,+,×) est un sous - anneau de l ′anneau (M2(R),+,×).

Demonstration. La table de multiplication des matrices I, J,K et L est lasuivante:

facteur de droite −→ I J K LI I J K LJ J aI L aKK K -L bI -bJL L -aK bJ -abI

A ⊂ M2(R) qui est un anneau unitaire. (A,+) est un sous - groupe de (M2(R),+).Enfin, la multiplication est une loi de composition interne pour A compte tenu dela table de multiplication precedente. On en deduit finalement que (A,+,×) est unsous-anneau unitaire de l′anneau (M2(R),+,×).

Les deux theoremes precedents montrent que l′ensemble A muni des operationshabituelles est une Q - algebre de dimension 4 - [1], VI.4 page 215.

Theoreme 3. Pour toute matrice A ∈ A l ′application fA de A dans A definiepar ∀X ∈ A, fA(X) = XA est lineaire. De plus, det fA = (detA)2.

Demonstration. Pour A ∈ A, fA est une application deA dansA puisqueX ∈ Aet A ∈ A ⇒ XA ∈ A. D′autre part: ∀X1 ∈ A, X2 ∈ A fA(X1+X2) = (X1+X2)A =X1A + X2A = fA(X1) + fA(X2); ∀X ∈ A, ∀λ ∈ Q, fA(λX) = (λX)A = λ(XA) =λfA(X), i.e. fA ∈ LQ(A). L′espace vectoriel A etant muni de la base (I, J,K,L) -voir theoreme 1 - soit A = x0I + y0J + z0K + t0L. Determinons en utilisant la tablede multiplication des matrices I, J,K et L la matrice de l′endomorphisme fA danscette base (I, J,K,L):

fA(I) = x0I + y0J + z0K + t0L = AfA(J) = ay0I + x0J + at0K + z0LfA(K) = bz0I − bt0J + x0K − y0LfA(L) = −abt0I + bz0J − ay0K + x0L,

d′ou la matrice de fA ∈ LQ(A) dans la base consideree:

Mat(fA, (I, J,K,L)) =

x0 ay0 bz0 −abt0y0 x0 −bt0 bz0z0 at0 x0 −ay0t0 z0 −y0 x0

et un calcul elementaire montre alors que det(Mat(fA, (I, J,K,L))=[detA(x0, y0, z0, t0)]

2

compte tenu de la remarque precedente.

Theoreme 4. Dans le sous-anneau A tout element non nul est inversible si etseulement si l ′egalite u2 = av2 + bw2 avec u, v et w entiers n′est possible que pouru = v = w = 0.

Demonstration. Supposons que

(2) u2 = av2 + bw2, (u, v, w) ∈ Z3 ⇒ u = v = w = 0.

38

Page 43: Revista (format .pdf, 4.1 MB)

L′hypothese (2) etant verifiee, montrons d′abord le lemme suivant:

Lemme 5. On a l ′implication:

u′2 = av′2

+ bw′2 , (u′, v′, w′) ∈ Q3 ⇒ u′ = v′ = w′ = 0.

Demonstration. α etant le P.P.C.M. des denominateurs de u′, v′ et w′ il vient:

u′2

= av′2

+ bw′2 ⇒ (u′α)2 = a(v′α)2 + b(w′α)2, u′α ∈ Z, v′α ∈ Z, w′α ∈ Z.

L′hypothese (2) montre alors que: u′α = v′α = w′α = 0, d′ou u′ = v′ = w′ = 0.

Demontrons alors que toute matrice deA−0 est inversible. Supposant la matriceA = xI + yJ + zK + tL ∈ A non inversible, montrons que A = 0. Il vient - voirremarque -: detA = 0 soit x2 − ay2 = b(z2 − at2), ce qui implique:

(z2 − at2)(x2 − ay2) = b(z2 − at2)2 ⇒ z2x2 + a2t2y2 − a(t2x2 + z2y2) = b(z2 − at2)2

⇒ z2x2 + 2axyzt+ a2t2y2 − a(t2x2 + 2xyzt+ z2y2) = b(z2 − at2)2

⇒ (zx+ aty)2 − a(tx+ zy)2 = b(z2 − at2)2

soit finalement:(zx+ aty)2 = a(tx+ zy)2 + b(z2 − at2)2,

ou(zx+ aty, tx+ zy, z2 − at2) ∈ Q3.

L′hypothese (2) etant verifiee il vient alors en particulier, compte tenu du lemmeprecedent: z2 − at2 = 0. En utilisant une nouvelle fois le lemme avec u′ = z, v′ = t etw′ = 0 on obtient: z = t = 0. Par suite x2−ay2 = 0 et, d′apres le meme raisonnement:x = y = 0, i.e. A = 0.

On suppose que tout element A ∈ A − 0 est inversible. Soit (u, v, w) ∈ Z3 telsque u2 = av2 + bw2 et considerons la matrice A = uI + vJ + wK ∈ A. Si A = 0, elleest inversible - hypothese -, donc son determinant n′est pas nul: u2 − av2 − bw2 = 0,ce qui est absurde. On en deduit que A = 0, d′ou u = v = w = 0. Le theoreme estainsi demontre.

Corollaire 6. Si b est un nombre premier et si pour tout n entier n2 - a n ′estpas divisible par b, alors tout element non nul du sous - anneau A est inversible dansA.

Demonstration. On se propose de demontrer l′implication suivante:

(3)b premier et ∀n, n2 − a non divisible par b ⇒

⇒ u2 = av2 + bw2 ou (u, v, w) ∈ Z3 ⇒ u = v = w = 0

ce qui demontrera le corollaire compte tenu du theoreme precedent.b etant premier et ∀n, n2- a n′etant pas divisible par b, soit (u, v, w) ∈ Z3 verifiant

u2 = av2 + bw2. On peut supposer u, v, w premiers entre eux dans leur ensemble, carsinon, δ etant leur P.G.C.D. on aurait: u = δu′, v = δv′, w = δw′ et u′

2

= av′2

+bw′2 .

39

Page 44: Revista (format .pdf, 4.1 MB)

Si b divise v alors b divise u2; comme b est premier, b divise alors u et b2 diviseu2 − av2. Donc b2 divise bw2; comme b est premier, b divise w : u, v, w ne seraientpas premiers entre eux dans leur ensemble, ce qui est impossible.

Si b ne divise pas v, on a:

u2 = av2 + bw2 ⇒ ∀λ ∈ Z, u2 + 2λbu+ λ2b2 − av2 = b(w2 + 2λu+ λ2b) ⇒

(u+ λb)2 − av2 = b(w2 + 2λu+ λ2b).

On peut toujours choisir l′entier λ de facon que v divise u + λb. En effet, b et vsont premiers entre eux, donc ∃(α, β) ∈ Z2 tels que bα + vβ = 1, ce qui entraınebαu + vβu = u; donc v divise u − b(αu). Il suffit de prendre λ = −αu pour queu+λb = nv, d′ou n2v2−av2 = (n2−a)v2 = b(w2+2λu+λ2b). On en deduit alors: bdivise (n2 − a)v2 et b ne divise pas v2 ⇒ b divise n2 − a ce qui est impossible parhypothese. L′implication (3) en resulte, ce qui acheve la demonstration du corollaire.

Exemples. Les couples (a, b) suivants verifient les hypotheses du corollaire:

(a, b) = (5k + 2, 5), (a, b) = (5k + 3, 5)(a, b) = (7k + 3, 7), (a, b) = (7k + 5, 7), (a, b) = (7k + 6, 7), ou k ∈ N.

En effet:− si b = 5, alors pour tout n ∈ N, n2 ≡ 0 ou 1 ou 4(mod5), donc a ≡ 2 ou

a ≡ 3(mod5).− si b = 7, alors pour tout n ∈ N, n2 ≡ 0 ou 1 ou 2 ou 4(mod7) et par suite a ≡ 3

ou a ≡ 5 ou a ≡ 6(mod7).

Remarque. Soit E le sous - ensemble de A forme par les matrices M(x, y, z, t),ou x, y, z et t appartiennent a Z. N etant un entier relatif non nul on dsigne par ENl′ensemble

EN = M(x, y, z, t) ∈ E/detM(x, y, z, t) = N

On demontre alors le theoreme suivant (la demonstration de ce theoreme depassele niveau de cet article):

Theoreme 7. a) E1 est un groupe multiplicatif.b) Il existe un sous - ensemble fini IN de l′ensemble EN verifiant: ∀M ∈ EN ,

∃(G,E) ∈ E1 × IN tels que: M = G × E - decomposition canonique de la matriceM ∈ EN .

References

1. J. M. Arnaudies, H. Fraysse – Cours de mathematiques, Tome 1, Algebre, Ed.Dunod, Paris, 1987.

2. J. Fresnel – Algebre des matrices, Ed. Hermann, Paris, 1997.

3. C. Nastasescu, C. Nita, C. Vraciu – Bazele algebrei, Vol. 1 , Ed. Academiei,Bucuresti, 1986.

40

Page 45: Revista (format .pdf, 4.1 MB)

O problema complexa

Marian TETIVA1

Abstract. This paper can be perceived as an author′s invitation to his own working laboratory.A problem, initially formulated for equation (1), is eventually solved for the more general equation(2). The stages of going along this way are presented and motivated and the employed proceduresare commented.

Keywords: equations, resolvent, root.

MSC 2000: 00A35, 97C20.

1. Pe cand eram elev de liceu, una din problemele care mi-au dat bataie decap, rezistand tentativelor mele disperate (daca exageram putin putem zice si asa) derezolvare a fost urmatoarea:

Problema 1. Fie α un numar real de modul mai mic ca 1. Sa se arate ca ecuatia

(1) z4 + αz3 + αz + 1 = 0

are toate radacinile de modul 1.

Enuntul se afla cu siguranta ın [1], ın capitolul ”Polinoame”, probabil ca problemapropusa la capatul paragrafului despre ecuatii reciproce (nu mai pot spune exact,deoarece nu mai am, sau nu mai gasesc, acest manual de pe care am ınvatat primelenotiuni de algebra avansata). Mai tarziu, peste cativa ani, am rezolvat problema, catanar profesor care nu se putea prezenta ın fata elevilor fara a cunoaste foarte bine(perfect ar trebui) cartea pe care o foloseste la clasa. Am facut ceea ce parea atuncisa decurga obligatoriu din formularea problemei si pozitionarea ei ın cadrul textului:am considerat ecuatia ca pe una reciproca; deci radacinile ei sunt radacinile ecuatiilor

z +1

z= y1 si z +

1

z= y2,

unde y1 si y2 sunt, la randul lor, solutiile rezolventei

y2 + αy − 2 = 0

(care se obtine notand z +1

z= y etc). Se vede apoi ca ecuatia care da pe z1 si z2,

adica

z +1

z= y1,

se mai scrie z2 − y1z + 1 = 0, deci z1z2 = 1 si, analog, z3z4 = 1 (unde z3 si z4sunt solutiile ecuatiei z2 − y2z + 1 = 0). Daca mai aratam ca aceste ecuatii au

1Profesor, Colegiul National ”Gheorghe Rosca Codreanu”, Barlad

41

Page 46: Revista (format .pdf, 4.1 MB)

coeficienti reali si radacini complexe nereale, rezulta ca z1 si z2 sunt conjugate, deci,avand produsul 1, au modulele egale cu 1; similar, z3 si z4 au aceeasi proprietate siProblema 1 este rezolvata.

Exercitiul 1. Completati detaliile acestei rezolvari!Nu va va fi greu sa demonstrati ca y1 si y2 sunt numere reale. Ca sa dovediti ca

∆1 = y21 − 4 < 0 si ∆2 = y22 − 4 < 0 aratati ca ∆1 + ∆2 < 0 si ∆1∆2 > 0. Folositirelatiile ıntre radacini si coeficienti!

Exercitiul 2. Deduceti urmatorul rezultat, o idee mai general decat Problema 1:Problema 1′. Daca α este un numar real oarecare, atunci ecuatia

z4 + αz3 + αz + 1 = 0

are ori doua radacini de modul 1, ori toate radacinile de modul 1.

2. Anii au trecut si am gasit, ın [2] la pagina 137, problema 147 (si solutia ei lapagina 290). Enuntul este urmatorul (usor modificat de noi, dar numai formal):

Problema 2. Fie α un numar real cu |α| ≤ 1 si n un numar natural, n ≥ 3.Ecuatia

zn + αzn−1 + αz + 1 = 0

are toate radacinile de modul 1.E vorba, evident, de cazul general al Problemei 1; metoda utilizata mai sus nu

prea da sperante de a obtine aceasta generalizare, dar, cum spuneam, exista solutiaın [2]. Tot o solutie algebrica, dar utilizand subtil (si elegant) proprietatile modululuisi conjugatului unui numar complex.

Examinand cu atentie aceasta solutie constatam ca ea se potriveste la fel de bineurmatorului enunt:

Problema 2′. Fie α un numar complex cu |α| ≤ 1 si n un numar natural, n ≥ 3.Ecuatia

(2) zn + αzn−1 + αz + 1 = 0

are toate radacinile de modul 1.

Solutie. Fie z o radacina a ecuatiei, pentru care avem zn−1(z + α) = −αz − 1,deci si

|z|2(n−1)|z + α|2 = |αz + 1|2.

Un calcul simplu (care utilizeaza |w|2 = ww si proprietatile conjugatului unui numarcomplex) ne arata ca putem sa ınlocuim |αz + 1|2 cu |z + α|2 − (|z|2 − 1)(1 − |α|2),astfel ca egalitatea de mai sus se mai scrie

(|z|2(n−1) − 1)|z + α|2 + (|z|2 − 1)(1− |α|2) = 0.

Evident, daca |α| < 1, de aici rezulta |z| = 1. Daca |α| = 1, ne ramane egalitatea

(|z|2(n−1) − 1)|z + α|2 = 0,

42

Page 47: Revista (format .pdf, 4.1 MB)

din care obtinem fie (iar) |z| = 1, fie z = −α, numar care are tot modulul 1. Ceea ceıncheie demonstratia.

3. Au mai trecut ani si m-am mai gandit ca exista si posibilitatea abordarii acesteiprobleme cu ajutorul formei trigonometrice a numerelor complexe. Dupa parerea mea,aceasta abordare conduce la solutia cea mai interesanta a Problemei 2′, adica a cazuluicelui mai general (din cate vedem ın aceasta nota) al problemei initiale. Vom vedeade ce.

Ideea este sa aratam ca functia F definita prin

F (z) = zn + αzn−1 + αz + 1

se anuleaza de n ori pe cercul unitate (adica pe multimea numerelor complexe demodul 1). Avantajul este ca lucram direct cu numere complexe de modul 1, care potfi exprimate ın forma cos t + i sin t, pentru un anumit t real (de fapt din intervalul[0, 2π)). Pentru prima parte a acestei a II-a solutii a Problemei 2′ e nevoie de nistecalcule trigonometrice, pe care va invit a le face.

Exercitiul 3. Aratati ca, daca notam α = a+ ib, atunci

F (cos t+ i sin t) = 2

cos

nt

2+ i sin

nt

2

·

·cos

nt

2+ a cos

(n− 2)t

2− b sin

(n− 2)t

2

.

Evident, numai a doua paranteza se poate anula si vom arata ca asta se ıntamplapentru n valori (distincte) ale lui t din intervalul [0, 2π]. Consideram deci functia,desigur continua, φ : [0, 2π] → R, data de

φ(t) = cosnt

2+ a cos

(n− 2)t

2− b sin

(n− 2)t

2, ∀t ∈ [0, 2π].

Avem1√

a2 + b2φ(t) =

=1√

a2 + b2cos

nt

2+

a√a2 + b2

cos(n− 2)t

2− b√

a2 + b2sin

(n− 2)t

2=

=1√

a2 + b2cos

nt

2+ cos

nt

2− t+ θ

unde θ este un numar real astfel ıncat

cos θ =a√

a2 + b2si sin θ =

b√a2 + b2

.

Observam ca avem, pentru t = (2kπ)/n (k ıntreg), nt/2 = kπ, deci

1√a2 + b2

φ

2kπ

n

= (−1)k

1√

a2 + b2+ cos

θ − 2kπ

n

.

43

Page 48: Revista (format .pdf, 4.1 MB)

In cazul |α| =√a2 + b2 < 1 avem 1/

√a2 + b2 > 1, deci a doua paranteza este strict

pozitiva, ceea ce ınseamna ca semnele valorilor φ(2kπ/n) pentru k ıntreg alterneaza.Daca alegem pentru k valorile 0, 1, . . . , n rezulta ca functia continua φ se anuleaza decel putin n ori ın intervalul [0, 2π]: cate o data ın fiecare interval

2kπ

n,2(k + 1)π

n

, k = 0, 1, . . . , n− 1.

Asta ınseamna ca F are cel putin n radacini distincte pe cercul unitate; dar, fiind ofunctie polinomiala de grad n, rezulta ca are exact n asemenea radacini, sau ca toateradacinile sale au modulele egale cu 1.

Daca |α| = 1, paranteza de mai sus devine

1 + cos

θ − 2kπ

n

si este, ın general, nenegativa. Deoarece numerele θ − 2kπ

npentru k = 0, 1, . . . , n se

gasesc ıntr-un interval de lungime 2π, pentru cel mult o valoare a lui k aceasta expresieeste 0. Daca asta se ıntampla, respectiva valoare a lui k furnizeaza o radacina a lui Fsi, din cauza ei, se pierd cel mult doua schimbari de semn ın sirul valorilor φ(2kπ/n),deci oricum raman n− 1 radacini de modul 1 pentru F de care putem fi siguri. Cumprodusul radacinilor lui F este 1 sau −1, cea de-a n-a radacina rezulta tot de modul1 si astfel se ıncheie si aceasta solutie.

Despre care oricine va spune, probabil, ca e mai complicata decıt cea traditionala(nici n-am scris toate calculele!). Totusi, eu o consider mai instructiva si (aici e parteainteresanta) mai productiva. Spun asta deoarece (cititorul atent probabil s-a ıntrebatdeja) apare ın mod natural o chestiune adiacenta: dar daca |α| > 1? Ce mai putemspune despre modulele radacinilor ecuatiei zn + αzn−1 + αz + 1 = 0 ın cazul ın caremodulul lui α este mai mare ca 1? Deocamdata ne oprim aici, lasandu-va cadouurmatoarea ıntrebare:

Exercitiul 4. Daca α este un numar complex, cu |α| > 1, se poate garanta existentaunor radacini de modul 1 ale ecuatiei

zn + αzn−1 + αz + 1 = 0?

Eu as zice ca da.

Bibliografie

1. C. Nita, C. Nastasescu, S. Popa – Algebra. Manual pentru clasa a X-a, EdituraDidactica si Pedagogica, Bucuresti, 1980.

2. L. Panaitopol, I. C. Draghicescu – Polinoame si ecuatii algebrice, Editura Alba-tros, Bucuresti, 1980.

44

Page 49: Revista (format .pdf, 4.1 MB)

Axe si centre radicaleale cercurilor adjuncte unui triunghi

Ion PATRASCU1

Abstract. The properties of the adjoint circles associated to a triangle, with respect to circlesradical axes and centres, are exhaustively presented in this Note.

Keywords: adjoint circle, Brocard point, radical axis, radical centre, symmedian.

MSC 2000: 51M04.

Proprietatile prezentate ın acest articol se refera la axele si centrele radicale alecercurilor adjuncte unui triunghi.

Fie ABC un triunghi oarecare. Notam cu CA cercul care trece prin varfurile C,Asi este tangent laturii AB ın varful A. Semnificatii analoage au notatiile AC, AB,BA, BC si CB. Aceste cercuri asociate unui triunghi se numesc cercuri adjuncte.Asadar, unui triunghi ıi corespund ın general sase cercuri adjuncte; daca triunghiuleste isoscel, vor fi cinci cercuri adjuncte, iar daca este echilateral, vor fi numai trei.

Teorema 1. i) Cercurile adjuncte CA,AB, BC au un punct comun Ω cu propri-

etatea: ÕΩAB ≡ ÕΩBC ≡ ÕΩCA.ii) Cercurile adjuncte AC,CB,BA au un punct comun Ω′ cu proprietatea: ÕΩ′AC ≡ÖΩ′BA ≡ÖΩ′CB.Demonstratie. i) Fie Ω al doilea punct de intersectie a cercurilor CA si AB

A

B

C

Ww

w

w

Fig. 1

(fig. 1). Avem: ÕΩCA ≡ ÕΩAB (ın cercul CA) si ÕΩAB ≡ÕΩBC (ın AB). Obtinem: ÕΩAB ≡ ÕΩBC ≡ ÕΩCA, adica

relatia ceruta, iar din ÕΩBC ≡ ÕΩCA rezulta ca Ω se gasestepe cercul ce trece prin B si C si este tangent laturii AC ınC, adica cercul BC. Afirmatia ii) se demonstreaza analog.

Punctul Ω se numeste primul punct al lui Brocard, iar Ω′

al doilea punct al lui Brocard.

Observatie. Punctul Ω este centrul radical al cercurilor adjuncte CA,AB,BC,iar Ω′ este centrul radical al cercurilor adjuncte AC,CB,CA. Intr-adevar, atat Ω catsi Ω′ au puteri egale (nule) fata de tripletele de cercuri adjuncte indicate.

Teorema 2. Punctele lui Brocard Ω si Ω′ sunt izogonale ın triunghiul ABC.

Demonstratie. Notam m(ÕΩAB) = ω. Aplicand teorema sinusurilor ın triun-ghiurile AΩB si AΩC obtinem:

sinω=

c

sin(BΩA)=

sin(B − ω)si

sinω=

b

sin(AΩC)=

sinω.

1Profesor, Colegiul National ”Fratii Buzesti”, Craiova

45

Page 50: Revista (format .pdf, 4.1 MB)

Deoarece m(ÕBΩA) = 180 −B si m(ÕAΩC) = 180 −A, rezulta ca

BΩ=b sinB

c sinA=

sin(B − ω)

sinω.

Dezvoltand sin(B − ω) si tinand cont cab

c=

sinB

sinCsi sin(A+ C) = sinB, se obtine:

(1) ctgω = ctgA+ ctgB + ctgC.

Daca vom nota m(ÕΩ′AC) = ω′ si vom rationa analog, vom obtine ctgω′ = ctgA+ctgB + ctgC. Aceasta si relatia (1) conduc la ω = ω′, ceea ce arata ca punctele Ω siΩ′ sunt izogonale.

Unghiul ω se numeste unghiul lui Brocard si apare ın multe ımprejurari ın geome-tria triunghiului (v. [1]).

Teorema 3. Cercurile adjuncte CA si BA se intersecteaza pe simediana din A.Demonstratie. Fie S al doilea punct de intersectie al cercurilor CA si BA

A

B C

S

Fig. 2

(fig. 2) si P = AS ∩ BC. Din faptul ca ÕSCA ≡ ÕSAB siÕSBA ≡ ÕSAC, deducem caSBA ∼ SAC, de unde SBSC

=

AB2

AC2. Pe de alta parte, din congruentele de mai sus obtinemÕBSP ≡ ÕCSP , iar cu teorema bisectoarei ın triunghiul BSC

vom aveaSB

SC=

PB

PC. Ca urmare,

PB

PC=

AB2

AC2, ceea ce

arata ca AP este simediana din A.

Teorema 4. Cercurile adjuncte AB si AC se intersecteaza pe mediana din A.Demonstratie. Fie Q al doilea punct de intersectie a cercurilor AB si AC si

M = AQ ∩ BC. Dreapta AQ fiind axa radicala a cercurilor AB si AC, avemMB2 =MQ ·MA =MC2, de unde rezulta ca MB =MC.

Observatii. a) Teorema 3 exprima faptul ca axa radicala a doua cercuri adjunctecare sunt tangente la doua laturi ce au un varf comun este simediana dusa din acelvarf, iar Teorema 4 spune ca axa radicala a doua cercuri adjuncte tangente la aceeasilatura este mediana corespunzatoare acestei laturi.

b) Drept axa radicala a doua cercuri adjuncte unui triunghi pot fi: cevienele Bro-card (AΩ, AΩ′ etc.), simedianele, medianele sau laturile triunghiului (latura BC, deexemplu, este axa radicala a cercurilor BC si CB).

Pentru noi precizari privind centrele radicale ale tripletelor de cercuri adjuncte,vom utiliza urmatoarea

Lema. Coardele comune a trei cercuri secante doua cate doua sunt concurente.Demonstratie. Fie C1, C2, C3 trei cercuri secante si a1, a2, a3 axele radicale ale

perechilor (C2, C3), (C3, C1) si respectiv (C1, C2). Notam P intersectia dintre a1 si a2si observam ca P va avea puteri egale fata de toate cercurile, deci P va fi si pe axaradicala a3 a cercurilor C1 si C2.

Teorema 5. Au loc afirmatiile:

46

Page 51: Revista (format .pdf, 4.1 MB)

i) Ceviana AΩ, simediana din B si mediana din C sunt concurente (ıntr-un punctnotat JA);

ii) Ceviana BΩ, simediana din C si mediana din A sunt concurente (ın JB);iii) Ceviana CΩ, simediana din A si mediana din B sunt concurente (ın JC).Demonstratie. Conform Teoremei 1, cercurile CA si AB se intersecteaza ın

A

B C

M

S

JA

W

Fig. 3

Ω. Din Teorema 3, al doilea punct comun cercurilorAB si CB, notat S (fig. 3), este situat pe simedianadin B. In sfarsit, aplicand Teorema 4, cercurile CAsi CB se intersecteaza a doua oara ıntr-un punct M ,care apartine medianei din C. Conform Lemei, ce-vienele AΩ, BS si CM sunt concurente, ceea ce erade demonstrat.

In acelasi mod se poate dovedi si

Teorema 6. Au loc afirmatiile:i) Ceviana AΩ′, simediana din C si mediana din B sunt concurente (ıntr-un punct

J ′A);ii) Ceviana BΩ′, simediana din A si mediana din C sunt concurente (ın J ′

B);iii) Ceviana CΩ′, simediana din B si mediana din A sunt concurente (ın J ′

C).

Observatii. 1. Teoremele 5 si 6 se pot demonstra si cu reciproca teoremei luiCeva. Intr-adevar, stim ca piciorul simedianei din A ımparte latura BC ın raportulc2

b2. Pe de alta parte, notand cu A1 si A′

1 picioarele cevienelor Brocard AΩ si AΩ′,avem

(2)BA1

A1C=c2

a2,

BA′1

A′1C

=a2

b2

(pentru prima egalitate:BA1

A1C=

AABA1

AAA1C=

c sinω

b sin(A− ω)= . . . (1) . . . =

c2

a2, iar

pentru a doua se procedeaza similar sau se aplica Teorema lui Steiner cevienelorizogonale AΩ si AΩ′).

2. In general, un triunghi are sase cercuri adjuncte, deci pot fi C36 triplete diferite

de cercuri adjuncte. Unele triplete au, ınsa, acelasi centru radical. Varfurile tri-unghiului sunt centre radicale, fiecare pentru patru triplete; de exemplu, C este centruradical pentru (BC,CB,AC), (BC,CB,CA), (AC,CA,BC) si (AC,CA,CB).

Ca urmare, drept centru radical a trei cercuri adjuncte unui triunghi pot fi: punctelelui Brocard Ω si Ω′, punctele JA, JB , JC , J

′A, J

′B , J

′C si varfurile A,B,C.

3. Daca triunghiul este isoscel, AB = AC, atunci BC coincide cu CB si simedianadin B si mediana din C se intersecteaza ın punctul Ω al lui Brocard [3].

Bibliografie

1. T. Lalescu – Geometria triunghiului, Editura Apollo, Craiova, 1993.

2. R.A. Johnson – Modern Geometry: An Elementary Treatise on the Geometry of theTriangle and the Circle, 1929.

3. I. Patrascu – O teorema relativa la punctul lui Brocard, G.M.-9/1984, 328-329.

47

Page 52: Revista (format .pdf, 4.1 MB)

Scoala Normala ”Vasile Lupu” din Iasi– o istorie zbuciumata –

A fost creata pentru a pregati serii de ınvatatori menite sa raspandeasca luminacartii ın satele si orasele tarii si sa contribuie la ridicarea lor social-economica. Lungaei istorie, de mai bine de 150 de ani, este presarata de mari ımpliniri, dar si demomente dramatice.

In Asezamantul pentru organizarea ınvataturilor publice din princi-patul Moldova din 1851, care proclama libertatea si gratuitatea ınvatamantului,se prevedea, ıntre altele, faptul ca statul avea obligatia de a ınfiinta si ıntretine scolipentru populatia de la orase si sate. Prin hrisovul domnitorului Grigore Alexan-dru Ghica din 15 decembrie 1855 lua fiinta Institutul Preparandal, prima scoaladin cele doua tari romanesti avand ca scop pregatirea ınvatatorilor ce trebuiau saraspandeasca lumina ın lumea satelor.

Acest institut, cu durata studiilor de un an, a functionat timp de 35 de ani ınıncaperile manastirii Trei-Ierarhi din Iasi, unde mai exista o scoala, Scoala PrimaraVasiliana, si unde s-a ınfiintat si internatul preparanzilor. Director a fost numitAnton Velini, care a condus ıntregul complex scolar de la Trei-Ierarhi pana ın anul1863. Cateva dintre meritele lui: a ımbunatatit programele scolii, adaugand si dis-cipline noi, a tiparit un Manual de metodica si pedagogie pentru profesorii scoalelorprimare utilizat de preparanzi si ınvatatori pentru pregatirea lor pedagogica, a pre-conizat realizarea scolii primare de aplicatie (”preparanzii sa fie si pedagogi ...”) s.a.

48

Page 53: Revista (format .pdf, 4.1 MB)

Al doilea director al Scolii Preparandale a fost Titu Maiorescu, ın perioada1864-1868. Renuntand la pozitii mai bine situate si remunerate, primeste directiaacestei scoli cu convingerea ca ”aici este un camp de activitate modesta, patienta,ın aparenta inferioara, ın realitate de importanta suverana pentru poporul nostru”.Indrumatorul culturii romanesti considera ca ”regenerarea poporului roman ıncepe dela cultivarea limbii romane” si ısi ındreapta atentia spre scoala primara.

Considera ca trebuie asigurata viitorului ınvatator o cultura generala si o pregatireprofesionala de calitate si o ınalta constiinta a misiunii sale de educator. Fostii saielevi Ion Creanga si Mihai Busuioc au ilustrat stralucit acest profil de ınvatator.

A publicat anuarul scolii pe anul 1863-1864, primul de acest fel pentru scoli nor-male. Termenul ”Normala” asociat de Titu Maiorescu acestui tip de scoala vinedin Apus. De altfel, la propunerea lui Institutul Preparandal este numit Institu-tul lui Vasile Lupu. Au fost utilizate si alte denumiri: Scoala Normala de laTrei-Sfetite etc., ın cele din urma s-a statornicit numele Scoala Normala ”VasileLupu”. Incepand cu anul scolar 1864-1865, s-a trecut la doi ani de studiu. A pusbazele scolii primare de aplicatie, institutie ın care elevii urmau sa deprinda artapredarii lectiilor.

La 1 februarie 1868 Titu Maiorescu este ınlocuit la directie, dar cadrul activitatiiscolii fixat de el a dainuit ınca timp de aproape 20 de ani. Pe parcursul celor douaministeriate ale sale a continuat sa vegheze si sa sustina Scoala Normala ”Vasile Lupu”cat si ıntregul ınvatamant primar si normal-primar.

Directoratul lui Constantin Meissner, 1886-1892, a ridicat prestigiul scolii prinimportante shimbari (organizare interna, programe, manuale, gradina si atelierelescolii etc.) si i-a dat perspectiva de dezvoltare pentru o perioada lunga de timp. In1891 scoala se muta ın noul local de la via ”Pester” din Breazu-Copou, care este siactualul ei local. C. Meissner este ctitorul scolii primare de aplicatie, un local anexaal scolii ın care viitorii ınvatatori deprindeau arta predarii.

Cerinte sporite de la etapa la etapa au facut ca numarul anilor de studii sa creasca.Scoala Normala a trecut treptat la trei ani de studii (ın 1874), patru ani (1877), cinciani (1893), sase ani (1903), ajungand la sapte ani ın 1930.

Ioan Mitru, cel mai longeviv director normalist iesean (1896-1919), a fost unexcelent organizator - ”gospodarul desavarsit”, cum l-a caracterizat St. Barsanescu. Inferma scolii, cu sprijinul agronomului N. Cojocaru, elevii dobandeau stiinta lucrariloragricole. Viitorii ınvatatori, prin ceea ce vor realiza pe lotul scolii sau cel propriu,urmau sa fie nu numai educatori ai copiilor, ci si ai satului. Aleile si parcul scoliiau fost amenajate prin proiectul inginerului peisagist al orasului si profesor al scolii,Gh. Apostolescu, ıncepand cu anul 1900.

Faima scolii ieseane trecuse dincolo de granitele tarii. Profesorii erau dintre ceimai alesi din Iasi. Elevii purtau uniforme si, o parte, activau ın formatii artisticesau sportive: cor, orchestra, fanfara, formatia de dansuri, echipa de oina etc., re-numite pentru nivelul artistic sau competivitatea lor. Biblioteca scolii, initiata deTitu Maiorescu, si-a sporit fondul prin stradaniile si donatiile profesorilor P. Cujba,C. Meissner, I.V. Praja, I. Mitru si, mai apoi, St. Barsanescu, V. Petrovanu s.a.

Urgia razboiului s-a abatut si asupra Scolii Normale din Iasi: ın anul scolar 1916-1917 cursurile au fost suspendate, iar ın cel urmator doar clasa a IV-a a functionat

49

Page 54: Revista (format .pdf, 4.1 MB)

ın localul scolii, celelalte fiind repartizate ın diferite case din Iasi sau alte localitati.Intre cele doua razboaie mondiale, la conducerea Scolii Normale vin cativa pe-

dagogi eminenti, ce aduc ınnoiri ın pas cu ultimele curente pedagogice aparute ınEuropa: Vasile Todicescu - adept al pedagogiei experimentale (a creat un labo-rator de psihologie experimentala), Stefan Barsanescu – promotor al unei politiciscolare bazata pe cultura si care sa fie ın ritm cu transformarile sociale, GheorgheComicescu – adept al integralismului pedagogic. Vasile Petrovanu, distins pro-fesor de istorie si director, 1938-1941, a facut istoricul Scolii Normale de la originisi pana ın 1892 si a contribuit la aparitia anuarelor scolii din aceasta perioada. Inseptembrie 1939, se deschid, numai la aceasta scoala, doua sectii cu profil practic:agricola si industriala. Catre sfarsitul perioadei interbelice Scoala Normala atinge celmai ınalt nivel de organizare, realizari si conceptie. Ministrul instructiunii de atunci,Petre Andrei, o considera cea mai buna scoala de acest tip din Peninsula Balcanica.

Al Doilea Razboi Mondial si instaurarea regimului comunist au cauzat Scolii Nor-male un sir lung de greutati, schimbari de profil si programe, contopiri cu alte unitatiscolare similare, schimbari de nume, peregrinari prin diverse localuri etc.

Cladirea scolii s-a aflat ın zona frontului si a fost avariata: o bomba a distruslaboratorul de fizica si chimie, aviatia inamica a distrus acoperisul, armata germanaa transformat salile de clasa ın grajd de cai. Localul scolii de aplicatie a ars pana latemelie. Cel mai grav lucru a fost pierderea bibliotecii cu fondul ei pretios de carti.

Sa notam si cateva date din odiseea scolii: martie 1944 – refugiul la Craiova siapoi com. Pocruia-Gorj; martie 1945 – revenirea la Iasi si reluarea cursurilor ın lo-calul sumar reparat; decembrie 1954 – mutarea abuziva ın cladirea Scolii Normalede ınvatatoare ”Mihail Sturdza”, str. Toma Cozma (actualul corp D al Universitatii);1955 – se unifica cu scoala gazda si si devine mixta sub numele de Liceul Pedagogic;acesta se muta, ın anul scolar 1958-1959, ın cladirea din dealul Copoului; septembrie1959 – transferarea la complexul scolar din Barlad; septembrie 1966 – revine ın Iasi(fara dotare), cu titulatura de Liceul pedagogic ”Vasile Lupu”, ın localul din str.M. Kogalniceanu, iar internatul ın alte doua locuri; 1970 – se muta ıntr-un local nou sicorespunzator din cartierul Tudor Vladimirescu; 1977 – liceul va avea profil nu numaide ınvatatori, ci si de educatoare, prin unificarea cu Scoala de conducatoare (ınfiintataın Iasi ın 1919); 1985 – Liceul pedagogic ”Vasile Lupu” revine la propriul lui local dinBreazu-Copou. Perioada de dupa anul 1990 a ınsemnat o noua perioada de incerti-tudini cauzate de multele schimbari de profil si structura. Scoala a supravietuit si atrecut peste multele momente dificile datorita devotamentului, abnegatiei si spirituluide ınalta responsabilitate ale corpului profesoral fata de generatiile tinere. Din galeriaprofesorilor care au slujit cu dragoste scoala, spicuim doar cateva nume: A. Hasgan,V. Mitrofan, V. Fetescu, S. Radoi, R. Macareanu si multi altii.

Intr-o societate care nu are o politica educationala clara, Scoala Normala ”VasileLupu” ıncearca sa pastreze traditia normalista si sa fie fidela sistemului de valori uni-versale – garantii sigure ale afirmarii sale.

Prof. Viorel PARASCHIVDirector al Scolii Normale, 2002-2006

50

Page 55: Revista (format .pdf, 4.1 MB)

Concursul ”Recreatii Matematice”

Editia a VII-a, Durau, 28 august 2009

Clasa a V-a1. a) Completati sirul urmator cu ınca trei termeni: 1, 3, 7, 15, 31, 63, ... .b) Puneti paranteze ın expresia 2 · 12 + 18 : 6 + 1 astfel ıncat rezultatul sa fie

numarul natural cel mai mic posibil.2. Catul ımpartirii a doua numere naturale este 3 iar restul este 4. Daca dublam

deımpartitul si pastram ımpartitorul, atunci restul obtinut este 3. Determinati nu-merele initiale.

3. Paginile unei carti sunt numerotate de la 1 la 336. Din aceasta carte se rup, laıntamplare, 111 foi. Sa se arate ca:

a) suma numerelor de pe foile ramase nu se ımparte exact la 10;b) produsul numerelor de pe foile ramase se ımparte exact la 3.

Clasa a VI-a1. a) Utilizand operatiile cunoscute (adunarea, scaderea, ınmultirea, ımpartirea

sau ridicarea la putere) determinati cel mai mare numar natural care se poate scrie,folosind o singura data cifrele 1, 2 si 3. Justificati. Petre Batranetu

b) Fie a, b ∈ N. Sa se arate ca, daca ultima cifra a numarului a2+b2 este 9, atunciultima cifra a lui (a+ b)2 este tot 9. Recreatii Matematice - 2/2007

2. Intr-un regat sunt 2009 orase. Regele a poruncit sa se realizeze drumuri ıntreorase astfel ıncat din fiecare oras sa porneasca exact 5 drumuri. Au putut supusii saındeplineasca ordinul regelui? Mihai Craciun

3. Fie multimea A = 1, 2, 3, ..., 98. Aratati ca oricum am alege 50 de elementeale lui A, exista doua printre ele avand suma cub perfect. Titu Zvonaru

Clasa a VII-a

1. a) Fie numarulA=aa...aa| z de n ori

bb...bb| z de n ori

. Aratati caA=10n − 1

3·a(10n − 1)

3+a+ b

3

.

b) Aratati ca numarul B = 44...4| z de n ori

22...2| z de n ori

poate fi scris ca un produs de doua

numere naturale consecutive. Mihai Craciun2. Demonstrati ca triunghiul determinat de picioarele bisectoarelor unui triunghi

cu un unghi de masura 1200 este dreptunghic.3. Fie m si n numere naturale nenule cu proprietatea ca m ≤ 1+2+ ...+n. Sa se

arate ca m poate fi scris ca suma catorva numere distincte dintre numerele 1, 2, ..., n,unde n ≥ 3. Recreatii Matematice - 2/2008

Clasa a VIII-a1. Fie x, y ∈ Z. Daca x2010+y2010 se divide cu 11, aratati ca x+y se divide cu 11.

Andrei Nedelcu

51

Page 56: Revista (format .pdf, 4.1 MB)

2. Fie triunghiul ABC ınscris ın cercul C(O,R). Se cere locul geometric alpunctelorM din planul triunghiului, pentru care are loc relatiaMA2+MB2+MC2 =3R2. Dan Branzei

3. Determinati x, y, z pentru care1

2x+

3

2y+

1

2z= 2x + 3 · 2y+2 + 2z+2 = 9.

Recreatii Matematice - 1/2008

Clasa a IX-a1. Rezolvati, ın necunoscuta (x; y) ∈ Q×Q, ecuatia x2009+ y2009 = x2010+ y2010.2. Intr-un careu cu 41 linii si 49 coloane se scriu la ıntamplare 2009 numere reale

distincte. Fie A multimea ce are ca elemente cele mai mici numere de pe fiecare linie,respectiv B multimea ce are ca elemente cele mai mari numere de pe fiecare coloana.Determinati probabilitatea ca cel mai mare element din multimea A sa fie chiar celmai mic element din multimea B.

3. Fie D un punct ın planul unui triunghi echilateral ABC ıncat BD = DC,m(^BDC) = 300 si dreapta BC separa A si D. Daca E ∈ (BD) si m(^BAE) = 150,sa se arate ca CE⊥AC. Recreatii Matematice - 2/2007

Clasa a X-a1. Rezolvati, ın necunoscuta (x; y) ∈ N× N, ecuatia x · (x+ 2) · (x+ 8) = 3y.2. Fie triunghiul ABC cu m(^ABC) = m(^ACB) = 800 si P ∈ (AB) astfel

ıncat m(^BPC) = 300. Aratati ca AP = BC.3. Determinati functiile f : N → N, pentru care are loc egalitatea 2 · f(n + 3) ·

f(n+ 2) = f(n+ 1) + f(n) + 1,∀n ∈ N. Recreatii Matematice - 2/2007

Clasa a XI-a1. Fie triunghiul ABC nedreptunghic. Paralela prin B la AC si simetrica

dreptei AC ın raport cu BC se intersecteaza ın A1; analog se obtin punctele B1 siC1. Daca dreptele AA1, BB1, CC1 sunt concurente, sa se arate ca triunghiul ABCeste echilateral. Recreatii Matematice - 1/2006

2. Fie functia f : A→ A, unde A este o multime finita din R. Daca |f(x)−f(y)| <|x−y|,∀x, y ∈ A, x = y, sa se arate ca functia f nu este nici injectiva, nici surjectiva.

Lucian Georges Ladunca

3. Fie sirul (xn)n≥1 definit recurent prin x1 ∈ N, xn+1 =

3xn2

+ 1,∀n ∈ N∗.

Este posibil sa alegem x1 numar natural, astfel ıncat primii 2008 termeni ai sirului safie numere pare si al 2009-lea sa fie impar? Lucian Georges Ladunca

Clasa a XII-a1. Fie sirul (xn)n≥1 dat de x1∈

−π4, 0, xn+1=2xn−tg xn,∀n ≥ 1. Sa se studieze

existenta limitelor limn→∞

xn si limn→∞

n√−xn. Recreatii Matematice - 1/2006

2. Sa se determine functiile derivabile f : I → (0;+∞) si intervalul I ⊂ R, stiindca f(0) = 1 si f3(x) + f ′(x) = 0, ∀x ∈ I. Gabriel Mırsanu

3. Fie functia g : [0; 1] → R derivabila pe (0; 1) cu g(0) = 0, iar f : [0; 1] → R+

o functie cu proprietatea f(x) = g′(x), ∀x ∈ [0; 1]. Sa se arate ca exista cel putin un

punct c ∈ (0; 1) astfel ıncatπ

2· g(c) · cos

π2c< f(c). Gabriel Mırsanu

52

Page 57: Revista (format .pdf, 4.1 MB)

Concursul de matematica ”Gaudeamus”

Editia I, Iasi, 2009

In perioada 30 octombrie - 1 noiembrie 2009, la Facultatea de Matematica a Universitatii”Alexandru Ioan Cuza” Iasi, s-a desfasurat prima editie a Concursului de Matematica”Gaudeamus”.

Organizatorii manifestarii sunt: Facultatea de Matematica si Fundatia Seminarului Mate-matic ”Alexandru Myller”. Colaboratori: Inspectoratul Scolar Judetean Iasi si Liceul deInformatica ”Grigore Moisil” Iasi.

Manifestarea s-a desfasurat pe doua sectiuni: 1)proba scrisa individuala; 2) concursul deproiecte ”Matematica ın lumea reala”.

1) Proba scrisa individuala s-a adresat elevilor claselor a X-a, a XI-a, a XII-a si acontinut subiecte din programele scolare ale anilor anteriori; tematica a fost publicata pesite-ul Facultatii de Matematica, www.math.uaic.ro . Listele cu elevii participanti, precumsi rezultatele lor, pot fi consultate la aceeasi adresa internet.

2) Concursul de proiecte ”Matematica ın lumea reala” s-a adresat elevilor sauechipelor de elevi (maxim patru), fara limitare de varsta, si a constat ın prezentarea unorsolutii matematice la probleme concrete ale lumii reale. Au fost sustinute 12 proiecte.

Acest concurs doreste sa fie, ın primul rand, un mijloc de apropiere ıntre Facultatea deMatematica si elevii din ınvatamantul preuniversitar. Numarul de participanti (aproximativ150 de elevi din judetele Bacau, Botosani, Neamt, Vaslui si Iasi), precum si rezultateleobtinute, sunt ıncurajatoare.

Subiectele propuse la proba scrisa individuala

Clasa a X-a1. i) Sa se arate ca, daca 3 | a2 + b2, unde a, b ∈ N, atunci 3 | a si 3 | b.ii) Sa se arate ca nu exista (a, b, c, d) ∈ N4, (a, b, c, d) = (0, 0, 0, 0) astfel ıncat

a2 + b2 = 3(c2 + d2).

2. Se considera n puncte S = A1, A2, ..., An dintr-un plan π si n numere realeΛ = λ1, λ2, ..., λn astfel ıncat λ1+λ2+ · · ·+λn = 0. Spunem ca A′ ∈ π este centrulde masa al sistemului (S,Λ) daca exista un punct O ∈ π astfel ıncat

−−→OA′ =

λ1λ1 + · · ·+ λn

−−→OA1 + · · ·+ λn

λ1 + · · ·+ λn

−−→OAn.

i) Sa se arate ca daca A′ este centrul de masa al sistemului (S,Λ) atunci pentruorice punct M ∈ π are loc

−−→MA′ =

λ1λ1 + · · ·+ λn

−−−→MA1 + · · ·+ λn

λ1 + · · ·+ λn

−−−→MAn.

Pentru un triunghi A1A2A3, consideram λ1, λ2, λ3 ∈ R astfel ıncat |λ1|, |λ2| si |λ3|reprezinta lungimile laturilor [A2A3], [A3A1] si respectiv [A1A2].

ii) Sa se arate ca A′ ∈ A1A2 este piciorul bisectoarei interioare (respectiv ex-

terioare) a unghiului cA3 daca si numai daca A′ este centrul de masa al sistemului(A1, A2, λ1, λ2), pentru o anume alegere a semnelor scalarilor λ1 si λ2.

iii) Sa se arate ca I ∈ π este centrul cercului ınscris (respectiv centrul unui cercexınscris) triunghiului A1A2A3 daca si numai daca I este centrul de masa al sistemului(A1, A2, A3, λ1, λ2, λ3), pentru o anume alegere a semnelor scalarilor λ1, λ2 si λ3.

53

Page 58: Revista (format .pdf, 4.1 MB)

3. Fie n ∈ N\0, 1 si a1, a2, ..., an numere reale. Construim: s0 = −a1−a2−...−an;sm = a1+a2+ ...+am−(am+1+am+2+ ...+an), ∀m ∈ 1, n− 1; sn = a1+a2+ ...+an.

Sa se arate ca exista un m ∈ 0, n astfel ıncat |sm| ≤ max|ak| | k ∈ 1, n.Clasa a XI-a

1. Doua sute de elevi participa la un concurs de matematica, la care se propun6 probleme. Dupa corectare, se observa ca fiecare problema a fost rezolvata corectde cel putin 120 de elevi (nu neaparat aceiasi pentru fiecare problema). Aratati case pot gasi doi participanti, astfel ca fiecare problema sa fi fost rezolvata de cel putinunul din cei doi.

2. i) Fie A = a1, a2, ..., an ⊆ Z si f : A → A o functie bijectiva. Presupunandca a1 < a2 < ... < an si ca ai+f(ai) < aj+f(aj) pentru orice i < j, i, j ∈ 1, 2, ..., n,sa se arate ca f = 1A. ii) Sa se gaseasca o functie bijectiva f : Z →Z care satisfaceconditiile m+ f(m) < n+ f(n) pentru orice m < n, m,n ∈ Z, si sa fie diferita de 1Z.

3. Fie a ∈ (0,+∞). Intr-un plan π, relativ la un sistem de coordonate cartezienexOy, consideram dreapta de ecuatie (δa) : y = ax. Fie Π = [0, 1)× [0, 1).

a) Definim f : R × R → Π, f(x, y) = (x, y), unde prin am notat parteafractionara. Sa se arate ca f este periodica, adica exista T ∈ R astfel ıncat f(x +T, y + T ) = f(x, y) pentru orice (x, y) ∈ R× R.

b) Consideram restrictia fa, a lui f la dreapta (δa). Sa se arate ca daca a ∈ Qatunci fa este de asemenea periodica.

Imaginea lui (δa) prin fa este o reuniune de segmente paralele cu directia lui (δa).c) Pentru a ∈ Q sa se arate ca numarul acestor segmente este finit.Vom nota cu N acest numar.

d) Pentru a =10

2009sa se determine N . Cat este N pentru a =

10

2010?

e) Ce se poate spune despre imaginea lui (δa) prin aplicatia fa cand a este numarirational (de exemplu

√2)?

Clasa a XII-a1. Fie A,B ∈ M(n,C) astfel ıncat A+B = In, A

2 = A3. Sa se arate ca:i) AB = BA; ii) In −AB si In +AB sunt inversabile.

2. Se considera functia f : (−1,+∞) → R , f(x) =√1 + x.

i) Sa se scrie ecuatia tangentei y = ax+ b la graficul functiei f , ın punctul x0 = 0.

ii) Sa se determine valoarea a pentru care limx→0

f(x)− 1− x2

xaexista si este = 0.

iii) Aratati ca, pentru x ∈−1

2,1

2

are loc inegalitatea:

f(x)− 1− x

2

≤ x2

2√2.

iv) Fie N numarul natural cu 2010 cifre, toate egale cu 1, adica N = 1111 . . . 11| z 2010 cifre

.

Determinati primele 2010 zecimale ale numarului√N .

3. Se cer valorile lui a ∈ R pentru care sistemul¨2|x| + |x| = y + x2 + a

x2 + y2 = 1

are o unica solutie reala.

54

Page 59: Revista (format .pdf, 4.1 MB)

Solutiile problemelor propuse ın nr. 1/2009

Clasele primareP.164. Scrie vecinii vecinului comun al numerelor 16 si 18.

(Clasa I) Diana Tanasoaie, eleva, IasiSolutie. Vecinii numarului 16 sunt numerele 15 si 17, iar ai numarului 18 sunt

numerele 17 si 19. Vecinul comun este 17. Vecinii numarului 17 sunt 16 si 18.

P.165. Dupa ce dau celor doi frati mai mari cate doua banane, mananc si eu treibanane. In cos ımi ramane un numar de banane ce poate fi scris cu doua cifre diferitesi care este cel mai mic numar de acest fel. Cate banane am avut ın cos?(Clasa I ) Inst. Maria Racu, Iasi

Solutie. Numarul bananelor ramase ın cos este 10. Numarul initial de bananedin cos este 2 + 2 + 3 + 10 = 17.

P.166. Din cei 8 catelusi albi si negri, cel mult 3 sunt albi. Care este numarulmaxim de catelusi negri? Dar cel minim?(Clasa a II-a) Ioana Baragan, eleva, Iasi

Solutie. Deoarece avem catelusi albi si negri, cel putin un catelus este alb.Numarul maxim de catelusi negri este 7. Cum cel mult 3 catelusi sunt albi, celputin 5 vor fi negri.

P.167. Intr-o camera se joaca un pisoi cu doi pisici, un catelus care tine ın gurao papusa si un baietel, calare pe un calut de lemn. Cate picioare participa la joc?(Clasa a II-a) Alexandru Dumitru Chiriac, elev, Iasi

Solutie. La joc participa pisoiul cu cei doi pisici, catelusul si baietelul. In totalparticipa la joc 4 + 4 + 4 + 4 + 2 = 18 picioare.

P.168. Exista numerele naturale a, b, c, d astfel ıncat a + b + c + d = 123 sia : b = b : c = c : d = 1?(Clasa a III-a) Amalia Cantemir, eleva, Iasi

Solutie. Din a : b = b : c = c : d = 1, rezulta a = b = c = d. Concluzionam canumarul 123 trebuie sa se ımparta exact la 4, ceea ce este fals.

P.169. Calculeaza diferenta urmatoare, fara a efectua parantezele: (2 + 4 + 6 +8 + . . .+ 1000)− (1 + 3 + 5 + 7 + . . .+ 999) =(Clasa a III-a) Madalina Bucsa, eleva, Iasi

Solutie. De la 1 la 1000 sunt 500 numere pare si 500 numere impare. Exercitiuldevine (2− 1) + (4− 3) + (6− 5) + . . .+ (1000− 999) = 1 + 1 + 1 + . . .+ 1| z

de 500 ori

= 500.

P.170. Doi frati au cumparat un teren ın forma de patrat pe care l-au ımpartitın doua dreptunghiuri egale. Fiecare doreste sa ımprejmuiasca propriul teren cu gard.Cat mai are de lucru fiecare, daca primul a realizat 430 m, al doilea 470 m, iarperimetrul patratului este de 1000 m?(Clasa a III-a) Dragos Iacob, elev, Iasi

55

Page 60: Revista (format .pdf, 4.1 MB)

Solutie. Latura patratului este de 1000m : 4 = 250m. Imprejmuirea fiecarui fratecuprinde jumatate din perimetrul patratului si ınca jumatate din portiunea comunade gard. Primul frate mai are de lucrat (500m+125m)− 430m = 195m, iar al doilea(500m+ 125m)− 470m = 155m.

P.171. Daca a+b+c = 175 si a+2c = 200, calculati produsul (2a+b+3c) ·(c−b).(Clasa a IV-a) Inst. Marian Ciuperceanu, Craiova

Solutie. 2a+b+3c = (a+b+c)+(a+2c) = 175+200 = 385. Din a+b+c = 175si a + c + c = 200, rezulta c − b = 200 − 175 = 25. Asadar, (2a + b + 3c) · (c − b) =375× 25 = 275× 100 : 4 = 9375.

P.172. Cate numere abc au suma cifrelor 7 si pot fi rotunjite cu numarul ab0?(Clasa a IV-a) Maria Nastasiu, eleva, Iasi

Solutie. Cifra unitatilor poate fi 0, 1, 2, 3 sau 4. Daca c = 0, atunci a + b = 7si gasim numerele 160, 250, 340, 430, 520, 610 si 700. Daca c = 1, atunci a + b = 6si obtinem numerele 151, 241, 331, 421, 511 si 601. Analog, ın celelalte cazuri gasimnumerele 142, 232, 322, 412, 502; 133, 223, 313, 403; 124, 214 si 304. In total, exista 25de numere cu proprietatile din enunt.

P.173. Se formeaza sirul de numere: 34, 334, 344, 3334, 3444, . . .. Cate cifre de3 are numarul de pe locul 2008?(Clasa a IV-a) Petru Asaftei, Iasi

Solutie. Numerele de pe locurile pare au o singura cifra de 4, restul fiind cifre de3. Astfel, numarul de pe locul 2008 are 2008 : 2 + 1 = 1005 cifre de 3.

Clasa a V-aV.102. Un ıntreprinzator doreste sa cumpere un numar de frigidere de la un

angrosist, pe care urmeaza sa le transporte catre firma sa cu ajutorul unui camion demare tonaj, care consuma 10 l de motorina la 100 km (1l de motorina costa 3 lei).Intreprinzatorul poate opta ıntre doi furnizori: A vinde frigiderul cu 1000 lei/buc.,iar B vinde acelasi produs cu 990 lei/buc., ınsa are depozitul mai departe decat A, lao distanta pe sosea AB = 150 km.

a) Daca ıntreprinzatorul doreste sa cumpere 20 de frigidere, ce furnizor va alege?b) La ce numar de frigidere costurile de achizitie nu depind de furnizor?

Marian Ciuperceanu, CraiovaSolutie. Fie n numarul de frigidere achizitionate, iar x cheltuielile de transport

de la firma ıntreprinzatorului pana la furnizorul A. Cheltuielile de transport panala furnizorul B vor fi x + 3 · 10 · 3 = x + 90 (distanta AB se parcurge dus-ıntors, ıntotal 300 km). Chletuielile totale cu furnizorul A vor fi CA = x + 1000n, iar cele cufurnizorul B vor fi CB = x+ 90 + 990n.

a) Daca n = 20, atunci CA = x+20000, iar CB = x+19890, prin urmare CB < CA.Furnizorul ales va fi B.

b) Avem ca CA = CB , de unde se obtine n = 9.

V.103. Se considera numerele naturale m =3x+ 5

2x+ 2, a =

2y + 5

3, b =

5z + 2

5,

unde x, y, z ∈ N. Demonstrati ca m nu poate fi divizor al lui a, dar poate fi divizor allui b.

Claudiu Stefan Popa, Iasi

56

Page 61: Revista (format .pdf, 4.1 MB)

Solutie. Pentru cam ∈ N, trebuie sa avem ca 2x+2 | 3x+5, de unde 2x+2 | 2(3x+5)− 3(2x+ 2), adica 2x+ 2 | 4. Gasim ca x ∈ 0, 1; daca x = 0, atunci m =

5

2/∈ N,

iar daca x = 1, atunci m = 2. Presupunand ca 2 | a, s-ar obtine ca 2y + 5 = 6k, cuy, k ∈ N, absurd (membrul stang este impar, iar cel drept par). Pentru z = 2, avemca b = 4, numar care se divide cu 2.

V.104. Scrieti numarul 2008 ca suma de trei cuburi perfecte pare. (Gasiti toateposibilitatile!)

Veronica Plaesu si Dan Plaesu , Iasi

Solutie. Deoarece 2008 = 23 · 251, este destul sa-l scriem pe 251 ca suma detrei cuburi perfecte. Cel mai mare dintre cele trei cuburi nu poate depasi 261 = 63,deoarece 73 = 343 > 251. Dupa o analiza a cazurilor posibile, gasim doar doua situatiifavorabile: 251 = 13+53+53 si 251 = 23+33+63. In concluzie, 2008 = 23+103+103 =23 + 63 + 123.

V.105. Se considera numarul a = 7 + 72 + 73 + . . .+ 72009.a) Demonstrati ca a nu poate fi patrat perfect.b) Aflati restul ımpartirii lui a la 400.

Damian Marinescu, TargovisteSolutie. a) Cum a se divide cu 7, dar nu si cu 72, ınseamna ca nu poate fi patrat

perfect.b) Avem ca a = 7+72(1+7+72+73)+ . . .+72006(1+7+72+73) = 7+400(72+

. . .+ 72006), deci restul ımpartirii lui a la 400 este 7.

V.106. Sa se determine numarul natural a si cifra b, daca (a+3) ·200b = a ·2009.Enache Patrascu, Focsani

Solutie. Cum 2009 = 72 · 41, rezulta ca (a + 3) · 200b...72 si (a + 3) · 200b

...41.Evident ca b ≤ 8 (deoarece a + 3 > a), iar dintre numerele 2000, 2001, . . . , 2008,

nicunul nu se divide nici cu 72, nici cu 41. Deducem ca a + 3...7 si a + 3

...41, prinurmare α+3 = 287k. Inlocuind, obtinem ca 200b · k = 7(287k− 3), de unde k(2009−200b) = 21. De aici, (k, b) ∈ (21, 8); (7, 6); (3, 2), deci solutiile problemei sunt (a, b) ∈(6024, 8); (2006, 6); (858, 2).

O alta rezolvare se poate da ıncercand pentru b fiecare dintre valorile 0, 1, 2, . . . , 8;se obtin astfel noua ecuatii simple, doar trei dintre acestea avand solutii naturale.

V.107. Daca n ∈ N\0, 1 este dat, determinati x, y ∈ N∗ pentru care x(x+2y+1) = 2n · 135.

Petru Asaftei, IasiSolutie. Daca x = 2i, 1 ≤ i ≤ n− 1, atunci am avea ca 2i + 2y + 1 = 2n−i · 135,

contradictie (membrul stang este impar, iar cel drept este par). Analog se arata canu putem avea x = 2i · p, unde 1 ≤ i ≤ n − 1, p ∈ D135\1. Ramane ca x = 2n · p,cu p ∈ D135. Cum x < x+2y+1, trebuie cercetate doar cazurile ın care p ∈ 1, 3, 5.Daca x = 2n, obtinem ca y = 67−2n−1, iar y ∈ N∗ doar pentru n ≤ 7. Daca x = 2n ·3,atunci y = 22−3 ·2n−1, care este numar natural cand n ≤ 3. In sfarsit, daca x = 2n ·5,atunci y = 13−5 ·2n−1, solutie convenabila pentru n ≤ 2. In concluzie, obtinem 3, 2, 1sau 0 perechi (x, y), dupa cum n = 2, n = 3, n ∈ 4, 5, 6, 7, respectiv n ≥ 8.

57

Page 62: Revista (format .pdf, 4.1 MB)

V.108. Pe tabla sunt scrise numerele 2, 0, 0, 9. Putem sterge de pe tabla oricaredoua numere, scriind ın loc succesorii acestora. Este posibil ca, ın urma mai multoroperatii de acest fel, sa obtinem patru numere egale?

Catalin Budeanu, IasiSolutie. Suma numerelor de pe tabla creste cu 2 la fiecare pas. Dupa a n-a

operatie, ea devine 2n+2+0+0+9 = 2n+11, numar care este impar, deci nu poatefi suma a patru numere egale.

Clasa a VI-aVI.102. O asociatie de locatari este formata din trei familii care au consumat

ıntr-o luna 27m3, 16m3, respectiv 4m3 de apa potabila. Din consumul total, pentru38m3 de apa trebuie platita o taxa de canalizare, care se ımparte proportional cuconsumul fiecarei familii. Daca pretul apei este de 1, 6 lei/m3, taxa de canalizare estede 0, 56 lei/m3 si fiecarei sume i se aplica T.V.A. de 19 %, aflati ce suma trebuie saplateasca fiecare familie (efectuati calculele cu doua zecimale exacte).

Petru Asaftei, IasiSolutie. Pentru apa, prima familie plateste 27 × 1, 6 × 1, 19 = 51, 4 lei, a doua

16 × 1, 6 × 1, 19 = 30, 46 lei, iar a treia 4 × 1, 6 × 1, 19 = 7, 61 lei (am trunchiatrezultatele la cifra sutimilor, conform cerintelor problemei). Observand ca 38m3

reprezinta 80, 85% din 47m3 (unde 47m3 este consumul total), deducem ca pentrucanalizare prima familie plateste 0, 8085 × 28 × 0, 56 × 1, 19 = 14, 54 lei, a doua0, 8085× 16× 0, 56× 1, 19 = 8, 62 lei, iar a treia 0, 8085× 4× 0, 56× 1, 19 = 2, 15 lei.In total, prima familie are de platit 65, 94 lei, a doua 39, 08 lei, iar a treia 9, 76 lei.

VI.103. Sa se determine numarul prim p si numerele ıntregi a si x pentru care(x− a)(x− 1)(a− 1) = p.

Gheorghe Iurea, IasiSolutie. Daca p ≥ 3, atunci toti divizorii sai ar fi impari, deci x−a, x− 1 si a− 1

vor fi numere impare. Insa (x− 1)− (x− a) = a− 1, egalitate care nu poate avea locpentru trei numere impare. Ramane sa studiem cazul p = 2; atunci a−1 ∈ ±1,±2,deci a ∈ −1, 0, 2, 3. Considerand fiecare dintre cele patru situatii, gasim solutiilea = −1, x = 0 si a = 2, x ∈ 0, 3.

VI.104. Determinati numerele prime p si q, stiind ca exista x, y ∈ N∗ astfel ıncatx2 + y2 = p, iar x+ y + 1 = q.

Andrei Cozma, elev, BucurestiSolutie. Observam ca p − q = x(x − 1) + y(y − 1) − 1, prin urmare p − q este

numar impar. Rezulta ca unul dintre numerele p sau q este par, deci egal cu 2. Dacap = 2, din x2 + y2 = 2 obtinem ca x = y = 1, prin urmare q = 3. Daca q = 2, atuncix+ y = 1, fals, deoarece x, y ∈ N∗. In concluzie, p = 2 si q = 3.

VI.105. Sa se arate ca numarul N = 332009 − 33

2008

se poate scrie ca produs atrei numere naturale consecutive.

Dan Nedeianu, Drobeta-Tr. SeverinSolutie. Notand a = 33

2008

, observam ca 332009

= 332008·3 = a3. Astfel, N =

a3 − a = a(a2 − 1) = (a− 1) · a · (a+ 1), ceea ce ıncheie rezolvarea.

VI.106. Se considera unghiul ÔxOy si punctele A,B ∈ (Ox,C,D ∈ (Oy astfel ıncat

58

Page 63: Revista (format .pdf, 4.1 MB)

A ∈ (OB), iar C ∈ (OD). Mediatoarele segmentelor [AB] si [CD] se intersecteaza ın

S, iar ÕSAB ≡ ÕSCD.a) Demonstrati ca BC = AD.

b) Daca, ın plus, punctele B,D si S sunt coliniare, iar m(ÕSAB) = 60, aratatica AC⊥SC ⇔ BS = 2 · SD.

Romanta Ghita si Ioan Ghita, Blaj

Solutie. a) Triunghiurile SAB si SCD fiind isoscele, din ipoteza ÕSAB = ÕSCDA

Q C

SB

x y

O

.D

obtinem ca 180 − 2m(ÕSAB) = 180 − 2m(ÕSCD), deci ÕASB =ÕCSD. Atunci ÕBSC ≡ ÕASD, ceea ce ne arata ca BSC ≡ASD (L.U.L.), de unde BC = AD.

b) In ipotezele acestui punct, triunghiurile ABS,CDS siOBD sunt echilaterale, iar AS∥Oy, CS∥Ox. Daca AC ⊥ SC,

cum m(ÕASC) = 60, atunci m(ÕCAS) = 30. In triunghiuldreptunghic ACS vom avea ca AS = 2CS, prin urmare BS =2SD. Reciproc, daca BS = 2SD, atunci AB = 2SC. Notand cu Q mijlocul lui AB,obtinem ca AQ = CS. Avem si ca AQ∥CS, deci ACSQ este paralelogram. In plus,CA ⊥ AB (o mediana a unui triunghi echilateral este si ınaltime); deducem ca ACSQeste dreptunghi, de unde AC ⊥ SC.

VI.107. Se considera A,B,C,D,E, F sase puncte ın plan astfel ıncat AB =CD = CF = DF = 3cm, BC = BE = CE = 5cm, iar AD = 11cm. Stabiliti catedrepte determina cele sase puncte.

Gabriel Popa, IasiSolutie. Cum AB + BC + CD = AD, rezulta ca punctele A,B,C si D sunt

coliniare si se afla ın aceasta ordine pe dreapta pe care o determina. Mai observamsi faptul ca triunghiurile CDF si BCE sunt echilaterale. In cazul ın care acestetriunghiuri se afla ıntr-un acelasi semiplan fata de dreapta AB, cele sase punctedetermina 10 drepte: AB,AE,AF,BE,BF,CE,CF,DE,DF si EF . Daca puncteleE si F sunt separate de dreapta AB, atunci C,E si F vor fi coliniare, prin urmareEF,CE si CF sunt una si aceeasi dreapta; ın acest caz, cele sase puncte determina 8drepte.

VI.108. Un ogar situat ın varful A al unei curti dreptunghiulare ABCD (AB =80m, BC = 160m), porneste ın urmarirea a trei iepuri aflati ın B,C si D, alergandde-a lungul gardurilor. Daca viteza ogarului este 4m/s, iar vitezele iepurilor sunt3m/s, aflati dupa cat timp reuseste ogarul sa prinda fiecare iepure.

Marian Ciuperceanu, CraiovaSolutie. Daca ogarul porneste catre varful B, va prinde iepurele aflat initial ın B

dupa 80 : (4− 3) = 80s. Iepurele din C va fi ajuns dupa (80 + 160) : (4− 3) = 240s,iar cel din D dupa (80 + 160 + 80) : (4− 3) = 320s. Daca ınsa ogarul alearga ın senscontrar, pornind ıntai catre D, va prinde iepurele de acolo dupa 160 : (4− 3) = 160s,apoi iepurele din C dupa (160 + 80) : (4 − 3) = 240s, iar iepurele aflat initial ın Bdupa (160 + 80 + 160) : (4− 3) = 400s.

59

Page 64: Revista (format .pdf, 4.1 MB)

Clasa a VII-aVII.102. In urma unui razboi dus ıntre doua triburi de canibali, ın mainile

ınvingatorilor raman zece prizonieri, printre care si capetenia ınvinsilor. Seful detrib al ınvingatorilor alege, pentru prepararea cinei, cativa prizonieri (macar unul), laıntamplare. Care este probabilitatea ca seful tribului ınvins sa ramana ın viata?

Gabriel Popa, Iasi

Solutie. Numarul cazurilor egal posibile este numarul submultimilor nevide alemultimii cu 10 elemente a prizonierilor, deci 210 − 1 = 1023. Capetenia ınvinsilorramane ın viata daca se alege o submultime nevida a multimii formata din ceilaltinoua prizonieri, deci exista 29 − 1 = 511 cazuri favorabile. Probabilitatea ceruta este511

1023.

VII.103. Aflati numerele ıntregi x si y pentru care y− 4x+6 < 0, 2y−x− 2 > 0si 3y + 2x− 24 < 0.

Gheorghe Iurea, Iasi

Solutie. Scriem cele trei conditii din ipoteza astfel:

y < 4x− 6 (1) ; y >x+ 2

2(2); y <

24− 2x

3(3).

Din (1) si (2) deducem cax+ 2

2< 4x − 6, deci x > 2. Din (2) si (3) rezulta ca

x+ 2

2<

24− 2x

3, de unde x < 6, prin urmare x ∈ 3, 4, 5. Din (1), (2) si (3),

ınlocuind pe rand x cu 3, 4 si 5, obtinem solutiile (3, 3); (3, 4); (4, 4); (4, 5); (5, 4).

Nota. La nivelul clasei a IX-a, se poate da o solutie folosind ımpartirea planuluiın regiuni.

VII.104. Spunem ca un numar natural are proprietatea (P ) daca este prim, celputin egal cu 5 si se poate scrie ca suma de doua patrate perfecte. Daca numerelep1, p2, . . . , pn au proprietatea (P ), aratati ca numarul A = p1+p2+. . .+pn+n

2−n+2nu poate fi patrat perfect.

Cosmin Manea si Dragos Petrica, Pitesti

Solutie. Un patrat perfect poate fi M4 sau M4 + 1, deci o suma de doua patrateva fi M4,M4 + 1 sau M4 + 2. Daca dorim ca aceasta suma de patrate sa fie numarprim cel putin egal cu 5, atunci ea va fi neaparat de forma M4 + 1. Deducem caA = (M4+1)+(M4+1)+. . .+(M4+1)+n2−n+2 =M4+n+n

2−n+2 =M4+n2+2

si, cum n2 =M4 sau n2 =M4 + 1, atunci A =M4 + 2 sau A =M4 + 3, prin urmareA nu poate fi patrat perfect.

VII.105. Pentru x, y ∈ R, definim a(x, y) = min(2x− y2, 2y − x2). Aratati ca:

a) a(x, y) ≤ 1,∀x, y ∈ R; b) maxa(x, y)|x, y ∈ R = 1.

Ovidiu Pop, Satu Mare

Solutie. a) Daca, prin absurd, ar exista x, y ∈ R pentru care a(x, y) > 1, arınsemna ca 2x − y2 > 1 si 2y − x2 > 1, pentru anumite valori ale numerelor x si y.Prin adunare, am obtine ca (x− 1)2 + (y − 1)2 < 0, imposibil.

b) Folosind a) si observand ca a(1, 1) = 1, rezulta cerinta.

60

Page 65: Revista (format .pdf, 4.1 MB)

VII.106. Se considera paralelogramul ABCD,E si F mijloacele laturilor [AB],respectiv [AD], G = CE∩BD, H = CF∩BD, P = FG∩BC, Q = EH∩CD.Aratati ca 3EF = 2PQ.

Mirela Marin, Iasi

Solutie. Din DHF ∼ BHC, deducem caDH

HB=DF

BC=

1

2. Cum DHQ ∼

DQ

C

P

HF

A E B

G

BHE, obtinem caDQ

BE=

DH

HB=

1

2, prin urmare

DQ =1

4AB, adica

DQ

DC=

1

4. Analog se arata ca

BP

BC=

1

4, deci PQ∥BD (reciproca teoremei lui Thales),

iarPQ

BD=CQ

CD=

3

4(teorema fundamentala a asemanarii). Astfel, 2PQ =

3

2·BD =

3 · 12BD = 3FE (deoarece [FE] este linie mijlocie ın ABD).

VII.107. Fie ABC un triunghi cu m(ÒC) = 60, L proiectia lui A pe BC, Mproiectia lui B pe AC, iar D mijlocul lui [AB]. Demonstrati ca triunghiul DML esteechilateral.

Neculai Roman, Mircesti (Iasi)Solutie. In triunghiurile LAB si MAB, LD si respectiv MD sunt mediane,

D

B L

M

A

C

prin urmare LD =MD =1

2AB, deci DML este isoscel, la fel

ca si triunghiul ADM . Vom avea ca ÖAMD = bA si, cum patru-

laterul ABLM este inscriptibil, avem si ca ÕCML ≡ ÒB. Astfel,

m(ÖDML) = 180 − m(ÖAMD) − m(ÕCML) = 180 − m( bA) −m(ÒB) = m(ÒC) = 60, deci DML va fi chiar echilateral.

VII.108. Consideram ın plan trei cercuri distincte, congru-ente, ale caror centre nu sunt coliniare. Construiti cu rigla si compasul un cerc lacare cercurile date sa fie tangente interior.

Adrian Corduneanu, IasiSolutie. Putem determina, folosind rigla si compasul, centrele celor trei cercuri

date; sa notam cu A,B si C aceste centre. Aflam, cu rigla si compasul, centrul Oal cercului circumscris triunghiului ABC si punctul M de intersectie al dreptei OAcu cercul de centru A, astfel ıncat OM > OA. Astfel, cercul de centru O si razaOM este cercul cautat: daca r este raza cercurilor date, iar N = OB ∩ C(B, r),P = OC ∩C(C, r), atunci ON = OB+BN = OA+ r = OM si analog OP = OM.Mai trebuie justificat ca C(O,OM) are cate un singur punct comun cu cercuriledate. Daca, de exemplu, ar exista un al doilea punct Q comun cercurilor C(O,OM) siC(A, r), atunci OQ < OA+AQ (inegalitatea triunghiului), deci OQ < OA+r = OMsi se ajunge la o contradictie.

Clasa a VIII-a

VIII.102. Rezolvati ın R ecuatia

x+ 2

x− 1

2

+

x− 2

x+ 1

2

− 26

5· x

2 − 4

x2 − 1= 0.

Vasile Chiriac, Bacau

61

Page 66: Revista (format .pdf, 4.1 MB)

Solutie. Se impune ca x ∈ R\1,−1. Daca u =x+ 2

x− 1, v =

x− 2

x+ 1, ecuatia

devine u2 + v2 − 26

5uv = 0 si, cum u si v nu pot fi simultan egale cu zero, putem

nota t =v

usi obtinem ca t2 − 26

5t+ 1 = 0, cu solutiile t1 =

1

5, t2 = 5. Daca v = 5u,

rezulta ca 2x2 − 9x− 4 = 0, de unde x1,2 =9±

√113

4, iar daca u = 5v, deducem ca

2x2 − 9x+ 4 = 0, deci x3 = 4, x4 =1

2. Ecuatia din enunt are patru solutii reale.

VIII.103. Aratati ca oricare ar fi n ∈ N∗, exista m ∈ N∗ astfel ıncat n4 ·m + 1este numar compus.

Lucian Tutescu si Ion Visan, Craiova

Solutia 1 (a autorilor). Pentru m = n4 +2, avem ca n4 ·m+1 = n8 +2n4 +1 =(n4 + 1)2 si, cum n4 + 1 ≥ 2, urmeaza concluzia problemei.

Solutia 2 (Titu Zvonaru). Daca n = 1, putem lua m = pq − 1, cu p, q ∈ N,p, q ≥ 2. Daca n > 1, luam m = n3k−4, k ∈ N, k ≥ 2 si vom avea ca n4 ·m + 1 =(nk)3 + 1 = (nk + 1)(n2k − nk + 1), unde ambele paranteze sunt cel putin egale cu 2.

VIII.104. Fie x, y, z ∈ R∗+ astfel ıncat x2y2+y2z2+z2x2 = 3x2y2z2. Demonstrati

ca1

x2 + x+ 1+

1

y2 + y + 1+

1

z2 + z + 1≤ 1.

Razvan Ceuca, elev, Iasi

Solutie. Problema este oarecum ınrudita cu G89 din RecMat2/2005 sau cuVIII.66 din RecMat 1/2006: observand ca x2 + x+ 1 ≥ 3x si tinand seama de faptul

ca x > 0, obtinem ca1

x2 + x+ 1≤ 1

3xsi ınca doua relatii analoage. Astfel, membrul

stang este majorat de1

3

1

x+

1

y+

1

z

. Folosind inegalitatea dintre media aritmetica

si cea patratica,1

3

1

x+

1

y+

1

z

≤r

1

3

1

x2+

1

y2+

1

z2

. Insa conditia din ipoteza

se poate scrie sub forma1

x2+

1

y2+

1

z2= 3 si de aici urmeaza inegalitatea din enunt.

Egalitatea se atinge pentru x = y = z = 1.

VIII.105. Determinati x, y ∈ N∗ pentru care x3 − y3 = 3xy + 17.

Liviu Smarandache si Ion Visan, Craiova

Solutia 1 (a autorilor). Cum 3xy+17 ∈ N∗, rezulta ca x > y, deci exista p ∈ N∗

astfel ıncat x = y + p. Inlocuind ın relatia din enunt, obtinem ca 3(p− 1)y2 + 3p(p−1)y + p3 − 17 = 0. Observam ca 3(p − 1)y2 ≥ 0 si 3p(p − 1)y ≥ 0, prin urmarep3 − 17 ≤ 0, adica p ∈ 1, 2. Daca p = 1 se ajunge la contradictia −16 = 0, iarpentru p = 2 deducem ca y2+2y−3 = 0, ecuatie a carei singura solutie naturala estey = 1. Rezulta ca x = 3, deci solutia ecuatiei din enunt este perechea (3, 1).

Solutia 2 (Gheorghe Iurea). Rezolvam ecuatia ın numere ıntregi. Notam d = x−y,p = xy, cu d, p ∈ Z. Cum x3 − y3 = (x− y)3 + 3xy(x− y) = d3 + 3dp, ecuatia devine

d3 + 3dp = 3p + 17. Rezulta ca 3p =17− d3

d− 1=

16

d− 1− d2 − d − 1, deci d − 1 este

62

Page 67: Revista (format .pdf, 4.1 MB)

divizor al lui 16. Analizand cazurile posibile, determinam d si p si apoi aflam solutiileecuatiei initiale: (x, y) ∈ (3, 1); (−1,−3).

VIII.106. In tetraedul V ABC, avem AB = 4cm,BC = 5cm,CA = 6cm, iar

ariile fetelor V AB, V BC si V CA sunt egale cu15√7

4cm2. Calculati sinusurile un-

ghiurilor ÕAV B,ÕBV C si ÕCV A.Vlad Emanuel, student, Bucuresti

Solutie. Calculand aria triunghiului ABC cu formula lui Heron, obtinem ca

aceasta este15√7

4cm2, prin urmare tetraedrul V ABC este echifacial. Rezulta ca

V A = BC = 5cm, V B = CA = 6cm, iar V C = AB = 4cm, de unde sinÕAV B =2AV AB

V A · V B=

√7

8, sinÕBV C =

2 · AV BC

V B · V C=

5√7

32, iar sinÕCV A =

2AV CA

V C · V A=

3√7

16.

VIII.107. Fie ABCD un tetraedru, iar m1,m2 si m3 lungimile bimedianelor sale.Demonstrati ca 3(AB2 +AC2 +AD2 +BC2 + CD2 +DB2) ≥ 4(m1 +m2 +m3)

2.D.M. Batinetu-Giurgiu, Bucuresti

Solutie. Se stie ca ın orice tetraedru ABCD are loc identitatea 4(m21+m

22+m

23) =

AB2+AC2+AD2+BC2+CD2+DB2 (a se vedea, de exemplu, D. Branzei, S. Anita,C. Cocea - Planul si spatiul euclidian, Ed. Academiei, Bucuresti, 1986). Folosindinegalitatea dintre media aritmetica si cea patratica, obtinem ca 3(m2

1 +m22 +m2

3) ≥(m1 +m2 +m3)

2, de unde cerinta problemei. Egalitatea se atinge atunci cand m1 =m2 = m3.

VIII.108. Intr-un reper cartezian xOy, se considera punctele Aij(i, j), unde1 ≤ i, j ≤ 5. Determinati numarul triunghiurilor care au ca varfuri trei dintre puncteledate.

Gabriel Popa, Iasi

Solutie. Cum avem 5 · 5 = 25 de puncte, putem considera25 · 24 · 23

6= 2300

de multimi formate din cate trei puncte. Pentru a gasi numarul triunghiurilor, tre-buie sa eliminam multimile formate din puncte coliniare. Punctele Ai1, i = 1, 5,

genereaza5 · 4 · 3

6= 10 multimi de cate trei puncte coliniare; aceeasi situatie are

loc pe fiecare dintre cele cinci orizontale, cinci verticale, precum si pe cele douadiagonale A11A55 si A15A51. Pe fiecare dintre directiile A12A45, A21A54, A14A41 siA25A52 avem cate 4 multimi de trei puncte coliniare, iar pe fiecare dintre directiileA31A53, A13A35, A13A31, A53A35, A11A53, A12A54, A13A55, A13A51, A14A52, A15A53,A11A35, A21A45, A31A55, A31A15, A41A25 si A51A35, exista cate o singura multimeformata din trei puncte coliniare. Astfel, numarul multimilor care trebuie eliminateeste 10 · 12 + 4 · 4 + 1 · 16 = 152. Raman 2300− 152 = 2148 de triunghiuri.

Clasa a IX-aIX.96. Determinati triunghiurile ın care tangentele unghiurilor se exprima prin

numere naturale. (In legatura cu X.78 din RecMat 1/2007.)Titu Zvonaru, Comanesti

Solutie. Fie A unghiul cel mai mic al triunghiului; atunci A ≤ π

3, deci tgA ≤

√3

63

Page 68: Revista (format .pdf, 4.1 MB)

si, cum tgA ∈ N, rezulta ca tgA = 1, adica A =π

4. Mai departe, din tgA + tgB +

tgC = tgA · tgB · tgC, obtinem ca (tgB − 1)(tgC − 1) = 2, de unde B = arctg 2,C = acrtg 3 (sau invers).

IX.97. Demonstrati ca ın orice triunghi are loc inegalitatea m2ahbhc +m2

bhcha +

m2chahb ≥ 4S2

2 +

r

2R

.

Catalin Cristea, Craiova

Solutie. Observam ca m2ahbhc + m2

bhcha + m2chahb = 4S2

2(b2 + c2)− a2

4bc+

2(a2 + c2)− b2

4ac+

2(a2 + b2)− c2

4ab

. Prin aplicarea inegalitatii Cebısev pentru siruri

de monotonii contrare, deducem ca2(b2 + c2)− a2

4bc+2(a2 + c2)− b2

4ac+2(a2 + b2)− c2

4ab≥

1

3· 3(a2 + b2 + c2) · 1

4

1

bc+

1

ac+

1

bc

≥ 9

4, ultima relatie obtinandu-se cu ajutorul

inegalitatii mediilor. Inegalitatea de demonstrat se deduce imediat, tinandu-se seamaca R ≥ 2r.

IX.98. Aflati a, b, c ∈ R, a = 0, pentru care |ax2 + bx+ c| ≤x− 1

a

2

,∀x ∈ R.Marian Ursarescu, Roman

Solutia I (Paul Georgescu). Pentru x =1

a, obtinem ca

1a +b

a+ c

≤ 0, de

unde c = −1

a− b

a. Substituind ın inegalitatea din enunt, obtinem ca |ax2 + bx −

1

a− b

a| ≤

x− 1

a

2

, deci

ax− 1

a

x+

1

a

+ b

x− 1

a

≤ x− 1

a

2

. Notand

x − 1

a= y, obtinem ca |ay2 + (b + 2)y| ≤ y2, ∀y ∈ R, de unde |ay + (b + 2)| ≤ |y|,

∀y ∈ R∗. Rezulta ca b + 2 = 0 si |ay| ≤ |y|, ∀y ∈ R∗, deci |a| ≤ 1. Urmeaza ca

a ∈ [−1, 0) ∪ (0, 1], b = −2, c =1

a.

Solutia a II-a (a autorului). Pentru x =1

a, obtinem ca

1a +b

a+ c

≤ 0, de

unde 1 + b + ac = 0. Conform ipotezei, avem ca ax2 + bx + c ≤x− 1

a

2

, deci

(a−1)x2+

b− 2

a

x+ c− 1

a2≤ 0, ∀x ∈ R. Acest fapt se petrece daca si numai daca

a−1 ≤ 0 si ∆ ≤ 0, adica atunci cand a < 1 si (ab+2)2−4(a−1)(a2c−1) ≤ 0. Inlocuindb = −1−ac, ultima conditie conduce la a2(1+ac)2−4a(1+ac)−4a3c+4a+4a2c ≤ 0,prin urmare a2(1− ac)2 ≤ 0, de unde ac = 1. Se observa usor ca, ın ipoteza ac = 1,

are loc inegalitatea ax2 + bx2 + c ≥ −x− 1

a

2

daca si numai daca a ≥ −1. In

concluzie, a ∈ [−1, 0) ∪ (0, 1], b = −2 si c =1

a.

IX.99. Fie k ∈ [0, 1), n ∈ N∗ si numerele αi ∈ R∗, βi ∈ R, εi ∈ −1, 1, i = 1, n,

64

Page 69: Revista (format .pdf, 4.1 MB)

astfel ıncat ε1α1 + ε2α2 + . . .+ εnαn = 0. Rezolvati ecuatia

|α1x+ β1|+ |α2x+ β2|+ . . .+ |αnx+ βn| = k|ε1β1 + ε2β2 + . . .+ εnβn|.Dumitru Mihalache si Gabi Ghidoveanu, Barlad

Solutie. CumnXi=1

εi(αix + βi) = x

nXi=1

εiαi

!+

nXi=1

εiβi =nXi=1

εiβi, rezulta ca nXi=1

εi(αix+ βi)

=

nXi=1

εiβi

, de undenXi=1

|αix + βi| ≥

nXi=1

εiβi

si atunci

k ·

nXi=1

εiβi

≥ nXi=1

εiβi

. DacanXi=1

εiβi = 0, obtinem k ≥ 1, prin urmare ecuatia nu

are solutii. DacanXi=1

εiβi = 0, ecuatia data este echivalenta cu sistemul αix+ βi = 0,

i = 1, n. Acest sistem nu are solutii daca (α1, a2, . . . , αn) si (β1, β2, . . . , βn) nu sunt

proportionale si are solutia x = −γ, unde γ =β1α1

=β2α2

= . . . =βnαn

, ın caz contrar.

IX.100. Fie (an)n≥1 si (bn)n≥1 doua siruri de numere reale, cu an = 0,∀n ≥ 1

si 3 ·nXk=1

(akb2k − a2kbk) =

nXk=1

ak

!3

−nXk=1

a3k,∀n ≥ 1. Demonstrati ca, pentru orice

n ≥ 1, exista αn ∈ 0, 1 astfel ıncat bn = αn(a1+ . . .+an)−(1−αn)(a1+ . . .+an−1).

Marian Tetiva, Barlad

Solutie. Pentru n = 1, relatia din enunt devine 3a1b1(b1 − a1) = 0, deci b1 = 0(si luam α1 = 0) sau b1 = a1 (si alegem α1 = 1). Pentru n ≥ 2, scadem dinrelatia din enunt pe aceea obtinuta din ea prin ınlocuirea lui n cu n− 1; ajungem la

3(anb2n − a2nbn) =

nXk=1

ak

!3

n−1Xk=1

ak

!3

− a3n ⇔ 3(anb2n − a2nbn) = 3

n−1Xk=1

ak

!2

·

an+3

n−1Xk=1

ak

!a2n ⇔ an

bn −

nXk=1

ak

! bn +

n−1Xk=1

ak

!= 0. Deoarece an = 0, de aici

rezulta fie ca bn =nXk=1

ak (deci se poate lua αn = 1), fie ca bn = −n−1Xk=1

ak (deci putem

alege αn = 0), ceea ce ıncheie solutia.

Autorul remarca faptul ca este adevarata si reciproca afirmatiei din eneunt.

Clasa a X-aX.96. Daca a, b, c sunt numere reale pozitive cu suma 1, demonstrati ca ab · bc · ca

+ ba · cb · ac ≤ 2(ab+ bc+ ca).

Dorin Marghidanu, Craiova

Solutie. Folosim inegalitatea dintre media aritmetica ponderata si media geo-metrica ponderata: p1a+ p2b+ p3c ≥ ap1 · bp2 · cp3 , ∀p1, p2, p3 > 0 cu p1 + p2 + p3 = 1Considerand p1 = b, p2 = c si p3 = a, rezulta ca ba+ cb+ ac ≥ ab · bc · ca. Luand apoip1 = c, p2 = a si p3 = b, obtinem ca ca+ bc+ ab ≥ ba · cb · ac. Adunand aceste relatii,se obtine inegalitatea din enunt.

65

Page 70: Revista (format .pdf, 4.1 MB)

X.97. Fie a, b, c ∈ C∗ numere complexe distincte astfel ıncat (a− b)3 = (b− c)3 =(c− a)3. Aratati ca |2a− b− c| = |2b− c− a| = |2c− a− b|.

Dan Nedeianu, Drobeta-Tr. Severin

Solutie. Conditia data este echivalenta cu

b− c

a− b

3

=

c− a

a− b

3

= 1. Cum

a − b = b − c (altfel b =a+ c

2si, folosind relatia din enunt, s-ar deduce ca a = c),

b−c = c−a si c−a = a−b, gasim ca b−c = zε si c−a = zε2, unde ε este radacina cubicaa unitatii, iar z = a− b. Deducem ca |2a− b− c| = |2b− c−a| = |2c−a− b| = |z| ·

√3.

X.98. Fie Ai(zi), i = 1, 3 varfurile unui triunghi din planul xOy si P (z) un punctdin acest plan (zi si z sunt afixele punctelor Ai, respectiv P ). Sa se arate ca P estesituat ın interiorul triunghiului A1A2A3 sau pe una din laturile sale daca si numaidaca exista αi ≥ 0, i = 1, 3, astfel ıncat α1 + α2 + α3 = 1 si z = α1z1 + α2z2 + α3z3.

Adrian Corduneanu, Iasi

Solutie. Punctul P este situat ın interiorul triunghiului A1A2A3 sau pe unadin laturile sale daca si numai daca exista M ∈ [A1A2] cu P ∈ [MA3]. DeoareceM ∈ [A1A2] ⇔ ∃t ∈ [0, 1] astfel ıncat zM = (1− t)z1 + tz2 si P ∈ [MA3] ⇔ ∃s ∈ [0, 1]cu proprietatea z = (1 − s)z3 + szM , rezulta ca z = (1 − t)sz1 + stz2 + (1 − s)z3.Considerand α1 = s(1− t), α2 = st si α3 = 1− s, obtinem αi ≥ 0, α1 + α2 + α3 = 1si z = α1z1 + α2z2 + α3z3, ceea ce ıncheie demonstratia.

X.99. Consideram triunghiurile echilaterale ABC si A1B1C1 si construim tri-unghiurile echilaterale AA1A2, BB1B2, CC1C2, AB1A3, BC1B3, CA1A3, AC1A4,BA1B4 si CB1C4; toate triunghiurile citate sunt orientate pozitiv. Fie puncteleM2 ∈ A2B, N2 ∈ B2C, P2 ∈ C2A, M3 ∈ A3B, N3 ∈ B3C, P3 ∈ C3A, M4 ∈ A4B,

N4 ∈ B4C si P4 ∈ C4A astfel ıncatM2A2

M2B=N2B2

N2C=P2C2

P2A=M3A3

M3B=N3B3

N3C=

P3C3

P3A=M4A4

M4B=N4B4

N4C=P4C4

P4A. Demonstrati ca triunghiurile M2N2P2,M3N3P3

si M4N4P4 sunt echilaterale si au acelasi centru.

Catalin Tigaeru, Suceava

Solutie. Notam afixul fiecarui punct care apare, cu litera mica ce ıi corespunde.Scriem conditiile ca cele 11 triunghiuri care apar ın ipoteza sa fie echilaterale:

a+ εb+ ε2c = 0; a1 + εb1 + ε2c1 = 0;a+ εa1 + ε2a2 = 0; b+ εb1 + ε2b2 = 0, c+ εc1 + ε2c2 = 0;a+ εb1 + ε2a3 = 0, b+ εc1 + ε2b3 = 0, c+ εa1 + ε2a3 = 0,a+ εc1 + ε2a4 = 0, b+ εa1 + ε2b4 = 0, c+ εb1 + ε2c4 = 0,

unde ε este radacina primitiva de ordin trei a unitatii. Demonstram ca triunghiurileAiBiCi, i = 2, 3, 4, sunt echilaterale; trebuie verificate relatiie ai + εbi + ε2ci = 0,i = 2, 3, 4. Vom da justificarea doar pentru i = 2 :

a2 + εb2 + ε2c2 = −(εa+ ε2a1)− ε(εb+ ε2b1)− ε2(εc+ ε2c1) =

= −ε(a+ εb+ ε2c)− ε2(a1 + εb1 + ε3c1) = 0.

66

Page 71: Revista (format .pdf, 4.1 MB)

Fie k valoarea comuna a rapoartelor egale din enunt. Obtinem ca mi =1

1 + kai +

k

1 + kb, ni =

1

1 + kbi+

k

1 + kc, pi =

1

1 + kci+

k

1 + ka, i = 2, 3, 4. Atunci, mi+ εni+

εpi =k

1 + k· 1ε(εb+ε2c+ε3a) = 0, i = 2, 3, 4, prin urmare MiNiPi sunt echilaterale.

Notam cuG siGi, i = 1, 4, centrele (de greutate) ale triunghiurilor ABC, respectiv

AiBiCi, i = 1, 4. Observam ca gi =1

3(ai + bi + ci) = −(εg + ε2g1), i = 2, 3, 4, deci

triunghiurile AiBiCi, i = 2, 3, 4, au acelasi centru, fie acesta G, de afix g = −εg−ε2g1.Din relatia g+εg+ε2g1 = 0, deducem ca G,G si G1 formeaza un triunghi echilateral.

Notam cuGi centrele triunghiurilorMiNiPi, i = 2, 3, 4; avem ca gi =1

3(mi+ni+pi) =

1

1 + kg +

k

1 + kg, prin urmare MiNiPi, i=2, 3, 4, au acelasi centru Gk, plasat pe

latura GG a triunghiului echilateral GGG1, pe care o ımparte ın raportul k.

X.100. Demonstrati ca ın orice triunghi ABC are loc inegalitatea

1

sin2A(sinB + sinC)2+

1

sin2B(sinC + sinA)2+

1

sin2 C(sinA+ sinB)2≥ 4

3.

Marius Olteanu, Rm. Valcea

Solutie. Este cunoscuta inegalitatea1

(x+ y)2+

1

(y + z)2+

1

(z + x)2≥ 9

1

xy + yz + zx, ∀x, y, z > 0 (a se vedea, de exemplu, Old and New Inequalities de

T. Andreescu, G. Dospinescu, V. Cartoaje, M. Lascu, aparuta la GIL, Zalau, 2004,pg. 22, ex. 114). Inlocuind x = sinA sinB, y = sinA sinC, z = sinB sinC, obtinem

caX 1

sin2A(sinB + sinC)2≥ 9

4

1

sinA sinB sinC(X

sinA). Pe de alta parte, avem

ca sinA sinB sinC ≤sinA+ sinB + sinC

3

3

(inegalitatea mediilor), iar

sinA+ sinB + sinC

3≤ sin

A+B + C

3=

√3

2(inegalitatea lui Jensen aplicata functiei

sinus pe [0, π]). Inlocuind, rezulta concluzia problemei.

Clasa a XI-aXI.96. Fie ε radacina primitiva de ordin trei a unitatii, iar A,B ∈ M3(R) cu

det(A+ εB) = 0. Demonstrati ca det(A−B) = detA− detB.Dan Popescu, Suceava

Solutie. Consideram polinomul f ∈ R[X], f(X) = det (A+XB) = detA+αX+βX2 +(detB) ·X3. Cum f(ε) = 0, rezulta ca detA+αε+ β(−ε− 1)+detB = 0, deunde α = β = detA + detB. Calculand f(−1) prin cele doua modalitati de scriereale lui f , obtinem ca f(−1) = det (A−B) = detA− detB.

XI.97. Fie n ≥ 3 un numar natural. Aratati ca pentru orice k ∈ 2, 3, . . . , n−1,exista A ∈ Mn(0, 1) astfel ıncat Ap = In, ∀p ∈ 1, 2, . . . , k − 1 si Ak = In.

Gheorghe Iurea, Iasi

67

Page 72: Revista (format .pdf, 4.1 MB)

Solutie. Consideram B =

0 1 0 . . . 00 0 1 . . . 0. . . . . . . . . . . . . .0 0 0 . . . 11 0 0 . . . 0

∈ Mk(0, 1). Se constata

ca, pentru p ∈ 1, 2, . . . , k − 1, avem ca Bp = (bij), unde b1,p+1 = b2,p+2 = . . . =

bk−p,p = 1, bk−p+1,1 = bk−p+2,2 = . . . = bk,p = 1, iar bij = 0 ın rest. In plus, Bk = Ik.

Atunci, matricea A =

B 00 In−k

verifica cerintele problemei.

XI.98. Demonstrati ca functia f :0,π

2

→ R, f(x) = ln

É1− cosx

1 + cosxeste

concava si, folosind eventual acest lucru, aratati ca ın orice triunghi ascutitunghic

ABC are loc inegalitatea1− cosA

1 + cosA· 1− cosB

1 + cosB· 1− cosC

1 + cosC≤ 1

27.

Bogdan Victor Grigoriu, Falticeni

Solutie. Functia f este de doua ori derivabila, iar f ′′(x) = − cosx

sin2 x< 0, ∀x ∈

0,π

2

, prin urmare f este concava. Aplicand inegalitatea lui Jensen, obtinem ca

f

A+B + C

3

≥ 1

3[f(A) + f(B) + f(C)], deci

ln

Ê1− cos π31 + cos π3

≥ 1

3ln

1− cosA

1 + cosA· 1− cosB

1 + cosB· 1− cosC

1 + cosC

,

rezulta imediat din monotonia functiei logaritmice.

Nota autorului. In aceeasi maniera se poate demonstra ca, ın orice triunghi

ABC, are loc inegalitatea1− sinA

1 + sinA· 1− sinB

1 + sinB· 1− sinC

1 + sinC≤ 1

(2 +√3)6

.

XI.99. Studiati convergenta sirului (vn)n≥1 definit prin vn+1 =(vcn + d)1/c

vn,

∀n ≥ 1, unde v1, c si d sunt numere reale pozitive date.

Gheorghe Costovici si Adrian Corduneanu, Iasi

Solutie. Vom demonstra ca limn→∞

vn = l, unde l =

1 +

√1 + 4d

2

1/c

. Daca

v1 = l, se demonstreaza prin inductie matematica faptul ca vn = l, ∀n ≥ 1. In cazulın care v1 ∈ (0, l), se arata (tot prin inductie) ca v2n−1 ∈ (0, l) si v2n ∈ (l,+∞),∀n ∈ N∗, apoi ca subsirul (v2n−1)n≥1 este strict crescator, ın timp ce (v2n)n≥1 estestrict descrescator. Urmeaza ca exista si sunt finite α = lim

n→∞v2n−1, β = lim

n→∞v2n si,

prin trecere la limita ın relatiile de recurenta, obtinem ca α =(βc + d)1/c

β, iar β =

(αc + d)1/c

α. De aici, α =

αc + d

αc+ d

1/c

:(αc + d)1/c

α, deci αc =

αc + d+ dαc

αc + d,

prin urmare α2c−αc− d = 0, de unde gasim ca α = l. Asemanator se arata ca β = l.In sfarsit, analog se trateaza cazul ın care v1 ∈ (l,+∞).

68

Page 73: Revista (format .pdf, 4.1 MB)

Nota. De fapt, sirul un = vcn, ∀n ∈ N∗, verifica relatia de recurenta un+1 =un + d

un, ∀n ∈ N∗, recurenta omografica care se studiaza ın mod uzual.

XI.100. Demonstrati ca

(x+ 1)

sin

π

x+ 1− cos

π

x+ 1

< x

sin

π

x− cos

π

x

,∀x ∈ [2,∞).

Petru Raducanu, IasiSolutie. Inegalitatea de demonstrat este echivalenta cu f(x+1)−f(x) < g(x+1)−

g(x), x ∈ [2,∞), unde f, g : [2,+∞) → R, f(t) = t sinπ

t, g(t) = t cos

π

t, ∀t ∈ [2,∞).

Aplicand teorema lui Lagrange functiei f pe intervalul [x, x + 1], x ≥ 2, obtinem

c ∈ (x, x+1) pentru care f(x+1)− f(x) = f ′(c) = hπc

, unde h(t) = sin t− t cos t,

t ∈0,π

2

i. Observam ca h′(t) = t sin t > 0, deci h este crescatoare, astfel ca h(t) ≤

hπ2

= 1, ∀t ∈

0,π

2

i. Deducem ca f(x + 1) − f(x) < 1, ıntrucat

π

c∈0,π

2

.

Analog se demonstreaza ca g(x+ 1)− g(x) > 1, ∀x ≥ 2, ceea ce ıncheie rezolvarea.

Clasa a XII-a

XII.96. Rezolvati ın S5 ecuatia x11 =

1 2 3 4 55 3 4 1 2

.

Liviu Smarandache si Ionut Ivanescu, Craiova

Solutie. Notam σ =

1 2 3 4 55 3 4 1 2

si observam ca ordσ = 5. Din x11 = σ

rezulta ca x55 = e, prin urmare ordx|55, deci ordx ∈ 1, 5, 11, 55. Pe de alta parte,cum ordS5 = 120, atunci ordx|120 si ramane ca ordx ∈ 1, 5. Daca ordx = 1, arrezulta ca x = e si se ajunge la contradictie e = e11 = x11 = σ. Daca ordx = 5,obtinem ca σ = x11 = x · x5 · x5 = x, adica singura solutie a ecuatiei date este x = σ.

XII.97. Fie ak ∈ R, k = 0, n, iar m ∈ (0,∞) astfel ıncatmXk=0

akm+ k

= 0. Sa se

arate ca ecuatia a0 + a1x+ . . .+ anxn = 0 admite solutie ın intervalul (0, 1).

Mihail Bencze, BrasovSolutie. Aplicam teorema de medie functiei f : [0, 1] → R, f(x) = (a0 + a1x +

. . .+ anxn) · xm−1, pentru care

Z 1

0f(x)dx =

nXk=0

akm+ k

= 0.

XII.98. Determinati primitivele functiei f : (0, π) → R, f(x)=sin3n−1 x · cosn−1 x

sin4n x+ cos4n x,

n ∈ N∗.I.V. Maftei, Bucuresti si Mihai Haivas, Iasi

Solutie. Fie I =

Zsin3n−1 x · cosn−1 x

sin4n x+ cos4n xdx, J =

Zcos3n−1 x · sinn−1 x

sin4n x+ cos4n xdx, unde

x ∈ (0, π). Observam ca

I + J =

Zsinn−1 x cosn−1 x(sin2n x+ cos2n x)

sin4n x+ cos4n xdx =

69

Page 74: Revista (format .pdf, 4.1 MB)

=

Z tgn−1 x · 1

cos2 x+ ctgn−1x · 1

sin2 xtg2n x+ ctg2nx

dx =1

n

Z(tgn x− ctgnx)′

(tgn x− ctgnx)2 + 2dx =

=1

n√2arctg

tgn x− ctgnx√2

+ C.

Analog se obtine ca I − J =1

2n√2ln

tgn x+ ctgnx−√2

tgn x+ ctgnx+√2

+ C. Adunand membru

cu membru cele doua relatii, gasim valoarea lui I.

XII.99. Se considera functia f : (0,∞) → (0, 1) continua si descrescatoare si

sirul strict crescator (an)n≥1 de numere reale pozitive, astfel ıncat sirul

an+1

an

n≥1

este strict descrescator. Definim In =1

an

Z an+1

an

f(x)dx, ∀n ∈ N∗.

a) Demonstrati ca (In)n≥1 este un sir descrescator.

b) Daca limn→∞

an+1

an= 1, calculati lim

n→∞In.

Cosmin Manea si Dragos Petrica, PitestiSolutie. Din teorema de medie, pentru fiecare n ∈ N, gasim cn ∈ (an, an+1) astfel

ca In =1

an(an+1 − an)f(cn) =

an+1

an− 1

f(cn).

a) Cum cn < an+1 < cn+1 iar f este descrescatoare, urmeaza ca (f(cn))n≥1 este

descrescator. De asemenea,

an+1

an− 1

n≥1

este descrescator, de unde (In)n≥1, este

descrescator, ca produs de doua siruri descrescatoare strict pozitive.b) Cum (f(cn))n≥1, este descrescator si marginit, el admite o limita finita l.

Urmeaza ca limn→∞

In = 0 · l = 0.

XII.100. In raport cu reperul xOy, consideram punctele A(a, 0), B(0, b) si T ∈(AB), unde a > 0, b > 0. Determinati parabola y = λx2 + µ care este tangenta ın Tla AB, stiind ca aria suprafetei determinata de parabola si axele de coordonate estemaxima.

Adrian Corduneanu, Iasi

Solutie. Ecuatia dreptei AB este y =b

a(a − x), deci T are coordonatele x0 ∈

(0, a), y0 =b

a(a−x0). Din conditiile de tangenta λx20+µ =

b

a(a−x0) si 2λx0 = − b

a,

rezulta λ = − b

2ax0si µ =

b(2a− x0)

2a. Parabola de ecuatie y = − b

2ax0x2+

b(2a− x0)

2a

va taia axa Ox ın punctul M(xM , 0), unde xM =È

2ax0 − x20. Aria ceruta este

S =

Z xM

0(λx2 + µ)dx =

λ

3x3M + µxM =

b

3a

Èx0(2a− x0)3. Se observa ca S este

maxima pentru x0 =a

2, deci parabola catata are ecuatia y = − b

a2· x2 + 3b

4.

70

Page 75: Revista (format .pdf, 4.1 MB)

Solutiile problemelor pentru pregatireaconcursurilor propuse ın nr. 1/2009

A. Nivel gimnazial

G156. Daca a, b, c ∈ R∗+,

1

a+

1

b+

1

c≤ 3, demonstrati ca

a2 + 1√a2 − a+ 1

+

b2 + 1√b2 − b+ 1

+c2 + 1√c2 − c+ 1

≥ 6.

I.V. Maftei, Bucuresti si Mihai Haivas, Iasi

Solutia I (a autorilor). Observam caa2 + 1√a2 − a+ 1

=√a2 − a+ 1+

a√a2 − a+ 1

2√a, ∀a ∈ R∗

+ si analog pentru celelalte doua fractii ale sumei din membrul stang.

Rezulta ca aceasta suma este cel putin egala cu 2(√a+

√b+

√c). Pe de alta parte,

√a ≥ 2a

1 + a(inegalitatea MG ≥MH, aplicata numerelor a si 1), deci

2(√a+

√b+

√c) ≥ 4

a

1 + a+

b

1 + b+

c

1 + c

≥ 36 · 11+aa + 1+b

b + 1+cc

=36

3 + 1a + 1

b +1c

≥ 36

3 + 3= 6,

de unde inegalitatea din enunt.Solutia a II-a (Oana Adascalitei si Florina Toma, eleve, Iasi). Vom

face aceeasi demonstratie ın doi pasi, cu alte argumente. CumÈa(a2 − a+ 1) ≤

a+ a2 − a+ 1

2=

a2 + 1

2, atunci

1Èa(a2 − a+ 1)

≥ 2

a2 + 1, deci

a2 + 1√a2 − a+ 1

2√a si ınca doua inegalitati similare. Apoi, din inegalitatea C-B-S, obtinem ca1√a+

1√b+

1√c

2

≤1

a+

1

b+

1

c

(1 + 1 + 1), de unde

1√a+

1√b+

1√c

≤ 3.

Insa

1√a+

1√b+

1√c

(√a+

√b+

√c) ≥ 9, prin urmare

√a+

√b+

√c ≥ 3 si astfel

rezulta inegalitatea din enunt.

G157. Spunem ca un numar natural are proprietatea (P) daca se poate scrie casuma a trei patrate perfecte nenule si ca are proprietatea (Q) daca se poate scrie casuma a patru patrate perfecte nenule.

a) Dati exemple de numere naturale care au: numai proprietatea (P ); numaiproprietatea (Q); atat proprietatea (P ) cat si proprietatea (Q).

b) Daca a, b, c ∈ N∗ au suma para si oricare dintre ele este diferit de suma celor-laltor doua, demonstrati ca numarul a2 + b2 + c2 are proprietatea (Q).

Ovidiu Pop, Satu MareSolutie. a) 6 = 12+12+22 are numai proprietatea (P ), 7 = 12+12+12+22 are

numai proprietatea (Q), iar 30 = 12 + 22 + 52 = 12 + 22 + 32 + 42 are si proprietatea(P ), si proprietatea (Q).

71

Page 76: Revista (format .pdf, 4.1 MB)

b) Cum suma si diferenta au aceeasi paritate, concluzia problemei rezulta din iden-

titatea a2+ b2+ c2 =

a+ b+ c

2

2

+

−a+ b+ c

2

2

+

a− b+ c

2

2

+

a+ b− c

2

2

.

G158. Se considera ecuatia x2+y2+z2 = (x−y)2+(y−z)2+(z−x)2, x, y, z ∈ N.a) Aratati ca ecuatia are o infinitate de solutii.b) Daca (x, y, z) este solutie a ecuatiei, demonstrati ca fiecare dintre numerele

xy, yz, zx si xy + yz + zx este patrat perfect.Liviu Smarandache, Craiova

Solutie. a) De exemplu, putem considera x = 1, y = a2, z = (a+ 1)2, cu a ∈ N.b) Din relatia din enunt, obtinem ca (x+y+z)2 = 4(xy+yz+zx), de unde rezulta

ca xy + yz + zx este patrat perfect. Apoi, tot din relatia din enunt, rezulta succesivx2+y2+z2 = 2(xy+yz+zx) ⇔ x2−2x(y+z)+(y+z)2−4yz = 0 ⇔ (x−y−z)2 = 4yz,deci yz este patrat perfect. La fel se arata ca xy si zx sunt patrate perfecte.

G159. Aflati ultimele doua cifre ale numerelor (70n+ 6) · 6n−1, n ∈ N.Ion Sacaleanu, Harlau

Solutie (Gheorghe Iurea). Notam an = (70n + 6) · 6n−1, n ∈ N. Deoarecean+1 − an = 50(7n + 9) · 6n−1 = M100, n ∈ N∗, rezulta ca toate numerele an auaceleasi ultime doua cifre. Cum a1 = 76, deducem ca toate numerele considerate setermina ın 76.

G160. Se considera multimile A = 1, 2, 3, . . . , 2009, B =

§a

a+ d+

b

b+ d+

c

c+ d

a, b, c, d ∈ A, a, b, c, d distincte

ªsi C =

§d

a+ d+

d

b+ d+

d

c+ d

a, b, c, d ∈ A,

a, b, c, d distincte

ª. Determinati A ∩ B ∩ C. (In legatura cu E: 13650 din G.M. 5-

6/2008.)Andrei Cracana, elev, Iasi

Solutie. Se constata usor ca, daca x =a

a+ d+

b

b+ d+

c

c+ d∈ B, atunci 3−x ∈ C.

In cazul ın care x ∈ A∩B∩C, obtinem ca x ∈ 1, 2. Vom arata ca 1, 2 ⊂ A∩B∩Csi astfel va rezulta ca A∩B∩C = 1, 2. Pentru (a, b, c, d) = (14, 21, 30, 42), obtinemca x = 1, deci 1 ∈ B si 3 − 1 = 2 ∈ C. Pentru (a, b, c, d) = (75, 375, 875, 125), avemca x = 2, prin urmare 2 ∈ B si 3 − 2 = 1 ∈ C si astfel rezolvarea problemei esteıncheiata.

G161. Fie M multimea numerelor de forma abc, cu a · b · c = 0. Determinaticardinalul maxim al unei submultimi N a lui M astfel ıncat x+ y = 1109, ∀x, y ∈ N.

Petru Asaftei si Gabriel Popa, IasiSolutie. Numarul cerut este 405. Cum fiecare dintre cifrele a, b, c este nenula,

cardinalul lui M este 9 · 9 · 9 = 729. Dintre elementele lui M , 9 · 9 · 1 = 81 se terminaın 9. Grupam celelalte elemente (ın numar de 729 − 81 = 648) ın perechi de forma(x, 1109 − x), obtinand 324 de perechi. Daca se considera o submultime N a lui Mcu cel putin 406 elemente, cum 324+ 81 = 405, din principiul cutiei rezulta ca macardoua dintre elementele lui N apartin unei aceleiasi perechi (x, 1109−x), deci au suma1109. Luand cate un element din fiecare dintre perechile considerate, precum si toate

72

Page 77: Revista (format .pdf, 4.1 MB)

numerele din M care se termina ın 9, obtinem o submultime a lui M de cardinal 405,care verifica proprietatea din enunt.

G162. Putem ınlocui un triplet de numere ıntregi (a, b, c) cu unul dintre tripletele(2b + 2c − a, b, c), (a, 2a + 2c − b, c) sau (a, b, 2a + 2b − c). Aratati ca daca pornimde la tripletul (31329, 24025, 110224) si efectuam succesiv asemenea ınlocuiri, se obtinmereu triplete formate numai din patrate perfecte.

Marian Tetiva, Barlad

Solutie. Daca pornim de la un triplet de forma (x2, y2, (x + y)2) si efectuamoricare dintre cele trei transformari, obtinem tot triplete de forma (m2, n2, (m+n)2).Intr-adevar, daca ınlocuim x2 cu 2y2+2(x+ y)2−x2 = (x+2y)2, atunci m = x+2y,n = −y; daca ınlocuim y2 cu 2x2+2(x+y)2−y2 = (2x+y)2, putem consideram = −x,n = 2x+y; ın sfarsit, daca ınlocuim (x+y)2 cu 2x2+2y2−(x+y)2 = (x−y)2, vom luam = x, n = −y. Observam acum ca tripletul initial este de forma (x2, y2, (x + y)2),unde x = 177, y = 155 si de aici urmeaza concluzia problemei.

G163. Fie ABC un triunghi cu m( bA) = 90 si punctele B1 ∈ (AC) si C1 ∈ (AB).Aratati ca axa radicala a cercurilor de diametre [BB1] si [CC1] trece prin punctul Adaca si numai daca B1C1∥BC.

Neculai Roman, Mircesti (Iasi)

Solutie. Fie M al doilea punct de intersectie dintre cercul de diametru [BB1]A

M

CB

N

B1C

1

si dreapta AB, iar N proiectia lui C pe AB. Unghiul ×BMB1

fiind ınscris ıntr-un semicerc, avem ca B1M ⊥ AB, de unde

MB1∥NC, deciAM

AN=AB1

AC. Atunci: A se afla pe axa radicala

a celor doua cercuri ⇔ AM ·AB = AC1 ·AN ⇔ AM

AN=AC1

AB⇔

AB1

AC=AC1

AB⇔ B1C1∥BC si astfel rezolvarea problemei este

completa.

G164. Fie B, b numere reale date, cu B > b > 0. Dintre toate trapezele circum-scriptibile care au lungimile bazelor B si b, determinati-l pe cel de arie maxima.

Claudiu Stefan Popa, Iasi

Solutie. Fie ABCD trapez circumscriptibil, cu bazele AB = B, CD = b; notam

A

D

E F

C

B

x = BC, y = AD. Vom avea ca B + b = x + y. ConstruimCF ⊥ AB, CE∥AD, cu E,F ∈ AB. Calculam h = CF ex-primand ın doua moduri aria triunghiului BCE. Observamca CE = y, BE = B − b, iar semiperimetrul BCE este1

2[x + y + (B − b)] =

1

2[(B + b) + (B − b)] = B. Din for-

mula lui Heron, ABCE =ÈB(B − x)(B − y)(B − (B − b)) =È

Bb[B2 −B(x+ y) + xy] =ÈBb[B2 −B(B + b) + xy] =

ÈBb(xy −Bb). Pe de

alta parte, ABCE =1

2BE · h =

1

2(B − b) · h, prin urmare h =

2ÈBb(xy −Bb)

B − b.

Cum AABCD =1

2h(B + b), iar lungimile B si b sunt date, atunci aria trapezului este

73

Page 78: Revista (format .pdf, 4.1 MB)

maxima cand produsul xy este maxim. Suma x + y fiind constanta (este egala cuB + b), produsul xy va fi maxim cand x = y, deci ın cazul trapezului isoscel. Princalcul direct, ınaltimea acestuia este h =

√Bb.

G165. Fie ABC un triunghi isoscel (AB = AC), M mijlocul laturii [BC], iar Pun punct ın interiorul triunghiului ABM . Notam D = BP ∩AC, E = CP ∩AB.Demonstrati ca BE < CD si PE < PD.

Cristian Pravat, Iasi si Titu Zvonaru, Comanesti

Solutie. Fie F = AP ∩ BC si x =BF

FC, y =

CD

DA, z =

AE

EB. Atunci, din

A

B CF M

DD

E

P

teorema lui Ceva, xyz = 1, iar x < 1 deoarece P ∈ IntABM .

Avem ca BE < CD ⇔ AB · 1

z + 1< AC · y

y + 1⇔ y + 1 <

yz + y ⇔ 1 < yz si, cum ultima inegalitate este adevarata,rezulta prima parte a concluziei. Fie D′ simetricul lui D fatade AM ; trapezul BCDD′ este isoscel, deci inscriptibil, iarE se va afla ın interiorul cercului circumscris. Deducem cam(ÖDD′C) < m(ÕDEC), de unde m(ÕPDE) < m(ÖPDD′) =

m(ÖDD′C) < m(ÕDEP ), prin urmare PE < PD.

B. Nivel licealL156. Fie M un punct exterior cercului C de centru O si raza R. Notam cu

T1, T2 punctele de contact cu cercul ale tangentelor duse din M la C si cu A punctulde intersectie a dreptei OM cu cercul C, astfel ıncat A /∈ [OM ]. Determinati puncteleM cu proprietatea ca se poate construi un triunghi cu segmentele [MT1], [MT2] si[MO], dar nu se poate construi un triunghi cu [MT1], [MT2] si [MA].

Temistocle Bırsan, IasiSolutie. A se vedea Recreatii Matematice 1/2009, pg. 41.

L157. In planul ABC definim transformarea P → P ′ astfel: 1. punctul P seproiecteaza pe dreptele BC,CA,AB ın D,E si respectiv F ; 2. simetricele punctelorD,E, F ın raport cu mijloacelor laturilor [BC], [CA] si respectiv [AB] se noteazaD′, E′, F ′; 3. P ′ este punctul de concurenta a perpendicularelor ın D′, E′, F ′ peBC,CA si respectiv AB. Aratati ca transformarea P → P ′ coincide cu simetriaın raport cu O, centrul cercului circumscris ABC.

Temistocle Bırsan, IasiSolutie (Daniel Vacaru, Pitesti). Cum avem de-a face cu doua izometrii, este

suficient sa demonstram ca transformatele punctelor A,B,C sunt aceleasi. Sa vedemcine sunt D,E, F cand P = A; avem ca D = A′, E = A si F = A, unde A′ esteproiectia lui A pe BC. Observam apoi ca D′ este simetricul lui A′ fata de mijlocullui [BC], E′ coincide cu C si F ′ cu B. Construind perpendicularele ın B pe AB si ınC pe AC, se obtine patrulaterul inscriptibil ABP ′C. Deducem ca P ′ coincide cu A′,simetricul lui A fata de O, centrul cercului circumscris. Rationamentul este analog ıncazul ın care P = B, respectiv P = C si astfel se ınchide demonstratia.

Nota. Aceasta problema este, pana la diferenta de formulare, tocmai Propozitia3 din articolul Simetria fata de centrul cercului cricumscris unui triunghi de BogdanIonita si Titu Zvonaru (G.M. - 3/1997, p. 98). Faptul ne este adus la cunostinta de

74

Page 79: Revista (format .pdf, 4.1 MB)

dl. Titu Zvonaru. Autorul problemei regreta si ısi cere scuze pentru acest incident.(Mentionam ınca o solutie diferita de cea din articol: Perpendicularele ın D si D′

pe BC determina ın cercul C(O,OP ) un dreptunghi, deci perpendiculara ın D′ treceprin simetricul fata de O al punctului P etc.).

L158. In interiorul triunghiului ABC cu latura [BC] fixa si varful A mobil, con-

sideram punctul T asfel ıncat ÕATB ≡ ÕBTC ≡ ÕCTA. Determinati pozitia punctului

A ın planul triunghiului pentru care m(ÕBAC) = α <5π

6, iar suma distantelor de la

T la varfurile triunghiului este maxima.Catalin Calistru, Iasi

Solutie. Remarcam faptul ca T ese tocmai punctul lui Toricelli asociat triun-

P

A

B C

T

ghiului ABC. Astfel, daca PAB este echilateral, con-struit ın exteriorulABC, atunci punctele P, T si C suntcoliniare, iar TA + TB + TC = CP (vezi, de exemplu,L. Niculescu si V. Boskoff - Probleme practice de geome-trie, Ed. Tehnica, 1990). Folosind teorema cosinusuluiın triunghiurile APC si ABC, obtinem ca

CP 2 = AP 2 +AC2 − 2AP ·AC · cosA+

π

3

=

= BC2 + 2AB ·AC · cosA− 2AB ·AC · cosA+

π

3

=

= BC2 + 2AB ·ACcosA− cos

A+

π

3

=

= BC2 + 4AB ·AC · sinA+

π

6

sin

π

6= BC2 +BC ·

sin(A+ π6 )

sinA· ha.

Cum BC este constanta, iar sinA+

π

6

> 0 (deoarece α <

6), deducem ca CP

este maxim atunci cand ha este maxim. Insa A se misca pe un arc capabil de unghiulα, prin urmare pozitiile cautate ale punctului A sunt date de intersectiile mediatoareisegmentului [BC] cu arcele capabile de unghiul α, construite pe [BC].

L159. Daca a, b, c ∈ R∗+ si x ∈

0,π

2

, demonstrati inegalitatea

a

sinx

x

3

+ b

sinx

x

2

+ c

sinx

x

+ 3

3√abc

tg x

x

> 6 · 3

√abc.

D.M. Batinetu-Giurgiu, BucurestiSolutie. Din inegalitatea mediilor, rezulta ca

a

sinx

x

3

+ b

sinx

x

2

+ c

sinx

x

≥ 3

3

Êabc ·

sinx

x

6

= 33√abc

sinx

x

2

.

Pentru a obtine inegalitatea din enunt, ar fi suficient sa demonstram ca

sinx

x

2

+

tg x

x> 2, ∀x ∈

0,π

2

. Aceasta inegalitate, atribuita lui Wilker, poate fi gasita ın

G.M. 1/2007, pg. 1.

75

Page 80: Revista (format .pdf, 4.1 MB)

Nota. Solutie corecta, bazata pe considerente de analiza matematica, s-a primitde la Dl. Daniel Vacaru, Pitesti.

L160. Demonstrati ca ın orice triunghi are loc inegalitatea

ma +mb +mc ≥ 6r

ma

mb +mc+

mb

ma +mc+

mc

ma +mb

≥ 9r.

Marius Olteanu, Rm. Valcea

Solutie. Avem ca1

r=

1

ha+

1

hb+

1

hcsi ma ≥ ha, mb ≥ hb, mc ≥ hc. In plus, ın

orice triunghi cu laturile a, b, c, are loc inegalitatea

(a+ b+ c)

1

a+

1

b+

1

c

≥ 6

a

b+ c+

b

c+ a+

c

a+ b

(a se vedea, de exemplu, Algebraic Inequalities de V. Cartoaje, aparuta la GIL, Zalau,2006, problema 70, pg. 379). Cum medianele unui triunghi pot fi laturi ale unui alttriunghi, obtinem ca

1

r(ma +mb +mc) =

1

ha+

1

hb+

1

hc

(ma +mb +mc) ≥

1

ma+

1

mb+

1

mc

(ma +mb +mc) ≥ 6

ma

mb +mc+

mb

ma +mc+

mc

ma +mb

,

adica tocmai prima inegalitate ceruta. Pentru a doua inegalitate, folosim binecunos-

cutax

y + z+

y

z + x+

z

x+ y≥ 3

2, ∀x, y, z ∈ R∗

+ (Nesbitt).

L161. Daca a, b, c ∈ R∗+ si a+ b+ c = 1, demonstrati inegalitatea

3 +X (a− b)2 + (a− c)2

1 + a≤ 4(a2 + b2 + c2)

X 1

1 + a

.

Titu Zvonaru, ComanestiSolutie. Notand Q = a2 + b2 + c2, avem ca

4

1 + a− 3a

a2 + b2 + c2=

4a2 + 4b2 + 4c2 − 3a(2a+ b+ c)

(1 + a)(a2 + b2 + c2)=

=(b− a)(4b+ a)

Q(1 + a)+

(c− a)(4c+ a)

Q(1 + a)

si ınca doua identitati similare. Pe de alta parte,

(b− a)(4b+ a)

Q(1 + a)+

(a− b)(4a+ b)

Q(1 + b)=a− b

Q· 4a

2 − 4b2 + 3a− 3b

(1 + a)(1 + b)=

=(a− b)2

Q· 4a+ 4b+ 3

(1 + a)(1 + b)≥ (a− b)2

Q· 1 + a+ 1 + b

(1 + a)(1 + b)=

(a− b)2

Q(1 + a)+

(a− b)2

Q(1 + b).

76

Page 81: Revista (format .pdf, 4.1 MB)

Analog se obtin ınca doua minorari si deducem ca

4

1 + a+

4

1 + b+

4

1 + c− 3(a+ b+ c)

a2 + b2 + c2≥

≥ (a− b)2 + (a− c)2

(1 + a)Q+

(b− c)2 + (b− a)2

(1 + b)Q+

(c− a)2 + (c− b)2

(1 + c)Q,

inegalitate echivalenta cu cea din enunt. Egalitatea se atinge pentru a = b = c =1

3.

L162. Daca n ∈ Z∗ este fixat, rezolvati ın R ecuatiahxn

i=

[x]

n

.

Dumitru Mihalache si Gabi Ghidoveanu, BarladSolutie. Daca n ∈ N∗, multimea solutiilor ecuatiei este R, conform unei cunoscute

proprietati a partii ıntregi. Intr-adevar,hxn

i= k ⇔ kn ≤ x < (k + 1)n⇔ kn ≤ [x] < (k + 1)n⇔

[x]

n

= k.

In cazul ın care n ∈ Z\N∗, exista si sunt unice numerele q ∈ Z si r ∈ R, 0 ≤ r < |n|,astfel ca x = nq + r. Urmeaza cahx

n

i=hnq + r

n

i=hq +

r

n

i= q +

h rn

i,

[x]

n

=

[nq + r]

n

=

nq + [r]

n

= q +

[r]

n

.

Daca r = 0, atuncih rn

i=

[r]

n

= 0. Daca r ∈ (0, 1), atunci

h rn

i= −1, deoarece

r

n∈− 1

|n|, 0

, iar

[r]

n

= 0. Daca r ∈ [1, |n|), atunci r

n,|r|n

∈−1,− 1

|n|

, decih r

n

i=

[r]

n

= −1. De aici se obtine ca, pentru n ∈ Z\N∗, multimea solutiilor ecuatiei

din enunt este R\Sq∈Z

(nq, nq + 1).

L163. Fie a un numar ıntreg impar, iar n ∈ N∗. Aratati ca polinomul X2n + a2n

este ireductibil ın Z[X] ınsa, pentru orice numar prim p, polinomul redus modulo peste reductibil ın Zp[X].

Dorel Mihet, TimisoaraSolutie. Deoarece f(X+a) = X2n +C1

2naX2n−1+ . . .+C2n−1

2n a2n−1X+2a2

n

, iar

coeficientii binomiali C12n , . . . , C

2n−12n sunt toti pari, din criteriul lui Eisenstein rezulta

ca f(X + a) (si la fel f) este ireductibil peste Q, deci si peste Z (avand coeficientuldominant 1).

Daca p = 2, atunci bf = X2n + b1 este reductibil ın Z2[X], deoarece este de gradmai mare decat unu si are radacina 1. Fie p un numar prim impar; putem scrie cabf = X2n +da2n = (X2n−1

)2 − (−b1da2n) = (X2n−1

+ Õa2n−1)2 − b2Õa2n−1 ·X2n−1

=

= (X2n−1

− Õa2n−1)2 − (−b2Õa2n−1 ·X2n−1

).

77

Page 82: Revista (format .pdf, 4.1 MB)

In cazul ın care ba = b0, concluzia este imediata. In caz contrar, ba va fi generator algrupului ciclic (Z∗

p, ·), prin urmare −b1 = as, b2 = at si −b2 = as+t, pentru anumitenumere naturale s si t. Deoarece cel putin unul dintre numerele s, t si s + t estepar, rezulta ca unul dintre elementele −b1,b2,−b2 este patrat perfect ın Zp, deci bf va fireductibil ın Zp(X).

L164. O secventa x1, x2, . . . , xn, y1, y2, . . . , yn de 2n numere reale are proprietatea(P ) daca x2i + y2i = 1, ∀i = 1, n. Fie n ∈ N∗ astfel ıncat pentru orice secventa cuproprietatea (P ), exista 1 ≤ i < j ≤ n cu xixj + yiyj ≥ 0, 947. Determinati cea maibuna constanta α asa ıncat xixj + yiyj ≥ α, pentru orice secventa cu proprietatea(P ).

Vlad Emanuel, student, Bucuresti

Solutie. Consideram punctele Mk(xk, yk), k = 1, n si vectorii vk =−−−→OMk. Pentru

o secventa cu proprietatea (P), avem ca |vk| = 1, k = 1, n, deci xixj + yiyj = vi ·vj = |vi| · |vj | cos(×−→v i,−→v j) = cos(×−→v i,−→v j). Vom demonstra ca, ın ipotezele problemei,n ≥ 20. Presupunem, prin absurd, ca n ≤ 19 si alegem Mk-imaginile radacinilor de

ordin n ale unitatii. Atunci xixj+yiyj ≤ cos2π

n≤ cos

19< 0, 947, dupa cum se poate

constata cu ajutorul unui calculator. Fie deci n ≥ 20; deoareceX

m((Ú−→v k,−→v k+1)) =

2π, unde vn+1 = v1, ınseamna ca cel putin unul dintre unghiurile (Ú−→v k,−→v k+1) este cel

mult egal cu2π

n. Pentru acel unghi, xkxk+1+ykyk+1 ≥ cos

n≥ cos

π

10=

r5 +

√5

8si aceasta constanta nu poate fi ımbunatatita, caci putem lua n = 20 si Mk imaginile

radacinilor de ordin 20 ale unitatii. In concluzie, α =

r5 +

√5

8.

L165. Fie n ≥ 2 un numar natural. Determinati cel mai mare numar naturalm pentru care exista submultimile nevide si distincte A1, A2, . . . , Am ale lui A =1, 2, . . . , n, cu proprietatea ca fiecare element al lui A este continut ın cel mult kdintre ele, unde:

a) k = 2; b) k = n; c) k = n+ 1.Marian Tetiva, Barlad

Solutie. Fie xi, i = 1, n, numarul acelora dintre submultimile A1, A2, . . . , An careau i elemente si fie yj , j = 1,m, numarul elementelor lui A care se gasesc ın exactj dintre multimile A1, A2, . . . , Am. Avem evident ca x1 + x2 + . . . + xn = m, iary1+y2+ . . .+ym ≤ n (se poate sa fie elemente ale lui A care sa nu apartina niciuneiadintre submultimi). Egalitatea

(1) x1 + 2x2 + . . .+ nxn = y1 + 2y2 + . . .+mym

se obtine numarand ın doua feluri toate elementele care apar ın A1, A2, . . . , Am, in-cluzand repetitiile (si este adevarata chiar daca A1, A2, . . . , Am nu sunt distincte).

a) In ipoteza k = 2, avem ca yj = 0, ∀j ≥ 3, deci y1+y2 ≤ n si x1+2x2+. . .+nxn =y1 + 2y2. Atunci

2m− x1 = x1 + 2(m− x1) = x1 + 2(x2 + . . .+ xn) ≤

78

Page 83: Revista (format .pdf, 4.1 MB)

≤ x1 + 2x2 + . . .+ nxn = y1 + 2y2 ≤ 2(y1 + y2) ≤ 2n,

deci 2m ≤ 2n + x1 ≤ 3n (deoarece x1 nu poate depasi C1n, numarul submultimilor

lui A cu un singur element). Am obtinut astfel ca m ≤3n

2

. Demonstram ca

3n

2

este chiar maximul cautat: submultimile cu un element ale lui A si ınca

hn2

isubmultimi cu doua elemente, alese astfel ıncat sa fie doua cate doua disjuncte, sunt3n

2

submultini cu proprietatea ca fiecare element al lui A se afla ın cel mult doua

dintre ele.b) Avem ca yj = 0, ∀j ≥ n+ 1, deci y1 + y2 + . . .+ yn ≤ n, iar relatia (1) devine

x1 + 2x2 + . . .+ nxn = y1 + 2y2 + . . .+ nyn. Atunci

3m− 2x1 − x2 = x1 + 2x2 + 3(m− x1 − x2) = x1 + 2x2 + 3(x1 + . . .+ xn) ≤

≤ x1 + 2x2 + . . .+ nxn = y1 + 2y2 + . . .+ yn ≤ n(y1 + . . .+ yn) ≤ n2,

deci 3m ≤ n2 + 2x1 + x2 ≤ n2 + 2C1n + C2

n =3n2 + 3n

2. Obtinem, ın cele din urma,

ca m ≤ n(n+ 1)

2. Cum submultimile nevide cu cel mult doua elemente ale lui A

sunt ın numar de n+n(n− 1)

2=n(n+ 1)

2si au proprietatile din enunt, ınseamna ca

n(n+ 1)

2este maximul cautat.

c) Cu un rationament asemanator, obtinem ca m maxim esten(n+ 1)

2+hn3

i.

ERATA

Dintr-o eroare de editare, pe care o regretam si pentru care ne cerem scuze, din listamembrilor Comitetului de redactie al Recreatiilor Matematice a fost omis, ıncepand cunr. 1/2007, dl. Alexandru CARAUSU, pasionat sustinator al revistei. In spiritulcorectitudinii, aducem aceasta rectificare de ordin formal.

∗∗ ∗

Recreatii Matematice – 1/2009- p. 9, r. 5: ın loc de ”bisectoare” se va considera ”bisectoarea”;- p. 26, r. 6 de jos: ın loc de ”R” se va considera ”1”;- p. 79, r. 9: ın loc de ”xiyj + yiyj” se va considera ”xixj + yiyj”;- p. 81, r. 9 de jos: ın loc de ”xiyi + yiyj” se va considera ”xixj + yiyj”.

Recreatii Matematice – 2/2009

- p. 96, r. 11 de jos: ın loc de ”aα1 + . . .+ aαn” se va considera ”aβ1 + . . .+ aβn”;- p. 172, r. 11 de jos: ın loc de Hasedeu se va considera ”Hasdeu”.

79

Page 84: Revista (format .pdf, 4.1 MB)

Probleme propuse1

Clasele primareP.184. Vreau sa pun ıntr-o cutie bile albe si verzi, ın total 10, astfel ıncat bile

albe sa fie cel mult 6. In cate moduri pot face acest lucru?(Clasa I ) Inst. Maria Racu, Iasi

P.185. Ce litera urmeaza ın ınsiruirea logica VKUJT...?(Clasa I ) Andreea Amarandei, studenta, Iasi

P.186. Completati dreptunghiurile de mai jos cu numere asa ıncat suma nu-merelor scrise ın oricare trei dreptunghiuri alaturate sa fie aceeasi. Ce observati?

35 65 35

(Clasa a II-a) Alexandru Chiriac, student, Iasi

P.187. Soricelul Chit a primit un zar de la matusa Mit. El a aruncat zarul depatru ori, obtinand ın total 21 de puncte. Stiind ca la primele doua aruncari a obtinutın total 9 puncte, aflati cat a obtinut la fiecare aruncare. (Gasiti toate posibilitatile!)(Clasa a II-a) Ioana Maria Popa, eleva, Iasi

P.188. In exercitiul a+ a : a = . . ., exista o valoare a lui a pentru care putem saefectuam operatiile ın ordinea scrisa, fara a modifica rezultatul?(Clasa a III-a) Ionela Baragan, studenta, Iasi

P.189. Aflati numarul natural a stiind ca, daca se ımparte 25 la 8 − 3 × a, seobtine restul 1.(Clasa a III-a) Mariana Nastasia, eleva, Iasi

P.190. In gradina casei mele sunt cativa pomi. Daca ar fi de patru ori mai multidecat sunt, atunci ar depasi numarul 20 cu atat cat lipseste, de fapt, pentru a fi 20.Cati pomi sunt ın gradina?(Clasa a III-a) Inst. Dumitru Paraiala, Iasi

?

a - ?

a - ?

a - b

P.191. Compuneti o problema care sa se rezolve dupa schemaalaturata, cu numerele a si b convenabil alese.(Clasa a III-a) Amalia Cantemir, eleva, Iasi

P.192. Bunica are mere si pere. Daca mi-ar da un sfert din numarulmerelor si o optime din numarul perelor, as avea 35 de fructe. Daca mi-arda o optime din numarul merelor si o patrime din numarul perelor, as avea 40 fructe.Cate fructe are bunica?(Clasa a IV-a) Mihaela Galca, eleva, Iasi

P.193. Mai multe perechi, formate din cate o fata si cate un baiat, culeg alune.In fiecare pereche, alunele culese de baiat sunt fie de patru ori mai multe, fie de patruori mai putine decat cele culese de fata. Numarul alunelor culese ımpreuna de fete side baieti poate fi 2009? Dar 2010?(Clasa a IV-a) Mihaela Obreja si Ioan Lungu, Vaslui

1Se primesc solutii pana la data de 31 decembrie 2010.

80

Page 85: Revista (format .pdf, 4.1 MB)

P.194. Aratati ca exista un singur sir format din zece numere naturale consecutiveastfel ıncat suma a opt dintre numere sa fie egala cu dublul sumei celorlaltor doua.(Clasa a IV-a) Petru Asaftei, Iasi

P.195. Trei frati, Ionut, Andrei si Mihai, primesc lunar cate o aceeasi suma debani de la bunicul lor. Pe ascuns, bunica le da si ea aceeasi suma. Odata, unuldintre cei trei a spart un geam. Bunicul, crezandu-l vinovat pe Ionut, a ımpartit baniicuveniti lui celorlalti doi. Bunica a procedat la fel, ınsa a crezut ca cel vinovat esteMihai. Andrei, stiind ca el a spart geamul, a ımpartit jumatate din suma sa celor doifrati si a constatat ca ramane cu 60 de lei mai putin decat Ionut si Mihai la un loc.Ce suma de bani primea fiecare nepot de la bunicul?(Clasa a IV-a) Cosmin Serbanescu, student, Iasi

Clasa a V-aV.116. Se considera numarul a = (2n · 5n+1 + 4) : 36, n ∈ N∗. Determinati

valorile lui n pentru care a este numar natural, a carui scriere ın baza 10 are toatecifrele distincte.

Andrei Nedelcu, Iasi

V.117. Aratati ca A = 61001 se poate scrie ca diferenta a doua patrate perfecte.Damian Marinescu, Targoviste

V.118. Determinati numerele naturale a si n pentru care a2n − 9 = 8(9 + 92 +93 + . . .+ 92009).

Gabriela Popa, Iasi

V.119. Fie n ∈ N un numar a carui scriere ın baza 10 este de forma . . . 55.a) Aratati ca (n− 5)(n+ 5) se divide cu 1000.b) Aflati ultimele trei cifre ale lui n2.

Mihai Craciun, Pascani

V.120. Consideram numarul natural a = 12345 . . . 9899. Aflati restul ımpartiriilui a prin 45.

Elena Iurea, Iasi

V.121. Aratati ca1

2 · 3 · 4+

2

3 · 4 · 5+ . . .+

2010

2011 · 2012 · 2013<

1

4.

Tinuta Bejan, Iasi

V.122. Cate fractii ireductibile de forma20xy

2yexista?

Diana Gregoretti, Galati

Clasa a VI-aVI.116. Se considera unghiul ÕAOB cu masura de 126 si semidrepte (OM1, (OM2,

. . . , (OMn−1 interioare lui, astfel ıncat interioarele unghiurilor ÖAOM1, ØM1OM2, . . . ,ÙMn−1OB sunt disjuncte doua cate doua, iar m(ÖAOM1) = 2, m(ØM1OM2) = (22),

. . . ,m(ÙMn−1OB) = (2n). Daca (OM este bisectoarea unghiului ÖAOM4, determinati

masura complementului lui ÖAOM .Catalina Dragan, Galati

81

Page 86: Revista (format .pdf, 4.1 MB)

VI.117. Fie I centrul cercului ınscris ın triunghiul ABC, iar x = m(ÕBIC)−m( bA),y = m(ÔAIC)−m(ÒB), z = m(ÔAIB)−m(ÒC). Aratati ca numerele x, y si z sunt masurileunghiurilor unui triunghi ascutitunghic.

Constantin Apostol, Rm. Sarat

VI.118. In triunghiul isoscel ABC, cu m( bA) = 120, se noteaza cu D mijlocullaturii [AB]. Perpendiculara din D pe BC intersecteaza dreptele AC si BC ın E,

respectiv F. Bisectoarea unghiului ÕDEA taie BC ın G. Aratati ca BC = 4 · FG.Catalin Budeanu, Iasi

VI.119. Un numar natural N se termina ın 0 si are exact 323 de divizori. Aflatiultimele 16 cifre ale numarului N .

Mirela Obreja, Vaslui

VI.120. Demonstrati ca numarul A = 12 +22 +32 + . . .+20102 − 2000 se dividecu 3.

Nicolae Ivaschescu, Craiova

VI.121. Fie n ∈ N∗ si a1, a2, . . . , an numere naturale consecutive. Aratati casuma S = a1 + a2 + . . .+ an se divide cu n daca si numai daca n este numar impar.

Andrei Pasa, elev, Iasi

VI.122. Vom spune ca un numar natural este anti-Goldbach daca poate fi scrisca suma de doua numere compuse. Determinati toate numerele anti-Goldbach.

Ionel Nechifor, Iasi

Clasa a VII-a

VII.116. Calculati suma S =

12 − 2

3

+ 23 − 3

4

+ . . .+

20082009− 2009

2010

.Daniela Munteanu, Iasi

VII.117. Fie x, y numere reale astfel ıncat x > 2011, iar xy = x+ y. Aratati ca

partea fractionara a lui y este mai mica decat1

2010.

Claudiu Stefan Popa, Iasi

VII.118. Aratati ca x2010+1 ≥ x1010+x1000,∀x ∈ R. Cand se atinge egalitatea?Catalin Melinte, student, Iasi

VII.119. Consideram numarul natural A = 5n + 2 · 3n−1 + 1, n ∈ N∗.a) Demonstrati ca A se divide cu 8, oricare ar fi n ∈ N∗.b) Determinati n ∈ N∗ pentru care A se divide cu 40.

Ciprian Baghiu, Iasi

VII.120. Un poligon are 170 de diagonale. Masurile unghiurilor sale se exprima,ın grade, prin numere naturale impare. Demonstrati ca poligonul are cel putin douaunghiuri congruente.

Catalin Budeanu, Iasi

VII.121. In triunghiul ascutitunghic ABC, notam cu X,Y si Z mijloacele ınal-timilor [AA′], [BB′], respectiv [CC ′] si cu M,N si P mijloacele laturilor [BC], [CA],respectiv [AB]. Demonstrati ca dreptele XM,Y N si ZP sunt concurente.

Doru Buzac, Iasi

82

Page 87: Revista (format .pdf, 4.1 MB)

VII.122. Se considera dreptunghiul ABCD, cu AB = 1 si AD = 1 +√3, iar

M este un punct interior dreptunghiului, astfel ıncat m(ÖMDC) = m(ÖMCD) = 75.Aratati ca triunghiul MAB este echilateral.

Dumitru Mihalache, Barlad

Clasa a VIII-aVIII.116. Raportam planul la un reper cartezian xOy. Determinati multimea

punctelor M(x, y) din plan, pentru care 2px2 − y2 = x+ y.Romanta Ghita si Ioan Ghita, Blaj

VIII.117. Numerele reale pozitive x, y, z sunt astfel ıncat xy = z+1 six

z + 1+y =

2. Demonstrati cax

y+y

z≥ 3. Cand se atinge egalitatea?

Gheorghe Molea, Curtea de Arges

VIII.118. Fie x, y, z numere reale astfel ıncat x2 + 5yz ≥ 6, y2 + 5zx ≥ 6 siz2 + 5xy ≥ 6. Aflati valoarea minima a lui |x+ y + z|.

Dan Nedeianu, Drobeta-Tr. Severin

VIII.119. Determinati x ∈0,

5

4

si y ∈

−5x

2,x

2

pentru careÈ

(7− 2x)(5x+ 2y) +È

(x− 2y)(5− 4x) = 6.

Liviu Smarandache si Lucian Tutescu, Craiova

VIII.120. Rezolvati ın numere naturale ecuatia x+ 2y + 3z = 4xyz − 5.Titu Zvonaru, Comanesti

VIII.121. Demonstrati ca nu putem alege trei puncte necoliniare A,B,C, deaceeasi parte a unui plan α, astfel ıncat dreptele AB,BC si CA sa formeze cu planulα unghiuri de aceeasi masura nenula.

Petru Asaftei, Iasi

VIII.122. In fiecare patratel al unei table de sah este scris cate un numar natural.Fiecare numar n de pe tabla apare de cate n ori, iar pe primul rand al tablei apar,ordonat strict crescator, toate numerele folosite.

a) Aratati ca, dintre cele opt numere de pe primul rand, cel mult sase sunt pare.b) Determinati cel mai mare numar care poate aparea pe tabla.c) Aratati ca exista o singura modalitate de alegere a numerelor care apar pe tabla,

pentru care produsul numerelor de pe primul rand este impar.Silviu Boga, Iasi

Clasa a IX-aIX.106. Stabiliti valoarea de adevar a propozitiilor p si q, unde

p : (∃a ∈ Z)(∃n ∈ N∗)(a4 + 1 = 3n); q : (∃a ∈ Z)(∃n ∈ N)(a4 − 1 = 3n).

Dan Popescu, Suceava

83

Page 88: Revista (format .pdf, 4.1 MB)

IX.107. Daca a, b ∈ N∗, atunci

3a+ 4b

2a+ 3b−

√2

< 1

4

a+ 2b

a+ b−√2

< 1

16

ab−√2.

Mihail Bencze, Brasov

IX.108. Daca a ∈ (0, 1), demonstrati ca (a+ b)

1

a+

1

b− 4

(a+ 1)2

≥ 4

(a+ 1)2,

pentru orice b ∈ (0,∞).Ovidiu Pop, Satu Mare

IX.109. Demonstrati ca tg x > 4 sinx− 2, oricare ar fi x ∈h0,π

2

i.

Ionut Ivanescu, Craiova

IX.110. In ABC, cu notatiile uzuale, aratati ca OI ⊥ OIa ⇔ m( bA) = 60.Temistocle Bırsan, Iasi

Clasa a X-aX.106. Cele m × n patratele ale unui dreptunghi cu m linii si n coloane se

coloreaza cu p culori, unde m < p < n. Spunem ca o colorare are o taietura daca,pe una dintre cele n coloane, toate cele m patratele au aceeasi culoare. Determinatinumarul colorarilor care au k taieturi, unde 0 ≤ k ≤ n. (In legatura cu problema 2,OJM 2006.)

Cecilia Deaconescu, Pitesti

X.107. Se considera variabila aleatoare X :

1 0p q

, unde p, q ∈ (0, 1), p = q.

Aratati ca variabilele aleatoare |X −M(X)| si (X −M(X))2 sunt dependente.Laurentiu Modan, Bucuresti

X.108. Daca a, b, c ∈ (1,∞), demonstrati ca are loc inegalitatea

(logb a+ logc a) · (loga b+ logc b)(loga c+ logb c) ≥ 8 log b+c2a · log c+a

2b · log a+b

2c.

Lucian Tutescu, Craiova

X.109. Se considera multimile nedisjuncte A si B si functiile f : A → (0,∞),g : B → (0, 1). Determinati numarul a ∈ (0, 1) ∪ (1,∞) pentru care af(x)·g(x) +loga g(x) ≥ af(x), ∀x ∈ A ∩B.

Mihai Haivas, Iasi

X.110. Fie D = z = x+ iy|y > 0 multimea numerelor complexe din semiplanulsuperior, iar D′ = z = x + iy|x2 + y2 < 1 discul unitate (fara frontiera). Daca

z0 ∈ D este fixat, aratati ca functia f : D → D′, f(z) =z − z0z − z0

este bijectiva.

Adrian Corduneanu, Iasi

Clasa a XI-a

XI.106. Daca A ∈ M2(R), aratati ca det(A2 +A+ I2) + det(A2 −A+ I2) ≥3

2.

Dan Nedeianu, Drobeta Tr. Severin

XI.107. Se da parabola y = ax2(a > 0). In fiecare punct P (x, y) al parabolei,

se considera vectorul tangent−−→PP ′ =

−→i + 2a

−→xj si vectorul normal

−−→PQ′, orientat spre

84

Page 89: Revista (format .pdf, 4.1 MB)

exterior, astfel ıncat |−−→PQ′| = |

−−→PP ′|. Apoi, se considera vectorul

−−→PQ = α(x) ·

−−→PQ′,

unde α(x) este o functie data. Determinati locul geometric al punctului Q, atuncicand P descrie parabola, ın fiecare din cazurile:

a) α(x) = a,∀x ∈ R; b) α(x) = −1

a,∀x ∈ R; c) α(x) =

1

2ax,∀x ∈ R∗.

Adrian Corduneanu, Iasi

XI.108. Calculati limn→∞

( n√n)1+

1ln 2+

1ln 3+...+

1lnn .

Cezar Lupu, student, Bucuresti

XI.109. Determinati a, b, c ∈ (0,∞) pentru care limita limn→∞

a

n√b+ c

nexista

si este finita.Constantin Chirila, Iasi

XI.110. Date functiile continue f, g : R → R, sunt echivalente afirmatiile:i) f = g;ii) daca h : R → R este continua, ecuatia f(x) = h(x) are solutii reale atunci si

numai atunci cand ecuatia g(x) = h(x) are solutii reale.Marian Tetiva, Barlad

Clasa a XII-a

XII.106. Pentru a > 0 dat, calculati

Zax+ 2

x(a+ x2eax)dx, unde x ∈ (0,+∞)

I.V. Maftei, Bucuresti si Mihai Haivas, Iasi

XII.107. Fie Un =

Z π2

0

sin(2n+ 1)x

sinxdx si Vn =

Z π2

0

cos(2n+ 1)x

cosxdx, unde n ∈

N∗. Aratati ca Un = (−1)nVn =π

2, ∀n ∈ N∗.

Gheorghe Costovici, Iasi

XII.108. Demonstrati ca sirul (an)n≥1 definit prin an =

Z n

1

lnx

xp + 1dx, ∀n ≥ 1,

unde p ∈ (1,∞) este fixat, este convergent.Rodica Luca Tudorache, Iasi

XII.109. Fie (G, ·) un grup comuativ, cu proprietatea ca exista n ∈ N∗ astfelıncat din xn = yn, x, y ∈ G, rezulta ca x = y. Daca f, g sunt endomorfisme ale lui G,aratati ca ecuatia f(x) = g(x−1) are solutie unica ın G, daca si numai daca functiah : G→ G, h(x) = f(xn) · g(xn) este injectiva.

D.M. Batinetu-Giurgiu, Bucuresti

XII.110. Fie P,Q ∈ C[X] polinoame neconstante, astfel ıncat P si Q au aceleasiradacini, iar P − 1 si Q− 1 au si ele aceleasi radacini. Aratati ca P = Q.

Adrian Reisner, Paris

85

Page 90: Revista (format .pdf, 4.1 MB)

Probleme pentru pregatirea concursurilor

A. Nivel gimnazial

G176. Fie a1, a2, . . . , an ∈ R∗+, cu

1

a1+

1

a2+ . . .+

1

an= 1. Aratati ca

1

a31 + a22+

1

a32 + a23+ . . .+

1

a3n + a21<

1

2.

Angela Tigaeru, Suceava

G177. Fie k > 0 si a, b, c ∈ [0,+∞) astfel ıncat a + b + c = 1. Demonstrati caa

a2 + a+ k+

b

b2 + b+ k+

c

c2 + c+ k≤ 9

9k + 4.

Titu Zvonaru, Comanesti

G178. Daca n ∈ N\0, 1, aratati ca 1− n

n≤ nx − x ≤ n− 1

n, ∀x ∈ R.

Gheorghe Iurea, Iasi

G179. Determinati numerele prime a, b, c, d si numarul p ∈ N∗, astfel ıncat a2p

+b2

p

+ c2p

= d2p

+ 3.Cosmin Manea si Dragos Petrica, Pitesti

G180. Aflati restul ımpartirii numarului S = 20102009!+20092008!+ . . .+21!+10!

prin 41.Razvan Ceuca, elev, Iasi

G181. Fie k ≥ 1 un numar natural dat. Aratati ca exista o infinitate de numerenaturale n astfel ıncat nk divide n!.

Marian Tetiva, Barlad

G182. Se considera triunghiul ABC si punctele M ∈ [AB], N ∈ [BC], P ∈ [CA]astfel ıncat MP∥BC, iar MN∥AC. Fie Q = AN ∩ MP si T = BP ∩ MN .Demonstrati ca AAMN = APTN +AQPC .

Andrei Razvan Baleanu, elev, Motru

G183. Se considera triunghiul isoscel ABC, cu AB = AC si m( bA) < 30. Stiindca exista D ∈ [AB] si E ∈ [AC] astfel ıncat AD = DE = EC = BC, determinati

masura unghiului bA.Vasile Chiriac, Bacau

G184. In planul xOy, consideram punctele Aij de coordonate (i, j), unde i, j ∈0, 1, 2, 3, 4. Fie P multimea patratelor care au toate varfurile printre punctele Aijconsiderate. Aflati lungimea minima a unui drum care parcurge numai laturi alepatratelor din P si care uneste punctele A00 si A44.

Claudiu Stefan Popa, Iasi

G185. Aratati ca exista o colorare a planului cu n culori, unde n ≥ 2 este unnumar natural dat, astfel ıncat orice segment din plan sa contina puncte colorate cufiecare dintre cele n culori.

Paul Georgescu si Gabriel Popa, Iasi

86

Page 91: Revista (format .pdf, 4.1 MB)

B. Nivel licealL176. Fie D,E, F proiectiile centrului de greutate G al triunghiului ABC pe

dreptele BC,CA, respectiv AB. Aratati ca cevienele AD,BE si CF sunt concurentedaca si numai daca triunghul este isoscel.

Temistocle Bırsan, Iasi

L177. Consideram triunghiul ABC, G centrul sau de greutate, iar L punctul deintersectie al simedianelor. Notam cuM siN proiectiile luiG pe bisectoarea interioarasi pe cea exterioara ale unghiului bA, iar P si Q sunt proectiile lui L pe bisectoareleinterioara, respectiv exterioara, ale unghiului bA. Demonstrati ca dreptele GK,MNsi PQ sunt concurente.

Titu Zvonaru, Comanesti

L178. Fie ABCD un romb de latura 1 si punctele A1 ∈ (AB), B1 ∈ (BC), C1 ∈(CD), D1 ∈ (DA). Demonstrati ca A1B

21 +B1C

21 + C1D

21 +D1A

21 ≥ 2 sin2A.

Neculai Roman, Mircesti, Iasi

L179. Demonstrati ca ın orice triunghi are loc inegalitatea

2(9R2 − p2)

9Rr≥ cos2A

sinB sinC+

cos2B

sinC sinA+

cos2 C

sinA sinB≥ 1.

I.V. Maftei si Dorel Baitan, Bucuresti

L180. Determinati numarul minim de factori din produsul P = sinπ

4n· sin 2π

4n·

sin3π

4n· . . . · sin (22n−1 − 1)π

4n, n ∈ N∗, astfel ıncat P < 10−9.

Ionel Tudor, Calugareni, Giurgiu

L181. Fie P un punct de pe frontiera circulara a unui semidisc, iar d tangentaın P la aceasta frontiera. Notam cu C corpul de rotatie care se obtine prin rotireasemidiscului ın jurul dreptei d. Studiati variatia volumului lui C, functie de pozitiapunctului P .

Paul Georgescu si Gabriel Popa, Iasi

L182. Fie (xn)n≥1 un sir de numere ıntregi cu proprietatea ca xn+2−5xn+1+xn =0, ∀n ∈ N∗. Aratati ca, daca un termen al sirului se divide cu 22, atunci o infinitatede termeni au aceasta proprietate.

Marian Tetiva, Barlad

L183. Consideram numerele a ∈ Z, n ∈ N∗ si polinomul p(X) = X2 + aX + 1.Aratati ca exista un polinom cu coeficienti ıntregi qn si un numar ıntreg bn, astfelıncat p(X)qn(X) = X2n + bnX

n + 1.Marian Tetiva, Barlad

L184. Aratati ca functia f : N∗×N∗ → N∗, f(x, y) =x2 + y2 + 2xy − x− 3y + 2

2,

este bijectiva.Silviu Boga, Iasi

L185. Fie f : R → R o functie cu proprietatea ca |f(x+y)−f(x)−f(y)| ≤ |x−y|,∀x, y ∈ R. Aratati ca lim

x→0f(x) = 0 daca si numai daca lim

x→0xf(x) = 0.

Adrian Zahariuc, student, Princeton

87

Page 92: Revista (format .pdf, 4.1 MB)

Training problems for mathematical contests

A. Junior highschool level

G176. Let a1, a2, . . . , an ∈ R∗+, with

1

a1+

1

a2+ . . .+

1

an= 1. Prove that

1

a31 + a22+

1

a32 + a23+ . . .+

1

a3n + a21<

1

2.

Angela Tigaeru, Suceava

G177. Let k > 0 and a, b, c ∈ [0,+∞) such that a+ b+ c = 1. Prove that

a

a2 + a+ k+

b

b2 + b+ k+

c

c2 + c+ k≤ 9

9k + 4.

Titu Zvonaru, Comanesti

G178. If n ∈ N\0, 1, show that1− n

n≤ nx − x ≤ n− 1

n, ∀x ∈ R.

Gheorghe Iurea, Iasi

G179. Determine the prime numbers a, b, c, d and the number p ∈ N∗, so thata2

p

+ b2p

+ c2p

= d2p

+ 3.Cosmin Manea and Dragos Petrica, Pitesti

G180. Find the remainder of the division of S = 20102009!+20092008!+. . .+21!+10!

by 41.Razvan Ceuca, hight-school student, Iasi

G181. Let k ≥ 1 be a given natural number. Show that there are infinitely manynatural numbers n such that nk divides n!.

Marian Tetiva, Barlad

G182. The triangle ABC is considered with the points M ∈ [AB], N ∈ [BC],P ∈ [CA] such that MP∥BC and MN∥AC. Let Q = AN ∩ MP and T =BP ∩MN . Prove that AAMN = APTN +AQPC .

Andrei Razvan Baleanu, hight-school student, Motru

G183. Let ABC be an isosceles triangle, with AB = AC sim( bA) < 30. Knowingthat the points D ∈ [AB] and E ∈ [AC] exist such that AD = DE = EC = BC,

determine the measure of the angle bA.Vasile Chiriac, Bacau

G184. The points Aij of coordinates (i, j) are considered in the plane xOy,where i, j ∈ 0, 1, 2, 3, 4. Let P be the set of the squares with their vertices amongthe considered points Aij . Find the minimum length of a path consisting of squaresides only, which joins the points A00 and A44.

Claudiu Stefan Popa, Iasi

G185. Show that there exists a coloring of the plane by n colours, where n ≥ 2 isa given natural number, so that any line segment in phe plane contain points coloredby each of the n colours.

Paul Georgescu and Gabriel Popa, Iasi

88

Page 93: Revista (format .pdf, 4.1 MB)

B. Highschool LevelL176. Let D,E, F be the projections of the centroid G of the triangle ABC onto

the lines BC,CA, and respectively AB. Show that the Cevian lines AD,BE and CFmeet at a unique point if and only if the triangle is isosceles.

Temistocle Bırsan, Iasi

L177. We consider the triangle ABC with its centroid G and Lemoine′s point L.Denote by M and N the projections of G onto the interior and exterior bisector linesof angle bA and let P and Q be the projections of L on the interior and respectivelyexterior bisector lines of angle bA. Prove that the lines GK,MN and PQ meet at thesame point.

Titu Zvonaru, Comanesti

L178. Let ABCD be a rhombus with its side length ℓ = 1 and the pointsA1 ∈ (AB), B1 ∈ (BC), C1 ∈ (CD), D1 ∈ (DA). Prove that A1B

21 +B1C

21 +C1D

21+

D1A21 ≥ 2 sin2A.

Neculai Roman, Mircesti, Iasi

L179. Prove that the following inequality holds in any triangle:

2(9R2 − p2)

9Rr≥ cos2A

sinB sinC+

cos2B

sinC sinA+

cos2 C

sinA sinB≥ 1.

I.V. Maftei and Dorel Baitan, Bucuresti

L180. Determine the minimum number of factors in the product P = sinπ

4n·

sin2π

4n· sin 3π

4n· . . . · sin (22n−1 − 1)π

4n, n ∈ N∗, so that P < 10−9.

Ionel Tudor, Calugareni, GiurgiuL181. Let P be a point on the circular boundary of a half-disc, and d the tangent

at P to this boundary. Denote by C the rotation body obtained by the rotation ofthe half-disc around the line d. Study the variation of the volume of C as a functionof the position of point P.

Paul Georgescu and Gabriel Popa, IasiL182. Let (xn)n≥1 be a sequence of integer numbers with the property that

xn+2 − 5xn+1 + xn = 0, ∀n ∈ N∗. Show that if a term of the sequence is divisible by22 then infinitely many terms there of have this property.

Marian Tetiva, BarladL183. Let us consider the numbers a ∈ Z, n ∈ N∗ and the polynomial p(X) =

X 2 + aX +1. Show that there exist a polynomial with integer coefficients qn and aninteger number bn such that p(X)qn(X) = X2n + bnX

n + 1.Marian Tetiva, Barlad

L184. Show that the function f :N∗×N∗→N∗, f(x, y)=x2 + y2 + 2xy − x− 3y + 2

2,

is bijective.Silviu Boga, Iasi

L185. Let f : R → R be a function with the property that |f(x + y) − f(x) −f(y)| ≤ |x− y|, ∀x, y ∈ R. Show that lim

x→0f(x) = 0 if and only if lim

x→0xf(x) = 0.

Adrian Zahariuc, student, Princeton

89

Page 94: Revista (format .pdf, 4.1 MB)

Pagina rezolvitorilor

CRAIOVAColegiul National ”Fratii Buzesti”. Clasa a VI-a (prof. BALASOIU Ramona).

ENE Cristina: V(109,112), VI(111,112), VIII.113. Clasa a VI-a (prof. IONESCUMaria). VIRLAN Leonard: P(171-173), V(102,104,108).

Colegiul National ”Carol I”. Clasa a VII-a. RADULESCU Adrian: V(102,105-107), VI(104,107,108), VII(102,107).

IASIScoala nr. 3 ”Al. Vlahuta”. Clasa a II-a (inst. MAXIM Gabriela). CUCURUZ

Raluca: P(168,174-177); DASCALU Lorena: P(168, 174-177); NICA Daniel: P(168,174-177); POPESCU Alexandru: P(168, 174-177); ROBU Carmen: P(168, 174-177); SERBANOIU Alexandru: P(168, 174-177); TORAC George: P(168, 174-177).Clasa a III-a (ınv. MARIUTA Valentina). ENEA Codrut-Alexandru: P(175,177-180). Clasa a III-a (inst. CRACIUN Marilena). POPESCU Claudia: P(175,177-180). Clasa a V-a (prof. MARIN Mirela). CRETU Cristiana-Paula: P(181-183), V(109,114); IFTIME Ioana Evelina: P(181-183), V(100, 111,112); SAFIONElena Marina: P(181-183), V(109,114). Clasa a VII-a (prof. MARIN Mirela).ASAVEI Alexandra: VI(113,114), VII(109-112); CELMARE Raluca: VI(113,114),VII(109-112); MARCU Anca: VI(113,114), VII(109-112); TIBA Stefana-Alexandra:VI(113,114), VII(109-112).

Scoala nr. 11 ”Otilia Cazimir”. Clasa a III-a (inst. PARAIALA Dumitru).POPA Ioana-Maria: P(164-173), V(102,108), VI(108).

Scoala nr. 13 ”Alexandru cel Bun”. Clasa a II-a (inst. COJOCARIU Ana).ACATRINEI Andra: P(174-178); BEJAN Matei: P(174-178); BULEI Iasmina-Ioana:P(174-178); COSTIN Mihaita-Alexandru: P(174-178); MUNTIANU Ioana-Andreea:P(174-178); PERDUN Patricia-Maria: P(174-178); PRISECARU Alexandru-Iulian:P(174-178); SAMSON Constantin-Catalin: P(174-178); STEFAN Tudor: P(174-178);ZAHARIA Stefan-Eusebiu: P(174-178).

Scoala nr. 22 ”B.P. Hasdeu”. Clasa a II-a (ınv. NECHIFOR Doina). FETECAUMihai: P(171,174,177,178,184). Clasa a IV-a (inst. DOHOTARIU Liliana). CER-CEL Smaranda: P(164,166-169,171,173); CIOFU Alexandra: P(164,166-173); COPA-CEANU CEZARA: P(164-172); FOTEA Oana: P(164-167,172); GHEORGHITAMatei: P(164-173); HERGHELEGIU Andreea: P(164-173); JOHN Patricia: P(164-172); MANEA Amalia: P(164-173); MIHAI Ana-Maria: P(164-173); OLANUTACalin: P(168-173); PENESCU Teodora: P(164-172); ROTARU Andreea: P(164-167,170); RUSU George: P(164-167, 169,170,172,173); SENDRUC Sanziana: P(164-167, 169-172); TUDOSE Miruna: P(164-173); VERUZI Diana: P(164-171).

Scoala nr. 26 ”George Cosbuc”. Clasa a III-a (inst. VARLAN Elena). AMARI-EI Romeo: P(174-177,179,180); GHEBAN Andreea: P(174-177, 178,180); PICHIUCosmin: P(174-177, 178,180); TATARU Alice: P(174-177,178,180); TIPLEA Iu-lian: P(174-177,178,180); TOFAN Raluca: P(174-177, 178,180). Clasa a IV-a (ınv.BUCATARIU Rica). ANDONESEI Lucian: P(174-175,177-179,181-183); BARHANStefana-Adina: P(174-175,177-178,181-183); CHIRIAC Alexandra: P(174-175,177-179,181-183); CUPET Valeria: P(174-175,177-179,181-183); FRUNZA Diana-Mihaela:

90

Page 95: Revista (format .pdf, 4.1 MB)

P(174-175,177-179,181-183); FRUNZA Andrei-David: P(174-175,177-179,181-183);IVANOV Alexandra: P(174-175,177-179,181-183); MANDRU Liana: P(174-175,177-179,181-183); RADU Andrei: P(174-175,177-179,181-183).

Colegiul National Iasi. Clasa a V-a (prof. POPA Gabriel). BUDESCU An-drada Ioana: P.183, V(109,110,114,115); MORUZI Mark-Louis: P(181-183), V(109-111,114); VERINGA Alexandru: P(182,183), V(109,110,115), VI(109,110); VER-NICA Bianca Elena: P(182,183), V(109-111,114,115). Clasa a VII-a (prof. POPAGabriel). ASAFTEI Mircea: VI(109,110), VII(109-111); CIOBANU Stefan: VI(109,110,114), VII(109,111); MANGALAGIU Ioan: V(110-112,114), VI(113,114); MURGUGeorge: VI(109, 110), VII(109-111); PURICE Ioana: V.112, VI.115, VII(110,113,114).

PASCANIScoala ”Iordache Cantacuzino”. Clasa a II-a (ınv. MIRON Petru). CRACIUN

Stefana-Maria: P(164-173).

TIGANASI (IASI)Scoala ”M. Kogalniceanu”. Clasa a III-a (ınv. PATRASCU Carmen). CAZA-

DOI Ioana-Cristina: P(168,170,173-175); DUCA Cristina-Mihaela: P(168,170,173-175); SANDU Rebeca: P(168,170,173-175). Clasa a V-a (prof. CHIRITESCUAnca). DUCA Mirela Beatrice: P(171,173,181,182), VI.108; GANESCU Tudor:P(171-173,181-183); SANDU Codruta: P(171-173,181,183); VERNERMadalina-Geor-giana: P(171,172,181-183), V.105, VI.108. Clasa a VI-a (prof. CHIRITESCUAnca). BOROS Paula Mihaela: P(173,181-183), V.106, VI.108; DUCA Liliana Da-niela: P(171,172,181,183), V(102,105), VI.108; PIU Debora Roxana: P(171,181-183), V(105,106). Clasa a VII-a (prof. CHIRITESCU Anca). GAVRILAS Ma-rius Alexandru: V(102,105-107),VI.108; GANEANU Stefan Bogdan: V(102,105-107),VI.108; GHIOACA Oana: V(102,105-107), VI.108.

Elevi rezolvitori premiati

Scoala nr. 26 ”George Cosbuc”, Iasi

1. CHIRIAC Alexandra (cl. a IV-a): 1/2009(7pb), 2/2009(7pb), 1/2010(8pb).

2. IVANOV Alexandra (cl. a IV-a): 1/2009(6pb), 2/2009(7pb), 1/2010(8pb).

3. MANDRU Liana (cl. a IV-a): 1/2009(7pb), 2/2009(8pb), 1/2010(8pb).

91

Page 96: Revista (format .pdf, 4.1 MB)

IMPORTANT

• In scopul unei legaturi rapide cu redactia revistei, pot fi utilizate urmatoareleadrese e-mail: t [email protected] si [email protected] . Peaceasta cale colaboratorii pot purta cu redactia un dialog privitor la ma-terialele trimise acesteia, procurarea numerelor revistei etc. Sugeram cola-boratorilor care trimit probleme originale pentru publicare sa le numerotezesi sa-si retina o copie xerox a lor pentru a putea purta cu usurinta o discutieprin e-mail asupra acceptarii/neacceptarii acestora de catre redactia revistei.

• La problemele de tip L se primesc solutii de la orice iubitor de matematicielementare (indiferent de preocupare profesionala sau varsta). Fiecare dintresolutiile acestor probleme - ce sunt publicate ın revista dupa un an - va fiurmata de numele tuturor celor care au rezolvat-o.

• Adresam cu insistenta rugamintea ca materialele trimise revisteisa nu fie (sa nu fi fost) trimise si altor publicatii.

• Rugam ca materialele tehnoredactate sa fie trimise pe adresa redactieiınsotite de fisierele lor (de preferinta ın LATEX).

• Pentru a facilita comunicarea redactiei cu colaboratorii ei, autorii materi-alelor sunt rugati sa indice adresa e-mail.

92

Page 97: Revista (format .pdf, 4.1 MB)

Revista semestrială RECREAŢII MATEMATICE este editată de ASOCIAŢIA “RECREAŢII MATEMATICE”. Apare la datele de 1 martie şi 1 septembrie şi se adresează elevilor, profesorilor, studenţilor şi tuturor celor pasionaţi de matematica elementară.

În atenţia tuturor colaboratorilor Materialele trimise redacţiei spre publicare (note şi articole, chestiuni de

metodică, probleme propuse etc.) trebuie prezentate îngrijit, clar şi concis; ele trebuie să prezinte interes pentru un cerc cât mai larg de cititori. Se recomandă ca textele să nu depăşească patru pagini. Evident, ele trebuie să fie originale şi să nu fi apărut sau să fi fost trimise spre publicare altor reviste. Rugăm ca mate-rialele tehnoredactate să fie însoţite de fişierele lor.

Problemele destinate rubricilor: Probleme propuse şi Probleme pentru pregătirea concursurilor vor fi redactate pe foi separate cu enunţ şi demonstra-ţie/rezolvare (câte una pe fiecare foaie) şi vor fi însoţite de numele autorului, şcoa-la şi localitatea unde lucrează/învaţă.

Redacţia va decide asupra oportunităţii publicării materialelor primite. În atenţia elevilor Numele elevilor ce vor trimite redacţiei soluţii corecte la problemele din

rubricile de Probleme propuse şi Probleme pentru pregatirea concursurilor vor fi menţionate în Pagina rezolvitorilor. Se va ţine seama de regulile:

1. Pot trimite soluţii la minimum cinci probleme propuse în numărul prezent şi cel anterior al revistei; pe o foaie va fi redactată soluţia unei singure probleme.

2. Elevii din clasele VI-XII au dreptul să trimită soluţii la problemele propuse pentru clasa lor, pentru orice clasă mai mare, din două clase mai mici şi imediat anterioare. Elevii din clasa a V-a pot trimite soluţii la problemele propuse pentru clasele a IV-a, a V-a şi orice clasă mai mare, iar elevii claselor I-IV pot trimite soluţii la problemele propuse pentru oricare din clasele primare şi orice cla-să mai mare. Orice elev poate trimite soluţii la problemele de concurs (tip G şi L).

3. Vor fi menţionate următoarele date personale: numele şi prenumele, clasa, şcoala şi localitatea.

4. Plicul cu probleme rezolvate se va trimite prin poştă (sau va fi adus direct) la adresa Redacţiei:

Prof. dr. Temistocle Bîrsan Str. Aurora, nr. 3, sc. D, ap. 6, 700 474, Iaşi Jud. IAŞI E-mail: [email protected]

Page 98: Revista (format .pdf, 4.1 MB)

CUPRINS Centenarul SEMINARULUI MATEMATIC "A. MYLLER" (V. OPROIU) ............... 1 SEMINARUL MATEMATIC DIN IAŞI – 100 de ani de învăţământ matematic românesc (A. PATRAŞ)...............4 Amintiri de la SEMINARUL MATEMATIC (V. OPROIU)................................................6

ARTICOLE ŞI NOTE G. POPA – Rigla şi compasul .............................................................................................. 12 Gh. CIORESCU, A. SANDOVICI – O inegalitate ponderată cu medii........................... 18 F. POPOVICI – Inegalitatea lui Jensen pentru funcţii J-convexe în raport cu medii cvasiaritmetice .......... 21 V. CHIRIAC, B. CHIRIAC – Inegalitatea H ≤ G ≤ A revizitată..................................... 25 P. MINUŢ, C. SIMIRAD – O extensiune a şirului Fibonacci......................................... 27 L. TUŢESCU – Generalizarea unei identităţi şi aplicaţii ................................................... 31 M. TETIVA – Problema G128 - comentarii ......................................................................... 33

NOTA ELEVULUI R. CEUCĂ – O problemă de numărare ............................................................................... 35

CORESPONDENŢE A. REISNER – Un sous-ensemble particulier de matrices carrées ..................................... 37

CUM CONCEPEM... CUM REZOLVĂM M. TETIVA – O problemă complexă .................................................................................. 41

CHESTIUNI COMPLEMENTARE MANUALELOR I. PĂTRAŞCU – Axe şi centre radicale ale cercurilor adjuncte unui triunghi................ 45

ŞCOLI ŞI DASCĂLI V. PARASCHIV – Şcoala Normală "Vasile Lupu" din Iaşi – o istorie zbuciumată ...... 48

CONCURSURI ŞI EXAMENE Concursul "Recreaţii Matematice", ed. a VII-a, 2009 ........................................................ 51 Concursul de matematică "Gaudeamus", ed. I, 2009 .......................................................... 53

PROBLEME ŞI SOLUŢII

Soluţiile problemelor propuse în nr. 1/2009.......................................................................... 55 Soluţiile problemelor pentru pregătirea concursurilor din nr. 1/2009 ................................. 71 Probleme propuse..................................................................................................................... 80 Probleme pentru pregătirea concursurilor .............................................................................. 86 Training problems for mathematical contests ....................................................................... 88 Pagina rezolvitorilor .............................................................................................................. 90

ISSN 1582 – 1765 7 lei